76
Programme 2013 Nouvelle collection INDICE STMG Sous la direction de : Michel PONCY Denis VIEUDRIN Jean-Louis BONNAFET Catherine LEBERT Marie-Christine RUSSIER

Nouvelle collection INDICE - Le livre du prof · Programme 2013 Nouvelle collection INDICE STMG Sous la direction de : Michel PONCY Denis VIEUDRIN Jean-Louis BONNAFET Catherine LEBERT

  • Upload
    ngonga

  • View
    242

  • Download
    1

Embed Size (px)

Citation preview

Programme 2013

Nouvelle collection INDICE

STMG

Sous la direction de :

Michel PONCYDenis VIEUDRIN

Jean-Louis BONNAFETCatherine LEBERTMarie-Christine RUSSIER

2

Sommaire

2

CHAPITRE 1 Information chiffrée............................................................................................................................................. 3

CHAPITRE 2 Suites numériques ............................................................................................................................................... 10

CHAPITRE 3 Fonctions polynômes ..................................................................................................................................... 18

CHAPITRE 4 Fonctions rationnelles ................................................................................................................................... 30

CHAPITRE 5 Statistiques ...................................................................................................................................................................... 39

CHAPITRE 6 Probabilités conditionnelles ................................................................................................................ 47

CHAPITRE 7 Loi normale ...................................................................................................................................................................... 57

CHAPITRE 8 Échantillonnage et estimation ......................................................................................................... 66

Ensembles – Raisonnement logique ................................................................................................................................ 73

Fonctionnalités de base du tableur .................................................................................................................................... 74

© Bordas, SEJER 2013ISBN : 978-2-04-733069-2

3Chapitre 1 Information chiffrée

A

1CHAPITRE

Information chiffrée

Ce chapitre traite intégralement la partie « Information chiffrée » du programme de la classe. Il fait suite aux chapitres « Proportions » et « Évolutions » du programme de Première STMG. Beaucoup de notions ont donc déjà été abordées en classe de Première.Il nous a paru utile de revenir sur la notion de proportion, et surtout sur la notion de taux d’évolution qui est centrale dans ce chapitre et dans ses applications. La notion d’indice a été un peu abordée en Première, elle est ici développée en détail. Nous avons repris la notion d’évolutions successives car elle permet de parler de taux d’évolution global et ensuite de définir le taux d’évolution moyen, qui est une notion nouvelle pour les élèves en Terminale.Les Savoir-faire reprennent les quatre capacités attendues du programme. Nous avons ajouté cinq autres savoir-faire qui reprennent pour la plupart des capacités attendues du programme de Première.

Les notions abordées dans le chapitre 1 • Proportions• Taux d’évolution• Indices simples• Évolutions successives• Racine n-ième d’un réel positif• Taux d’évolution moyen

B Notre point de vue

Objectif : Consolider les acquis sur les notions de proportion et d'évolution en introduisant la notion d'indice en base 100 et la notion de taux d'évolution moyen.

Contenus Capacités attendues Commentaires

Indice simple en base 100. • Passer de l’indice au taux d’évolution, et réciproquement.

Le calcul d’un indice synthétique, comme par exemple l’indice des prix, n’est pas au programme.

Racine n-ième d’un réel positif.

Notation a1n .

Taux d’évolution moyen.

• Déterminer avec une calculatrice ou un tableur la solution positive de l’équation xn = a, lorsque a est un réel positif.• Trouver le taux moyen connaissant le taux global.

La notation n n’est pas exigible.

Exemple : taux mensuel équivalent à un taux annuel.

Le programme

C Avant de commencerVoir manuel page 250 et le site www.bordas-indice.fr pour les corrigés détaillés.Les notions abordées dans ces exercices permettent de réactiver les notions de base sur la proportionnalité et les pourcentages, en particulier celles déjà abordées en classe de Première STMG.

4

D ActivitésActivité Twitter en France

et dans le monde1

Cette activité a pour but de revenir sur les différents types de pourcentages rencontrés au cours des années précédentes : pourcentages d’une partie par rapport à un tout, et pourcentages d’évolution.1. a. En janvier 2012, pourcentage d’utilisateurs de Twitter en France : 1,4 % ; pourcentage d’utilisateurs de Twitter aux États-Unis : 28,1 %.b. En juillet 2012, pourcentage d’utilisateurs de Twitter en France : 1,4 % ;pourcentage d’utilisateurs de Twitter aux États-Unis : 27,4 %.2. a. Dans le monde : 35 %.b. En France : 40,4 %.3. a. Par 1,23.b. 41 millions environ.4. a. Par 1,93.b. 1,5 million environ.

Activité Le SMIC horaire brut en France2Cette activité est consacrée au SMIC horaire brut en France. Afin de comparer les diverses évolutions du SMIC au cours des années, on introduit un nouvel outil, déjà abordé en classe de Première : l’indice. Une propriété remarquable de la notion d’indice est conjecturée avec le tableur.Fichiers associés sur le site www.bordas-indice.fr et sur le CD-Rom Professeur :01_TSTMG_activite2.xlsx (Excel 2007),01_TSTMG_activite2.xls (Excel 2003),01_TSTMG_activite2.ods (OpenOffice).

1. a = 100 × 8,277,61

, soit a ≈ 108,7.

2. b ≈ 114,5.3. Indice du SMIC au 1er juillet 2010 : 116,4.Indice du SMIC au 1er juillet 2012 : 123,5.4. a. Le taux d’évolution est 0,235 à 10–3 près.b. Le taux d’évolution correspondant de l’indice est 0,235 à 10–3 près.5. On remarque que lorsque l’indice est 108,7, le taux d’évolution du SMIC depuis 2004 est 8,7 %. De même, lorsque l’indice est 114,5, le taux d’évolution du SMIC depuis 2004 est 14,5 %.Pratiquement, on peut donc déduire directement le taux d’évolution de l’indice en retranchant 100 à l’indice et en adjoignant au résultat trouvé le symbole %.

Activité Des puissances bizarres3Cette activité permet de découvrir la notion d’exposant 1

n: pour

cela, on revient dans un premier temps sur la notion connue de

racine carrée d’un nombre, et on étend ensuite la notion à un exposant 13 , puis à d’autres exposants de la forme 1

n.

Pour que cette notion ne reste pas abstraite pour les élèves, on applique ceci aux fréquences des notes de la gamme en musique.Fichiers associés sur le site www.bordas-indice.fr et sur le CD-Rom Professeur :01_TSTMG_activite3.xlsx (Excel 2007),01_TSTMG_activite3.xls (Excel 2003),01_TSTMG_activite3.ods (OpenOffice).1. a. c = 52 , soit environ 7,21 cm.b. On obtient le même résultat qu’à la question a.2. a. a = 4, car 43 = 64.b. On obtient le même résultat qu’à la question a.c. On cherche a tel que a3 = 315. Avec la calculatrice, on obtient : a ≈ 6,80.3. La calculatrice donne :231/3 ≈ 2,84 ; 451/4 ≈ 2,59 ; 721/5 ≈ 2,35.4. On remarque que, quand on calcule b = a1/n, puis bn, on retrouve toujours a.On peut donc conjecturer que (a1/n)n = a.5. a. k = 21/12 ≈ 1,059.b. La fréquence du ré est : 261,63 × k 2, soit environ 293,67 hertz.

Activité Évolution d’une population de bouquetins

4

Cette activité introduit la notion de taux d’évolution moyen sur un thème concret : l’évolution au cours des années d’une population de bouquetins dans un parc national.Fichiers associés sur le site www.bordas-indice.fr et sur le CD-Rom Professeur :01_TSTMG_activite4.xlsx (Excel 2007),01_TSTMG_activite4.xls (Excel 2003),01_TSTMG_activite4.ods (OpenOffice).1. a. 1,05 × 1,2 = 1,26 donc le taux global d’augmentation est 26 %.b. 1 + t = 1,26 , soit 1 + t ≈ 1,122 et t ≈ 0,122. Le taux t est environ 12,2 % à 0,1 % près.c. Il n’est pas égal à la moyenne des taux (cette moyenne est 12,5 %).2. a. 1,05 × 1,2 × 1,3 = 1,638. Donc le taux global d’augmentation est 63,8 %.b. 1 + t’ = 1,6381/3, soit t’ ≈ 0,179.Le taux t’ est de 17,9 % à 0,1 % près.c. Il n’est pas égal à la moyenne des taux, puisque celle-ci est de 18,3 % à 0,1 % près.3. On constate que le taux moyen n’est jamais égal à la moyenne des taux.

5Chapitre 1 Information chiffrée

E Exercices34 Les coefficients multiplicateurs sont 1,044 – 1,02 – 1,02.

Le taux global est 8,6 % à 0,1 % près.35 a. x = 13 ≈ 3,606. b. x = 81/3 = 2.

c. x = 471/4 ≈ 2,618.36 a. x = 881/3 ≈ 4,448. b. x = 2751/6 ≈ 2,550.

c. x = 6 3201/12 ≈ 2,074.37 Voir manuel p. 250.38 a. 1 + t = 2,51/4, soit t ≈ 0,257.

b. 1 + t = 0,21/4, soit t ≈ – 0,331.c. 1 + t = 1,51/5, soit t ≈ 0,084.d. 1 + t = 0,011/5, soit t ≈ – 0,602.39 a. 1 + t = 1,761/3, soit t ≈ 0,207.

b. 1 + t = 1,221/4, soit t ≈ 0,051.c. 1 + t = 0,851/6, soit t ≈ – 0,027.d. 1 + t = 2,51/10, soit t ≈ 0,096.40 a. 1 + t = 1,981/12, soit t ≈ 0,059.

b. 1 + t = 41/48, soit t ≈ 0,029.c. 1 + t = 0,41/24, soit t ≈ – 0,037.d. 1 + t = 0,11/30, soit t ≈ – 0,074.41 1. (1 + t)2 = 1 + 0,2 = 1,2.

2. 1 + t = 1,2 et t ≈ 0,095.

42 (1 + t)2 = 1 – 30100

= 0,7. D’où t ≈ – 0,163.

43 Voir manuel p. 250.

44 (1 + t)2 = 1 – 0,1 = 0,9. D’où t ≈ – 0,051.

45 (1 + t)12 = 1 + 24100

= 1,24. D’où t ≈ 0,018.

P o u r s ’ e n t r a î n e r46 1. Environ 3,1 %.

2. 8,73100

× 332 ≈ 28,98 : il y avait donc 29 nageurs français

engagés.47 Pourcentage des Coupes du monde :

– remportées par le Brésil : 26,3 % ;– remportées par l’Italie : 21,1 %.48 Environ 52,632 cL.49 1. 434 hommes.

2. Pourcentage d’hommes mineurs : 65 %.3. Pourcentage de femmes mineures : 10 %.50 Vrai.51 Faux : elle est vendue 84 €.52 1. Le taux d’évolution est 0,415, soit 41,5 %.

2. 19,205 milliers de boîtes, soit 19 205 boîtes.53 1. Pourcentage d’augmentation : 433,3 %.

2. 180 intérimaires.3. Taux d’évolution : 11, soit 1 100 %.54 Taux d’évolution : 4,333 soit 433,3 %.55 a. 45 030 euros.

b. 7 261,90 euros.

P o u r D É M a r r e r1 a. 60 b. 1,35 c. 150.2 15 %.3 40 %.4 Voir manuel p. 250.5 8,4864 soit 8,486 g à 0,001 près.6 1. 0,7 2. 0,2 3. 0,25 4. – 0,3757

Évolution Hausse de 12 %

Baisse de 12 %

Hausse de 90 %

Baisse de 1 %

Taux d’évolution 0,12 – 0,12 0,90 – 0,01

8

Taux d’évolution +0,43 – 0,22 +1,45 – 0,55

Évolution Hausse de 43 %

Baisse de 22 %

Hausse de 145 %

Baisse de 55 %

9 Voir manuel p. 250.10 a. 60 % b. 6 % c. –70 %

d. –5 % e. 1 % f. 120 %11 6,7 %, à 0,1 % près.12 20 %.13 Voir manuel p. 250.14 9 degrés.15 13,3 %, à 0,1 % près.16 35,9 %, à 0,1 % près.17 220 €.18 1,6 kg.19 Voir manuel p. 250.20 10 %.21 106.22 1. 20 %.

2. De 2009 à 2012 : 2 %.De 2011 à 2012 : 25,8 % à 0,1 % près.23 y = 1 368 ; I = 130.24 1,155.25 0,72.26 Voir manuel p.250.27 Il y a une baisse de 1 %, donc le nouveau prix est moins

élevé que le prix initial.28 1,2 × 1,3 = 1,56.

Le taux d’évolution global est de 56 %.29 1,1 × 0,92 = 1,012 : le taux d’évolution global est de 1,2 %.30 Les coefficients multiplicateurs sont 1,02 – 1,05 – 1,03.

Le taux global est 10,3 % à 0,1 % près.31 Le coefficient multiplicateur est 1,05 à chaque fois.

Le taux global est 15,8 %, à 0,1 % près.32 Les coefficients multiplicateurs sont 0,88 et 0,92.

Le taux d’évolution global est –19,04 % à 0,1 % près.33 Les coefficients multiplicateurs sont :

1,1 – 1,15 – 1,12 – 1,04.Le taux global est 47,3 % à 0,1 % près.

6

74 0,716 soit 71,6 %.75 61,1 %.76 Voir manuel p. 250.77 1. – 0,496 soit 49,6 % de diminution.

2. 250 habitants.78 1. 1 332 000

1 198 000 ≈ 1,11 : ceci justifie le taux de + 11 %.

1 380 0001 332 000

≈ 1,04 : ceci justifie le taux de + 4 %.

1 346 0001 380 000

≈ 0,975 : ceci justifie le taux de – 2,5 %.

2. Première façon : 1 346 0001 198 000

≈ 1,124 soit un taux de + 12,4 %.

Seconde façon : 1,11 × 1,04 × 0,975 ≈ 1,125 soit un taux de + 12,5 %.La différence provient des arrondis successifs dans la seconde méthode.79 Exercice résolu.80 1. C’est vrai.

2. Réciproque : « Si une quantité baisse après deux évolutions, alors les deux évolutions sont des diminutions. »Cette réciproque est fausse. Pour cela, il suffit de considérer une baisse de 20 % suivie d’une hausse de 10 %, qui conduit à une diminution de 12 %.81 1. a. t = – 0,22 donc il affiche « diminution ».

b. t = 0,008 donc il affiche « augmentation ».2. Cet algorithme permet de savoir si trois évolutions successives de taux donnés t1, t2 et t3 conduisent à une augmentation ou à une diminution.82 La baisse est de 19,64 % dans le magasin A, et de 19,01 %

dans le magasin B : c’est le magasin A le plus avantageux.83 Le coefficient multiplicateur sur 35 ans est 1,0235 ≈ 2.

Le coefficient multiplicateur sur 70 ans est 1,0270 ≈ 4.Le coefficient multiplicateur sur 100 ans est 1,02100 ≈ 7,2.Les affirmations de l’auteur sont donc exactes.84 Exercice résolu.85 Faux : l’augmentation est de 56 %.86 Faux : elle est compensée par une baisse de 1 %.87 Faux : elle double après 2 h 30 environ.88 1. Ce taux est – 0,183, soit une diminution de 18,3 %.

2. Le taux moyen annuel est – 0,096, soit une diminution de 9,6 %.89 1. Le taux est 0,259, soit une augmentation de 25,9 %.

2. Le taux moyen annuel est 0,080, soit une augmentation de 8 %.90 1. Le taux est – 0,113, soit une diminution de 11,3 %.

2. Le taux moyen annuel est – 0,039, soit une diminution de 3,9 %.91 1. Le taux est 0,341, soit une augmentation de 34,1 %.

2. Le taux moyen annuel est 0,030, soit une augmentation de 3 %.92 1. Le taux est – 0,75, soit une diminution de 75 %.

2. Le taux moyen annuel est – 0,007, soit une diminution de 0,7 %.93 1. Le taux est 0,024, soit une augmentation de 2,4 %.

2. Le taux moyen annuel est 0,002, soit une augmentation de 0,2 %.

56 Fichiers associés sur le site www.bordas-indice.fr et sur le CD-Rom Professeur :01_TSTMG_exercice56.xls (Excel 2003),01_TSTMG_exercice56.xlsx (Excel 2007),01_TSTMG_exercice56.ods (OpenOffice).Voir manuel p. 250.57 Fichiers associés sur le site www.bordas-indice.fr et sur

le CD-Rom Professeur :01_TSTMG_exercice57.xls (Excel 2003),01_TSTMG_exercice57.xlsx (Excel 2007),01_TSTMG_exercice57.ods (OpenOffice).1. Taux d’évolution : 0,012 soit 1,2 %.

2. = C2*1,025 .58 1. De 2006 à 2007 : – 0,019 soit – 1,9 % ;

de 2007 à 2008 : – 0,075 soit – 7,5 % ;de 2008 à 2009 : – 0,003 soit – 0,3 % ;de 2009 à 2010 : – 0,063 soit – 6,3 % ;de 2010 à 2011 : – 0,007 soit – 0,7 %.2. Ces chiffres montrent qu’il y a eu deux arrêts : l’un en 2009 et l’autre en 2011.59 Exercice résolu.60 1. t = 0,24.

2. Il calcule le taux d’évolution d’une grandeur lorsque celle - ci évolue de la valeur n à la valeur n’.61 Faux : elle correspond à un triplement du volume.62 Faux : la baisse est d’environ 9 %.63 Vrai : le taux d’évolution est bien égal à 12.64 1. L’indice est environ 101,7 (à 0,1 près).

2. Montant des droits : 180,96 €.65 Indice en 2008 : 109,8 à 0,1 près.

Indice en 2009 : 126,1 à 0,1 près.Indice en 2010 : 139,3 à 0,1 près.66 Indice en 2005 : 100.

Indice en 2008 : 181.Indice en 2012 : 159,28.67 Voir manuel p. 250.68 1. Pourcentage d’augmentation : de 2009 à 2010 : 1 % ;

de 2009 à 2011 : 3,3 % ; de 2009 à 2012 : 6,9 %.2. Augmentation de 3,5 %.69 1.

Marques Peugeot Citroën Renault2010 100 81,8 124,2

2011 100 87,3 123,2

2. Indices 2011 : 92,3 pour Peugeot, 98,5 pour Citroën et 91,6 pour Renault.70 a. 23 % ; b. 2 % ; c. – 13 %.71 1. C’est vrai.

2. Réciproque : « Si l’indice de cet aliment au 01/01/2013 est 90, alors le prix de cet aliment a diminué de 10 % du 01/01/2012 au 01/01/2013 ».Cette proposition réciproque est vraie.72 Vrai.73 Vrai : cela arrive si la valeur de la grandeur devient égale

à 0.

7Chapitre 1 Information chiffrée

111 Indice en août 2011 : 103,9.

Indice en août 2012 : 86,2.112 Indice en 1980 : 49,5.

Indice en 2008 : 30,4.

Le taux d’évolution est – 0,696, soit une diminution de 69,6 %.

t r aVau X P r at I Q u e s

TP Euros constants de 1992 à 2011Fichiers associés sur le site www.bordas-indice.fr et sur le

CD-Rom Professeur :

01_TSTMG_TP.xls (Excel 2003),

01_TSTMG_TP.xlsx (Excel 2007),

01_TSTMG_TP.ods (OpenOffice),

01_TSTMG_INSEE.xls (Excel 2003),

01_TSTMG_INSEE.xlsx (Excel 2007),

01_TSTMG_INSEE.ods (OpenOffice),

01_TSTMG_TP_correction.xls (Excel 2003),

01_TSTMG_TP_correction.xlsx (Excel 2007),

01_TSTMG_TP_correction.ods (OpenOffice).

Il est toujours difficile de comparer les prix d’une même grandeur

d’une année à l’autre, car le coût de la vie se modifie et certains

prix suivent plus ou moins cette évolution.

Pour comparer des prix, la façon classique est d’utiliser l’indice

des prix à la consommation et de calculer tous les prix en euros

constants pour une année fixée. Ce calcul utilise la notion d’indice

vue dans ce chapitre.

On pourra aussi, à la fin du TP, vérifier ses résultats en utilisant la

table des euros constants éditée chaque année par l’ INSEE. Cette

table intègre les variations du pouvoir d’achat des Français.

A. Indice des prix à la consommation

1. =(B2*100)/123,7 .

2. En 1998, l’indice des prix est :

100 ×100123,7 ≈ 80,84.

B. Quelques prix en euros constants

1. a. 1,15 €.

b. 19,06 €.

c. Si l’on raisonne en euros constants 2011, l’augmentation du

prix de l’essence sans plomb est forte, alors que le prix de la

coupe de cheveux est resté stable.

2. a. Le SMIC horaire brut pour 1992 est égal en euros constants

2011 à :5,08 ×100

73,65 ≈ 6,90 €.

b. =D2*100/C2 .

c. =F2*100/C2 .

94 Le taux mensuel est 0,2 %.95 Exercice résolu.96 1. Le taux est – 0,185, soit une diminution de 18,5 %.

2. Le taux moyen annuel est – 0,020, soit une diminution de 2 %.97 Fichiers associés sur le site www.bordas-indice.fr et sur

le CD-Rom Professeur :01_TSTMG_exercice97.xls (Excel 2003),01_TSTMG_exercice97.xlsx (Excel 2007),01_TSTMG_exercice97.ods (OpenOffice).1. =(B3-B2)/B2 .2. Le taux est 0,737, soit une augmentation de 73,7 %.3. Le taux décennal moyen est 0,096, soit une augmentation de 9,6 %.98 1. Le taux est – 0,04964, soit une diminution de 4,964 %.

2. Le taux moyen annuel t vérifie (1 + t)28 = 0,95036, ce qui donne t ≈ – 0,00182. La diminution moyenne est donc de 0,182 %.3. 46,91 × 0,9981811 ≈ 45,98 donc une estimation de ce record en 2020 est 45,98 s.99 1. Le coefficient multiplicateur global est :

1,047 × 1,106 × 1,041 × 1,058 × 1,075 ≈ 1,371.Le taux annuel moyen t vérifie (1 + t)5 = 1,371 soit t ≈ 0,065, soit une augmentation de 6,5 %.2. 67,5 milliards d’euros.100 1. Taux global : 0,5 c’est-à-dire 50 %.2. Taux annuel moyen : 0,041, soit 4,1 %.3. 16 288,95 euros.101 Faux : le taux mensuel moyen est 0,9 %.102 Vrai.103 1. 48,7 % . 2. 1 885 spectateurs.104 Taux d’évolution du patrimoine des moins fortunés : 0,092 soit 9,2 %.Taux d’évolution du patrimoine des plus fortunés : 0,476 soit 47,6 %.105 10,7 %.106 D’août 2005 à août 2012 : 267,6 %.D’août 2009 à août 2012 : 98,2 %.107 1. Le taux est – 0,707 soit une baisse de 70,7 %.2. Le taux annuel moyen de diminution est 3 %.108 Correctif : il faut lire « Le pourcentage de voix exprimées… »Le pourcentage de voix exprimées est de 22,8 % au premier tour et de 23,2 % au second tour : il a donc augmenté.

P o u r Fa I r e L e P o I n tVoir manuel page 223. Les corrigés détaillés sont disponibles sur le site www.bordas-indice.fr.

aCCo M PaG n e M e n t P e r s o n n a L I s É109 10,7 %.110 1 362,2 %.

8

Calculatrices Casio

Logiciel AlgoBox

Avec l’hypothèse faite, la barre des 9 secondes sera franchie en 2054.

116 1. N(x) = 200x + 20 000.

2. D(x) = 1+ x + 3100( )N(x) = 20 600 + 406x + 2x 2.

3. L’équation x 2 + 203x – 410 = 0 a pour solution valide x = 2.Le taux d’augmentation entre octobre et novembre est donc 2 %, et celui entre novembre et décembre est 5 %.4. t ≈ 3,489.

C a P V e r s L e B aC

Zoom sur le tableur

Fichiers associés sur le site www.bordas-indice.fr et sur le CD-Rom Professeur :01_TSTMG_zoomtableur.xls (Excel 2003),01_TSTMG_zoomtableur.xlsx (Excel 2007),01_TSTMG_zoomtableur.ods (OpenOffice).

Sujet A1. 1 717 milliers de clients.2. Le taux (arrondi à 1 %) est 12 %.3. a. Le taux est 87 %.b. Le taux moyen annuel est 23 %.c. 21 795 milliers de clients.4. La supposition est peu pertinente car le taux moyen de 23 % provient d’augmentations très fortes entre 2004 et 2005 (44 %) et entre 2005 et 2006 (34 %) ; les derniers taux d’évolution (24 % puis 12 %) montrent un net ralentissement des évolutions annuelles.

Sujet B1. 488 000 euros.2. Le taux est 0,713, soit 71,3 %.3. a. Le taux annuel moyen est 14,4 %.b. 588 000 euros.

3. a.

1992 1994 1996 1998 2000 2002 2004 2006 2008 2010 20120

1

2

3

4

5

6

7

8

9

10

SmicBaguette

b. On constate sur le graphique que le SMIC a progressé alors que le prix de la baguette est resté stable.4. Lorsque l’on utilise la seconde table du fichier de l’INSEE, on constate qu’à la ligne 97 de ce tableau, 1 franc 1992 vaut 0,20694 euros 2011 (colonne W de la table).En appliquant la conversion « un euro = 6,55957 francs », on en déduit que le SMIC de 1992 égal à 5,08 € équivaut à 33,32 F (en multipliant par 6,55957) et donc à 6,90 € en 2011 (en multipliant par le coefficient 0,20694 de la table).

P o u r a P P r o F o n D I r113 1. En 1999 : 65,7 % ; en 2010 : 63,2 %.2. a. + 0,8 % ; b. – 9,6 %.3. Pour les hommes : – 33,3 % ; pour les femmes : + 25 %.114 1. Point A : il traduit le fait que l’augmentation du chiffre d’affaires entre 2008 et 2009 a été nulle (0 %).Point B : il traduit le fait que la diminution du chiffre d’affaires entre 2010 et 2011 a été de 4 %.2. Non, car il a augmenté de 2 % de 2007 à 2008, et il est resté stable entre 2008 et 2009.3. a. 146,5 milliers d’euros ; b. 144,9 milliers d’euros ;c. 146,1 milliers d’euros.

115 Fichier associé sur le site www.bordas-indice.fr et sur le CD-Rom Professeur :01_TSTMG_exercice115. alg (AlgoBox).

1. 9,58 – 9,869,86

≈ – 0,0284, soit un pourcentage de

diminution de 2,84 %.2. (1 + t)21 = 0,9716 ce qui donne t ≈ 0,0014, soit un pourcentage moyen de diminution de 0,14 %.3. a. T prend la valeur T × 0,9986.b. Calculatrices Texas

9Chapitre 1 Information chiffrée

2. a. Le taux d’évolution est 0,353.b. Le taux hebdomadaire moyen est 16,3 %.3. a. 107 000 personnes.b. 168 000 personnes.4. a. I prend les valeurs 1 et 2.– Quand I prend la valeur 1, alors A prend la valeur 107 000 et cette valeur est affichée.– Quand I prend la valeur 2, alors A prend la valeur 124 000 et cette valeur est affichée.b. Cet algorithme affiche le nombre de personnes contaminées au bout de N semaines.

Sujet GFichiers associés sur le site www.bordas-indice.fr et sur le CD-Rom Professeur :01_TSTMG_sujetG.xls (Excel 2003),01_TSTMG_ sujetG.xlsx (Excel 2007),01_TSTMG_ sujetG.ods (OpenOffice).1. La bonne réponse est la réponse A.2. La bonne réponse est la réponse C.3. La bonne réponse est la réponse C.

Sujet HFichiers associés sur le site www.bordas-indice.fr et sur le CD-Rom Professeur :01_TSTMG_sujetH.xls (Excel 2003),01_TSTMG_ sujetH.xlsx (Excel 2007),01_TSTMG_ sujetH.ods (OpenOffice).1. On peut saisir en A3 la formule : =A2+3 .2. Le nombre de pauvres en 2005 était environ 1,38 milliards.3. Le taux de pauvreté en 2008 est 22,4 %.4. Le pourcentage de diminution du taux de pauvreté absolue entre 1981 et 2005 est 51,9 %.5. Le taux d’évolution moyen annuel est environ – 0,030, soit – 3 % à 0,1 % près.

117 L’action vaut 72 € en août.De janvier à août, son pourcentage de baisse est égal à 64 %.118 Pour le magasin A, la baisse globale est de 19,64 %. Pour le magasin B, la baisse globale est de 19,01 %.Le magasin le plus avantageux est le magasin A.119 L’indice du nombre de longs-métrages est : 95,8 en 2009 ; 108,7 en 2010 et 112,5 en 2011.120 Le taux d’évolution de la consommation de yaourts entre 1998 et 2008 est égal à 0,124, c’est-à-dire 12,4 %.Le taux d’évolution annuel moyen est 0,012, soit 1,2 %.

Sujet CFichiers associés sur le site www.bordas-indice.fr et sur le CD-Rom Professeur :01_TSTMG_sujetC.xls (Excel 2003),01_TSTMG_ sujetC.xlsx (Excel 2007),01_TSTMG_ sujetC. ods (OpenOffice).1. La part de la puissance des éoliennes est 7,1 %.2. a ≈ 207.3. a. =(B3-B2)/B2 .b. Le taux est 306,8 %.c. Le taux annuel moyen est 19,2 %.

Sujet DFichiers associés sur le site www.bordas-indice.fr et sur le CD-Rom Professeur :01_TSTMG_sujetD.xls (Excel 2003),01_TSTMG_ sujetD.xlsx (Excel 2007),01_TSTMG_ sujetD.ods (OpenOffice).1. a. 14,8 %. b. 2,3 %.2. a. 38,1 %. b. 5,5 %.3. a. La formule est =B3/B2 .b. Dans H4, il s’affiche 17,2 %.c. Le taux moyen de recyclage de 2001 à 2007 est 3,9 %. Si l’on suppose que ce taux moyen se maintient entre 2007 et 2012, le taux de recyclage en 2012 serait de 20,8 %.Cet objectif n’est donc pas réaliste.

Sujet EFichiers associés sur le site www.bordas-indice.fr et sur le CD-Rom Professeur :01_TSTMG_sujetE.xls (Excel 2003),01_TSTMG_ sujetE.xlsx (Excel 2007),01_TSTMG_ sujetE.ods (OpenOffice).1. La bonne réponse est la réponse C.2. La bonne réponse est la réponse C.3. La bonne réponse est la réponse A.4. La bonne réponse est la réponse B.

Sujet FFichiers associés sur le site www.bordas-indice.fr et sur le CD-Rom Professeur :01_TSTMG_sujetF.alg (AlgoBox).1. a. Le taux d’évolution est 0,966, soit en pourcentage : 96,6 %.b. On trouve environ 34 588. En arrondissant au millier, le nombre de personnes est 35 000.

10

A Le programmeObjectifs : – Approfondir les connaissances sur les suites arithmétiques et géométriques.– Utiliser de façon complémentaire les différents outils de calcul et de représentation (à la main, à la calculatrice, au tableur…) et l’algorithmique.

Contenus Capacités attendues Commentaires

Suites arithmétiques et géométriques

Expression du terme général.

Comparaison de suites.

• Écrire le terme général d'une suite arithmétique ou géométrique définie par son premier terme et sa raison.◊ Calculer avec la calculatrice ou le ta-bleur la somme de n termes consécutifs (ou des n premiers termes) d’une suite arithmétique ou géométrique.

• Dans le cadre de résolution de pro-blèmes, comparer deux suites géomé-triques, une suite géométrique et une suite arithmétique.

Pour les suites géométriques, on se limite aux suites à termes strictement positifs.Pour certaines résolutions, le tableur est indispensable.L’expression de la somme de n termes consécutifs n’est pas un attendu du programme.Exemples : emprunt à annuités constantes, valeur actuelle d’une suite d’annuités constantes.

Exemples : intérêts simples, intérêts composés ; taux équivalent, taux proportionnel

B Notre point de vueCe chapitre traite des notions de suites arithmétiques et de suites géométriques qui ont déjà été abordées en classe de Première. Le chapitre est scindé en deux séquences, l’une consacrée à l’étude des suites arithmétiques et à l’obtention de termes consécutifs d’une suite avec la calculatrice ou le tableur, l’autre consacrée à l’étude des suites géométriques et aux comparaisons de différentes suites.L’activité 1 permet, à partir d’un problème de tarification de type affine, de revoir la notion de suite arithmétique et d’en découvrir l’expression du terme général. Dans la partie cours, nous donnons deux formules du terme général d’une suite arithmétique : l’une en fonction du terme d’indice 0, l’autre en fonction du terme d’indice 1. Un savoir-faire est consacré à l’obtention de l’expression du terme général ainsi que la première partie de la page dédiée à l’accompagnement personnalisé.L’activité 2 permet de travailler sur la notion de somme de termes consécutifs d’une suite arithmétique, et de voir l’intérêt de calculer de telles sommes dans un problème de calculs de salaires cumulés. Dans la partie cours, nous présentons différentes méthodes pour obtenir ce genre de sommes avec le tableur ou la calculatrice, puis deux savoir-faire sont consacrés à cette notion. Pour compléter le cours sur cette notion de somme, deux pages TICE (page 46 et 47) lui sont dédiées.Conformément à l’esprit du programme, le quatrième savoir-faire est consacré à un problème de modélisation.Dans la seconde séquence, l’activité 3 permet de revoir la notion de suite géométrique et d’établir l’expression du terme général. Comme pour les suites arithmétiques, nous donnons dans la partie cours deux formules du terme général,

Suites numériques2CHAPITRE

11Chapitre 2 Suites numériques

l’une en fonction du terme d’indice 0 et l’autre en fonction du terme d’indice 1. Nous consacrons un savoir-faire à cette notion, ainsi que la seconde partie de la page dédiée à l’accompagnement personnalisé.Nous n’avons pas repris dans le cours les techniques d’obtention d’une somme de termes consécutifs d’une suite géométrique, puisqu’elles sont identiques à celles utilisées pour les suites arithmétiques, mais nous y consacrons un savoir-faire.Dans l’activité 4, nous comparons l’évolution de trois suites croissantes : deux suites géométriques et une suite arithmétique. Il s’agit de faire sentir aux élèves l’ordre qui s’établit pour les grandes valeurs de l’indice des suites, et que cet ordre ne dépend ni du premier terme, ni de la raison pour les suites arithmétiques. Sans leur donner l’importance d’une propriété, nous reprenons ces résultats dans le cours, puis dans un savoir-faire.Dans les pages consacrées aux exercices ou le Cap vers le Bac, nous avons intégré de nombreuses questions relatives à l’utilisation du tableur, et comme dans tous les chapitres du livre une page permet à l’élève de travailler les compétences requises au baccalauréat.Nous avons évidemment intégré des exercices d’algorithmique. Conscients de la difficulté que peut poser ce genre d’exercices à un élève de STMG, nous avons essentiellement travaillé la compréhension d’un algorithme donné, l’élève étant parfois amené à modifier ou compléter cet algorithme.Un TP permet de travailler les différentes notions rencontrées dans ce chapitre, notamment la comparaison et l’obtention de la somme de termes consécutifs.

Les notions abordées dans le chapitre 2• Suites arithmétiques• Suites géométriques• Somme de termes consécutifs d’une suite• Comparaison de suites

C Avant de commencerVoir manuel pages 250 et 251 et le site www.bordas-indice.fr pour les corrigés détaillés.Fichiers associés sur le site www.bordas-indice.fr et sur le CD-Rom Professeur :02_TSTMG_p36.xls (Excel 2003),02_TSTMG_p36.xlsx (Excel 2007),02_TSTMG_p36.ods (OpenOffice).

D ActivitésActivité Garde d’enfants1

Cette activité permet de revoir la notion de suite arithmétique déjà vu en classe de Première et de découvrir la formule du terme général d’une telle suite.1. u2 = 150 + 2 × 20 = 150 + 40 = 190.u3 = 150 + 3 × 20 = 150 + 60 = 210.2. u451 = u450 + 20 = 9 170.u452 = u451 + 20 = 9 190.3. un+1 = un + 20 pour tout entier naturel n donc la suite (un) est une suite arithmétique de raison 20.4. a. 320 × 20 = 6 400. Pour 320 heures de garde, la famille Jeanblanc débourse 6 400 euros.b. Pour n heures de garde, la famille débourse 20n.c. un = 150 + 20n.

Activité Une progression de salaire2Fichiers associés sur le site www.bordas-indice.fr et sur le CD-Rom Professeur :02_TSTMG_activite2.xls (Excel 2003),02_TSTMG_ activite2.xlsx (Excel 2007),02_TSTMG_ activite2.ods (OpenOffice). Dans cette activité, on travaille sur la notion de somme de termes consécutifs d’une suite arithmétique que l’on obtient à l’aide de la fonction SOMME du tableur.1. a2 = 1 500 + 7 = 1 507 ; a3 = 1 507 + 7 = 1 514.2. a1 + a2 + a3 = 4 521. Au cours du premier trimestre 2012, Manon a perçu 4 521 euros.3. an+1 = an + 7 ; la suite (an) est arithmétique de raison 7.4. Pour tout entier naturel n, an = a1 + (n – 1) × 7 donc an = 1 493 + 7n.

12

Activité Petite épargne deviendra grande4Fichiers associés sur www.bordas-indice.fr et sur le CD-Rom Professeur : 02_TSTMG_activite4.xls (Excel 2003),02_TSTMG_ activite4.xlsx (Excel 2007),02_TSTMG_ activite4.ods (OpenOffice).Il s’agit dans cette activité de comparer les évolutions de trois suites croissantes : une suite arithmétique et deux suites géométriques puis de conjecturer un résultat plus général à l’aide du tableur.1. an+1 = an + 35. La suite (an) est arithmétique de raison 35 et de premier terme a0 = 650 donc an = 650 + 35n.2. bn+1 = 1,05bn. La suite (bn) est géométrique de raison 1,05 et de premier terme b0 = 600 donc bn = 600 × 1,05n.cn+1 = 1,06cn, la suite (cn ) est géométrique de raison 1,06 et de premier terme c0 = 500 donc cn = 500 × 1,06n.3. a10 = 1 000 ; b10 ≈ 977,34 et c10 ≈ 895, 42. Alice disposera du capital le plus élevé pour son dixième anniversaire.4. a14 = 1 140 ; b14 ≈ 1 187,96 et c14 ≈ 1 130,45. Pour son quatorzième anniversaire, c’est Bilal qui disposera du plus gros capital.a18 = 1 280 ; b18 ≈ 1 443,97 et c18 ≈ 1 427,17. À dix-huit ans, c’est Bilal qui disposera du plus gros capital.À vingt ans, c’est Clément qui disposera du plus gros capital puisque a20 ≈ 1 350, b20 ≈ 1 591,98 et c20 ≈ 1 603,57.5. Voir pages 246 et 248 du manuel.6. Pour 0 � n � 11, an � bn � cn.Pour 12 � n � 14 , bn � an � cn.Pour 15 � n � 19, bn � cn � an.Enfin c20 � b20 � a20.7. Il semble que les suites (bn) et (cn) « dépassent » la suite (an) à partir d’un certain rang qui dépend de la raison de (an).

5. a.

n 1 2 3 4 5 6 7 8 9 10 11 12

an 1 500 1 507 1 514 1 521 1 528 1 535 1 542 1 549 1 556 1 563 1 570 1 577

b. Le salaire annuel pour la première année est 18 462.

6. a. Les salaires mensuels de l’année 2013 sont affichés

dans la zone de cellules C14:C25 : on saisit la formule =SOMME(C14:C25) .

b. Les salaires mensuels des cinq premières années sont

affichés dans la zone de cellules C2:C61 : on saisit la formule =SOMME(C2:C61) .

Activité Les centenaires se portent bien3Cette activité permet de revoir la notion de suite géométrique

déjà vu en classe de Première et de découvrir la formule du terme

général d’une telle suite.

1. C1 = 2 × C0 = 30 000.

2. C2 = 2 × C1 = 60 000.

3. Cn+1 = 2 × Cn. La suite (Cn) est géométrique de raison 2.

4. Il y a doublement toutes les décennies, donc dans cinq

décennies le nombre de centenaires aura doublé cinq fois :

C5 = C0 × 2 × 2 × 2 × 2 × 2 = C0 × 25 = 480 000.

5. a. Dans n décennies, la population de centenaires aura

doublé n fois.

b. Cn = C0 × 2 × 2 × … × 2 = C0 × 2n = 15 000 × 2n.

c. C2 = C0 × 2² = 60 000 ;

C5 = C0 × 25 = 480 000.

n fois

}E Exercices

13 La suite est croissante.14 Indice du onzième terme : 10.15 Indice du trentième terme : 29.16 Indice du quinzième terme : 15.17 Voir manuel page 250.18 1. un = u0 + nr = −7 + n × 2 = −7 + 2n.

2. u25 = 43.19 1. un = u1 + (n − 1)r = 9 + (n − 1) × 3 = 6 + 3n.

2. u100 = 306.20 1. vn = v1 + (n − 1)r = 15 + (n − 1) × ( −5) = 20 − 5n.

2. v40 = −180.21 1. u1 = 8 , u2 = 13 , u3 = 18, u4 = 23.

2. u0 + u1 + u2 + u3 + u4 = 65.22 1. Indice du premier terme 0 ; indice du dernier terme 12.

2. Indice du premier terme 1 ; indice du dernier terme 13.23 Voir manuel page 250.

P O U R D É M A R R E R1 u1 = 6 ; u2 = 11 ; u3 = 16 ; u4 = 21 et u5 = 26.2 u1 = 7 ; u2 = 5 ; u3 = 3 ; u4 = 1 et u5 = −1.3 u1 = 8,5 ; u2 = 9 ; u3 = 9,5 ; u4 = 10 et u5 = 10,5.4 u1 = 10,8 ; u2 = 9,6 ; u3 = 8,4 ; u4 = 7,2 et u5 = 6.5 1. v2 = 9, v3 = 6 et v4 = 3.

2. On a v5 = 0 donc v6 = −3.6 r = 6.7 r = −111.8 r = 9.9 r = −15.10 Les quatre premiers termes ne sont pas ceux d’une suite

arithmétique car 10 − 8 = 2 et 14 − 10 = 4.11 Les quatre premiers termes sont ceux d’une suite

arithmétique de raison −5.12 La suite est décroissante.

13Chapitre 2 Suites numériques

48 1. un = –26 + 15n.2. u21 = 289.49 Exercice résolu.50 1. La suite (un ) est croissante car r 0.

2. un = 14 + 7n.3. n = 213.51 1. L’énoncé est vrai car pour tout n, un+1 = un + 5 en

particulier pour n = 2.2. « Si u3 = u2 + 5, alors la suite (un ) est arithmétique de raison 5 ». L’énoncé est faux , en voici un contre-exemple : la suite de terme général un = n2.52 Voir manuel page 250.53 Faux : un = u0 + nr = 7 + 4n.54 Vrai : vn = v1 + (n – 1)r = 9 + (n – 1) × (–2) = 11 – 2n.55 Vrai : u0 = 28 , un = 28 + 11n, le dixième terme est u9 = 127.56 Fichiers associés sur www.bordas-indice.fr et sur le

CD-Rom Professeur :02_TSTMG_exercice56.xlsx (Excel 2007),02_TSTMG_exercice56.xls (Excel 2003),02_TSTMG_exercice56.ods (OpenOffice).1. =B2-13 .2. =B3+C2 .57 1. un = 16 + 1,5n.

2. S = 875.58 1. un = 150 + 12n.

2. S = 19 866.59 1. un = 53 – 6n.

2. S = –7 680.60 1. S = 28.

2. a. À la somme u0 + u1 + u2 + u3.b. On remplace 3 par 10 dans l’instruction « I allant de 0 à 3 ».61 Vrai : la suite a pour terme général un = 1 + 6n, on obtient

la somme à la calculatrice :Casio : Σ(1+6*X,X,0,9)

Texas : somme(suite(1+6*X,X,0,9)) .62 Faux : la formule =SOMME(B1;B12) permet seulement

d’ajouter les valeurs contenues dans les deux cellules B1 et B12. Il faudrait saisir =SOMME(B1:B12) .63 Faux : il s’agit de la somme des 26 premiers termes.64 1. v2 = 39, v3 = 48. S3 = 117.

2. Pour tout entier naturel n, vn+1 = vn + 9, donc (vn ) est une suite arithmétique de raison 9.3. vn = 21 + 9n.4. 2 310 kilomètres.65 1. u1 = 115, u2 = 120 et u0 + u1 + u2 = 345.

2. Pour tout entier naturel n, un+1 = un + 5 donc la suite (un ) est arithmétique de raison 5.3. un =110 + 5n.4. 2 360 euros.66 Voir manuel page 250.67 1. 0,25 %.

2. a. 2 000 × 0,25 % = 5 donc un+1 = un + 5 pour tout entier naturel n ; la suite (un ) est donc arithmétique de raison 5.

Fichiers associés sur www.bordas-indice.fr et sur le CD-Rom Professeur :02_TSTMG_exercice23.xlsx (Excel 2007),02_TSTMG_exercice23.xls (Excel 2003),02_TSTMG_exercice23.ods (OpenOffice).24 1. On calcule la somme S = u3 + … + u14.

2. S = 1 104.25 1. u1 = 37 000 et u2 = 38 000.

2. un+1 = un + 1 000 ; la suite (un) est arithmétique de raison 1 000.26 v1 = 2, v2 = 4, v3 = 8. v4 = 16 donc v5 = 32.27 v1 = 8, v2 = 4, v3 = 2. v4 = 1 donc v5 = 0,5.28 Voir manuel page 250.29 w2 = 100, w3 = 10, w4 = 1. w5 = 0,1 donc w6 = 0,01.30 q = 32.31 q = 14.32 q = 1,02.33 q = 0,9.34 Les réels 25 - 20 - 16 - 12,8 sont les premiers termes d’une

suite géométrique de raison 0,8.35 1 ; 2 ; 4 et 8 sont les premiers termes d’une suite

géométrique de raison 2.36 La suite (vn ) est décroissante.37 La suite (vn ) est croissante.38 1. un = u0 × qn = 2 × 5n.

2. u6 = 31 250.39 vn = v0 × qn = 16 000 × 1

2( )n = 16 000 × 12 n = 16 000

2 n .

2. v10 = 1258

.

40 1. vn = v1 × qn–1 = 240 × 0,2n–1.2. v6 = 0,0768.41 1. vn = v1 × qn–1 = 7 × 5n–1.

2. v7 = 109 375.42 1. v1 = 20, v2 = 80 et v3 = 320.

2. S = v0 + v1 + v2 + v3 = 425.43 Fichiers associés sur www.bordas-indice.fr et sur le

CD-Rom Professeur : 02_TSTMG_exercice43.xlsx (Excel 2007),02_TSTMG_exercice43.xls (Excel 2003),02_TSTMG_exercice43.ods (OpenOffice).1. On calcule la somme des termes u0, u1, …, u7.2. =SOMME(B4:B9) .44 Voir manuel page 250.45 1. u1 = 172 500.

2. a. Le coefficient multiplicateur associé à une augmentation de 15 % est 1,15. un+1 = 1,15un.b. (un ) est géométrique de raison 1,15.

P O U R S ’ E N T R A Î N E R46 1. un = 7 – 5n.

2. u9 = –38.47 1. un = –6 + n.

2. u13 = 7.

14

89 Fichier associé sur www.bordas-indice.fr et sur le CD-Rom Professeur :02_TSTMG_exercice89.alg (AlgoBox).1. S prend la valeur S + U.U prend la valeur U × Q.2. Voir fichier AlgoBox.90 Vrai : un = 4 × 5n et u0 + … + u7 = 390 624 obtenue avec

la calculatrice :Casio : Σ(4*5^X,X,0,7) .Texas : somme(suite(4*5^X,X,0,7)) .91 Faux : La cellule B14 contient le terme v11 et la cellule B28

contient le terme v25.92 Voir manuel page 251.93 1. v1 = 16 800 ; v2 = 17 640 et v3 = 18 522.

2. vn+1 = 1,05vn pour tout entier naturel n donc la suite (vn ) est géométrique de raison 1,05.3. vn =16000 × 1,05n.4. v13 ≈ 30 170 arrondi à l’unité.94 1. T1 = 541,554.

2. Tn+1 = 0,786Tn .3. La suite (Tn ) est géométrique de raison 0,786.4. Tn = 689 × 0,786n.5. T5 ≈ 206,7.95 Exercice résolu.96 1. c1 = 153 et c2 = 156,06.

2. cn+1 = 1,02cn. La suite (cn ) est géométrique de raison 1,02.3. cn = 150 × 1,02n.4. c9 ≈ 179,26.5. c0 + c1 + … + c9 ≈ 1 642,46.97 Faux : cn+1 = 1,04cn donc (cn ) est géométrique de raison

1,04.98 Vrai : 1,0418 ≈ 2,03.99 Voir manuel page 251.100 1. un = 70 + 15n et vn = 80 × 1,1n.2. – Pour 0 � n � 1, un � vn .– Pour 1 � n � 12, un � vn .– Pour 13 � n � 20, un � vn .3. Pour les grandes valeurs de n, vn est supérieur à un .101 1. un = 200 + 50n et vn = 40 × 1,15n.2. – Pour 0 � n � 25 , un � vn .– Pour 26 � n � 30, un � vn .3. vn est supérieur à un pour les grandes valeurs de n.102 Vrai : pour 0 � n � 5, on a un � vn et u6 � v6.103 Faux : v14 ≈ 13 890 donc v14 � 105.104 Faux : comme 1,1 � 1,4 pour les grandes valeurs de n, wn � vn .105 1. un = –12 + 7n.2. u17 = 1073. La somme est égale à 14 812.106 1. un + 1 = 1,03 un donc (un) est géométrique de raison 1,03.2. un = 600 × 1,03n.3. Pour 2020 : u8 ≈ 760.107 1. un = 1 200 × 1,04n et vn = 800 × 1,06n.2. n = 22.

b. un = 2 000 + 5n et u7 = 2 035.68 Exercice résolu.69 Fichiers associés sur www.bordas-indice.fr et sur le

CD-Rom Professeur :02_TSTMG_exercice69.xlsx (Excel 2007),02_TSTMG_exercice69.xls (Excel 2003),02_TSTMG_exercice69.ods (OpenOffice).1. a1 = 4 700 ; a2 = 4 400.2. Pour tout entier naturel n, an+1 = an – 300.3. an = 5 000 – 300n.4. a10 = 2 000.

5. a. =SOMME(C2:C12) .

b. 38 500 tonnes70 Faux : (un ) est arithmétique de raison 3.71 Faux : un = 124 + 3n donc u6 = 146.72 Faux : u0 + … + u8 = 1 224.73 1. un = 5 × 4n.

2. u11 = 20 971 520.74 1. un = 5 × 0,7n.

2. u8 ≈ 0,288 à 10–3 près.75 1. vn = 2 500 × 1,1n–1.

2. v21 ≈ 16 818,75 à 0,01 près.76 Voir manuel page 250.77 1. q = 5

12.

2. vn = 12 × 512( )n–1

.

3. v17 = 12 × 512( )16

≈ 10–5.

78 Exercice résolu.79 1. (un ) est croissante car u0 � 0 et 1,2 � 1.

2. un = 450 × 1,2n.3. n = 18.80 1. (un ) est décroissante car u0 � 0 et 0 � 0,7 � 1.

2. un = 5 000 × 0,7n.3. n = 31.81 Faux : vn = 125 × 3n.82 Faux : vn = 40 × 5n–1.83 Vrai : le sixième terme est 400 × 0,95 = 2 361,96.84 Voir manuel page 250.85 1. un = 400 × 0,75n.

2. S ≈ 1599,1.86 1. un = 2 500 × 0,4n–1.

2. S ≈ 10 416,67.87 1. un = 50 × 1,3n.

2. S ≈ 39 817,87.88 Fichiers associés sur www.bordas-indice.fr et sur le

CD-Rom Professeur :02_TSTMG_exercice88.xlsx (Excel 2007),02_TSTMG_exercice88.xls (Excel 2003),02_TSTMG_exercice88.ods (OpenOffice).1. un = 200 × 0,9n.

2. =SOMME(B2:B54) .

3. S ≈ 1 991,65

15Chapitre 2 Suites numériques

P O U R A P P R O F O N D I R112 1. x0 = 0,6 et y0 = 0,4.x1 = 0,8x0 + 0,1y0 = 0,8 × 0,6 + 0,1 × 0,4 = 0,48 + 0,04 = 0,52.y1 = 0,9y0 + 0,2x0 = 0,9 × 0,4 + 0,2 × 0,6 = 0,36 + 0,12 = 0,48.2. a. xn+1 = 0,8xn + 0,1yn.b. yn+1 = 0,2xn + 0,9yn.c. yn = 1 – xn donc xn+1 = 0,8xn + 0,1(1 – xn ) soit xn+1 = 0,7xn + 0,1.

3. a. un+1 = xn+1 – 13

= 0,7xn + 0,1 – 13

= 0,7xn – 730

.

b. un+1 = 0,7 xn – 1030( ) = 0,7un. La suite (un ) est géométrique de

raison 0,7 et de premier terme u0 = 415

.

c. un = 415

× 0,7n

d. xn = un + 13= 4

15× 0,7n + 1

3.

4. À partir de 2021.113 Fichiers associés sur www.bordas-indice.fr et sur le CD-Rom Professeur :02_TSTMG_exercice113.xlsx (Excel 2007),02_TSTMG_exercice113.xls (Excel 2003),et 02_TSTMG_exercice113.ods (OpenOffice).1. u1 = 24 480.2. a. un+1 = 1,02un.b. La suite (un ) est géométrique de raison 1,02.c. un = 24 000 × 1,02n.3. Salaire en 2018 : u5 ≈ 26 497,94 à 0,01 près.4. a. v1 = 1,025 × 4 000 + 4 000 = 8 100.b. La somme disponible l’année 2013+n est obtenue en augmentant de 2,5 % la somme disponible l’année précédente et en ajoutant 4 000 : vn+1 = 1,025vn + 4 000.c. =1,025*B2+4000 .5. En 2019.114 1. 1,0210C0 = 9 000 donc C0 ≈ 7 383,13.2. a. Valeur actuelle de la première dette : 2 803,74.b Valeur actuelle de la seconde dette : 3 493,75.c. Valeur actuelle de la dette de montant X : 6 297,49 donc X = 7 714,70.115 1. a. u1 = 320.b. un+1 = un + 10. La suite (un) est arithmétique de raison 10.c. un = 310 + 10n ; u7 = 380.2. a. v1 = 820.b. vn+1 = 1,025vn . La suite (vn) est géométrique de raison 1,025.c. vn = 800 × 1,025n ; v7 ≈ 950,95.3. 39,96 %.4. a.

b. n = 8 soit en 2021.c. n = 12 soit en 2025.

P O U R FA I R E L E P O I N TVoir manuel page 251.Les corrigés détaillés sont disponibles sur le site www.bordas-indice.fr.

ACCO M PAG N E M E N T P E R S O N N A L I S É108 un = 7 – 28n ; u45 = –1 253.109 un = –25 + 5n ; u123 = 590.110 vn =15 × 1,2n–1 ; v14 ≈ 160,5 à 0,1 près.111 vn = 5 000 × 0,9n ; v21 ≈ 547,1 à 0,1 près.

T R AVAU X P R AT I Q U E STP Évolutions du trafic d’axes routiers

L’objectif de ce TP est de construire une feuille de calcul pour

comparer l’évolution de deux suites, l’une arithmétique et l’autre

géométrique. La fonction SOMME du tableur est utilisée pour

calculer la somme de termes consécutifs des suites étudiées.

Fichiers associés sur le site www.bordas-indice.fr et sur le CD-Rom Professeur :02_TSTMG_TP.xlsx (Excel 2007) , 02_TSTMG_TP.xls (Excel 2003),02_TSTMG_TP.ods (OpenOffice),02_TSTMG_ TP_correction.xlsx (Excel 2007),02_TSTMG_TP_correction.xls (Excel 2003),02_TSTMG_ TP_correction.ods (OpenOffice).A. Préparation d’une feuille de calcul2. On saisit dans A3 la formule =A2+1 .

3. On saisit dans B3 la formule =B2+1 .

B. Évolutions du trafic de l’axe Sud1. v1 = 13 390 et v2 ≈ 13 792 à l’unité près.

2. 1,03.

3. vn+1 = 1,03vn . La suite (vn ) est géométrique de raison 1,03.

4. On saisit la formule =C2*1,03 .

5. On recopie la formule précédente vers le bas.

C. Évolutions du trafic de l’axe Nord1. un+1 = un + 500. La suite (un ) est arithmétique de raison 500.

2. On saisit la formule =D2+500 .

3. On recopie la formule précédente vers le bas.

4. n = 18 ce qui correspond à l’année 2013.

5. un � vn pour 18 � n � 30.

D. Fréquentation totale1. b. La valeur affichée correspond au nombre de véhicules qui

circulent entre 2013 et 2027 sur l’axe Sud.

2. =365*SOMME(D2:D16) .

3. – Pour la période 2013-2027, l’axe Nord est le plus fréquenté.

– Pour la période 2013-2043, il s’agit de l’axe Sud.

16

c. Casio : Σ(500*1,02^X,X,0,8) .

Texas : somme(suite(500*1,02^X,X,0,8)) .3. a. 585,83.b. 574,34.c. 4 877,31.120 1. S1 = 1 ; S2 = 3 ; S3 = 6 ; S4 = 10.2. S13 = 91.3. Sn+1 = Sn + n + 1.

4. a. V1 = 12

; V2 = 1 ; V3 = 32

et V4 = 2.

b. On conjecture que la suite (Vn ) est arithmétique de raison 12

.

5. a. Vn+1 = Sn+1 – 12

(n + 1)2 = Sn + n +1 – 12

(n + 1)2.

b. Vn+1 = Sn + n + 1 – 12

(n2 + 2n + 1) = Sn – 12

n2 + 12

donc

Vn+1 = Vn + 12

. La suite (Vn ) est arithmétique de raison 12.

6. Vn = 12

n.

7. Sn = Vn + 12

n2 = 12

n + 12

n2 = n + n2

2= n(n +1)

2.

8. 1 + 2 + … + 10 = 55.

C A P V E R S L E B ACZoom sur le tableur

Fichiers associés sur www.bordas-indice.fr et sur le CD-Rom Professeur :02_TSTMG_zoomtableur.xlsx (Excel 2007),02_TSTMG_zoomtableur.xls (Excel 2003),02_TSTMG_zoomtableur.ods (OpenOffice).

Sujet AFichiers associés sur www.bordas-indice.fr et sur le CD-Rom Professeur :02_TSTMG_sujetA.xlsx (Excel 2007),02_TSTMG_sujetA.xls (Excel 2003),02_TSTMG_sujetA.ods (OpenOffice).1. un+1 = 1,1un donc la suite (un ) est géométrique de raison 1,1.2. un = 5,6 × 1,1n ; u10 ≈ 14,52.3. =D2*0,96 .4. En 2027 et u17 ≈ 28,31.

Sujet BFichiers associés sur www.bordas-indice.fr et sur le CD-Rom Professeur :02_TSTMG_sujetB.xlsx (Excel 2007),02_TSTMG_sujetB.xls (Excel 2003),02_TSTMG_sujetB.ods (OpenOffice).1. Réponse c.2. Réponse b.3. Réponse b.4. Réponse a.5. Réponse a.

116 1. t1 = 0,5 %.2. t2 ≈ 0,00487 ≈ 0,497 %.3. a. Placement A.b. Les deux placements ont la même rentabilité.c. Placement B.117 Partie A

1. 30 0001,04 ≈ 28 846,15.

2. 30 0001,042

≈ 27 736,69.

3. Une valeur placée pendant n années à un taux de 4 % est multipliée par 1,04n donc Vn × 1,04n = 30 000 soit

Vn = 30 0001,04n .

4. E = V1 + … + V10. On obtient E ≈ 243 326,87.

Partie B1. Vn × 1,04n = a donc Vn = a

1,04n.

2. S ≈ 8,1109.3. 350 000 est la somme des valeurs actuelles Vn donc350 000 = a

1,04+ a1,042

+…+ a1,04n

= aS.

4. a ≈ 43 151,81.5. Montant des intérêts : 81 518,10 euros, ce qui représente 18,89 % des sommes versées.118 A . Étude d’une suite1. u1 = 740 et u2 = 644.2. a. vn+1 = un+1 – 500 = 0,6un – 300.b. vn+1 = 0,6(vn + 500) – 300 = 0,6vn + 300 – 300 =0,6vn.c. La suite (vn ) est géométrique de raison 0,6 et de premier terme v0 = 400.d. La suite (vn ) est décroissante et la suite (un ) est croissante.e. vn = 400 × 0,6n donc un = vn + 500 = 400 × 0,6n + 500.f. n = 12 ; u11 ≈ 501,45 ; u12 ≈ 500,87.B. Application1. a0 = 900 et a1 = 0,8 × 900 + 0,2 × 100 = 740.2. a2 = 0,8 × 740 + 0,2 × (1 000 – 740) = 644.3. an est le nombre de clients de A l’année 2013+n ; 1 000 – an

est le nombre de clients de B l’année 2013+n. A perd 20 % de ses clients chaque année donc l’année 2013+n+1, il lui reste 0,8an de ses clients de l’année précédente.20 % des clients de B viennent chez A, donc l’année 2013+n+1, A compte 0,2(1 000 – an) nouveaux clients :

an+1 = 0,8an + 0,2(1 000 – an).4. an+1 = 0,8an + 200 – 0,2an = 0,6an + 200.5. La suite (an ) est égale à la suite (un ). On peut conjecturer que le marché va s’équilibrer vers une répartition égalitaire entre A et B.119 1. a. qS = q + q2 + … + q9 ;S – qS = (1 + q + … + q8)–( q + q2 + … + q9) = 1 – q9.

b. S – qS = (1 – q)S donc S = 1– q9

1– q.

c. Pour q = 1,02 , S ≈ 9,75462 à 10–5 près.2. a. Pour tout entier n, vn = v0 × 1,02n.T = v0 + v0 × 1,02 + v0 × 1,022 + … + v0 × 1,028 doncT = v0 × (1 + 1,02 + 1,02² + … + 1,028).b. T ≈ 4 877,31.

17Chapitre 2 Suites numériques

5. – Pour 0 � n � 4 , le placement le plus intéressant est le placement B.– Pour n � 5, le placement le plus intéressant est le place-ment A.

Sujet FFichiers associés sur www.bordas-indice.fr et sur le CD-Rom Professeur :02_TSTMG_sujetF.xlsx (Excel 2007),02_TSTMG_sujetF.xls (Excel 2003),02_TSTMG_sujetF.ods (OpenOffice).1. Réponse a.2. Réponse a3. Réponse b4. Réponse b5. Réponse b

Sujet GFichiers associés sur www.bordas-indice.fr et sur le CD-Rom Professeur :02_TSTMG_sujetG.xlsx (Excel 2007),02_TSTMG_sujetG.xls (Excel 2003),02_TSTMG_sujetG.ods (OpenOffice).

A. Étude des dépenses du service A1. a. La suite (an) est arithmétique de raison 4 000.b. an = 16 000 + 4 000n.c. a10 = 56 000.

2. =R2+Q3 .

3. 380 000 : cela représente la dépense totale au cours des dix prochaines années pour l’entreprise A.B. Étude des dépenses du service B1. a. =S2*1,15 .b. La suite (bn ) est géométrique de raison 1,15.c. bn = 20 000 × 1,15n–1.2. b10 ≈ 70 400.C. Comparaison des deux servicesb1 + b2 +… + b10 = 406 074. Il s’agit de l’entreprise B.

121 1. un = 17,8 × 1,07n.2. u12 = 40,1 donc 40 100 entreprises.3. 358 500 à la centaine près.122 1. (an) est arithmétique de raison 160.2. an = 3 460+160n.3. a11 = 5 220, donc le nombre de logements n’aura pas doublé.

Sujet CFichiers associés sur www.bordas-indice.fr et sur le CD-Rom Professeur :02_TSTMG_sujetC.xlsx (Excel 2007),02_TSTMG_sujetC.xls (Excel 2003),02_TSTMG_sujetC.ods (OpenOffice).1. (un ) est géométrique de raison 1,05.2. un = 4 × 1,05n.3. u6 ≈ 5,36.4. a. =C2*1,05 .

b. =SOMME($C$2 :C3) et =D2+C3 .5. a. 32,57 Mo.b. Non car 32,57 �30.

Sujet D1. a. (An ) est arithmétique de raison 20 et de terme initial 150.b. (Bn) est géométrique de raison 1 et de terme initial 130.2. An = 130 + 20n.Bn = 130 × 1,2n–1.3. a. A6 = 250 et B6 ≈ 323.b. Formule A : 1 200 euros ;Formule B : 1 291 euros.c. Formule B.4. a. S contient la valeur 320.T contient la valeur 286.Résultat affiché : « Formule A ».b. L’algorithme permet d’afficher la formule pour laquelle Florence a le plus économisé au bout de N mois.

Sujet EFichiers associés sur www.bordas-indice.fr et sur le CD-Rom Professeur :02_TSTMG_sujetE.xlsx (Excel 2007),02_TSTMG_sujetE.xls (Excel 2003),02_TSTMG_sujetE.ods (OpenOffice).1. U1 = 2 562,5 et V1 = 2 565.2. a. La suite (Un ) est géométrique de raison 1,025 car Un+1 = 1,025Un pour tout entier naturel n.b. Un = 2 500 × 1,025n.3. a. Pour tout entier naturel n, Vn+1 = Vn + 65 donc la suite (Vn) est arithmétique de raison 65.b. Vn = 2 500 + 65n.

4. a. =C2*1,025 .

b. =D2+65 .

18

Ce chapitre est un prolongement de l’étude des fonctions polynômes du second ou du troisième degré faite en classe de Première.La première séquence du cours, consacrée aux notions nouvelles du programme : dérivation des fonctions x x n(n entier naturel non nul) et des fonctions ku (k réel) et u + v, est introduite par les activités 1 et 2. Dans ces deux activités, les élèves sont amenés à utiliser les expressions des dérivées étudiées en Première. Par la suite, afin d’éviter les confusions, nous n’utilisons plus ces formules.La seconde séquence du cours est consacrée à l’étude d’une fonction polynôme. Il s’agit uniquement de rappeler et généraliser les propriétés étudiées en classe de Première. Les Savoir-faire permettent aux élèves de se réapproprier les méthodes pour étudier les variations d’une fonction polynôme, déterminer des extremums, déterminer une équation de la tangente en un point d’une courbe et tracer cette tangente.Afin que les élèves ne rencontrent pas de difficultés lors de l’étude du signe de la dérivée (celle-ci pouvant être de degré supérieur à trois), nous avons rappelé comment identifier f ’ (x) à une expression factorisée donnée dans un énoncé (Savoir-faire 5), comment étudier le signe de ax + b ou de ax 2 + bx + c (Savoir-faire 6) et comment factoriser par x (ou x 2), lorsque c’est possible (exemples du cours). Dans les exercices, nous avons veillé à ne pas systématiser l’utilisation du discriminant lorsque la fonction dérivée est du second degré.Les exercices sont très progressifs, allant du calcul de la dérivée de fonctions x x n, à l’étude de fonctions polynômes permettant de modéliser l’évolution d’un chiffre d’affaires, du cours d’une action, du nombre d’abonnés à une revue, etc. Nous avons proposé quelques exercices d’algorithmique et de nombreux exercices faisant intervenir un tableur.Les exercices Cap vers le Bac sont le plus souvent composés de deux parties, l’une faisant appel aux capacités liées au tableur ou aux interprétations graphiques, l’autre aux capacités liées au calcul.

Les notions abordées dans le chapitre 3 • Fonction dérivée : somme et produit par un réel• Variations d’une fonction polynôme• Tangente à une courbe

B Notre point de vue

Contenus Capacités attendues Commentaires

DérivationFonction dérivée de x xn. • Connaître la fonction dérivée de x xn. L'étude des ensembles de définition et de dériva-

tion n'est pas un objectif du programme.

Fonction dérivée d’une somme, d’un produit par une constante.Application à l'étude des variations des fonctions.

Dans le cadre d’une résolution de problème :• déterminer la fonction dérivée d’une fonc-tion polynôme ;• étudier les variations et les extremums d’une fonction à partir du signe de sa fonc-tion dérivée ;• déterminer une équation de la tangente en un point d’une courbe représentative ; tracer cette tangente.

On se limite à des fonctions simples.

Cette partie du programme se prête particuliè-rement à l’étude de situations issues des autres disciplines (résolutions graphiques ou numé-riques d’équations et d’inéquations, problèmes d'optimisation…).

Le programme

3CHAPITRE

Fonctions polynômes

A

19Chapitre 3 Fonctions polynômes

Voir manuel page 251 et le site www.bordas-indice.fr pour les corrigés détaillés.

C Avant de commencer

D ActivitésActivité Et si on dépassait

le troisième degré…1

Cette activité a pour objectif d’introduire la dérivée de la fonction x x n, n étant un entier naturel non nul.Après le calcul de la dérivée des fonctions x x2 et x x3 à l’aide des formules étudiées en classe de Première, les élèves sont amenés à faire une conjecture sur la dérivée de x x n, à partir des résultats donnés par un logiciel de calcul formel.Fichier associé sur www.bordas-indice.fr et sur le CD-Rom Professeur :03_TSTMG_activite1.xws (Xcas).1. a. f ’(x) = 2ax + b.Pour f (x) = x 2 : a = 1, b = c = 0 donc f ’(x) = 2x.b. f ’(x) = 3ax 2 + 2bx + c.Pour f (x) = x 3 : a = 1, b = c = d = 0 donc f ’(x) = 3x 2.2. a.

f (x) n f ’(x)x 2 2 2xx 3 3 3x 2

x 4 4 4x 3

x 5 5 5x 4

x 7 7 7x 6

x 100 100 100x 99

b. ConjecturesPour f (x) = x 6 : f ’(x) = 6x 5.Pour f (x) = x 9 : f ’(x) = 9x 8.c. Conjecture : f ’(x) = nx n – 1.d. Pour n = 2 et pour n = 3, on obtient bien les mêmes expressions qu’avec les formules de la question 1.

Activité Des opérations avec les fonctions2Cette activité a pour objectif d’amener les élèves à faire une conjecture sur la dérivée des fonctions ku et u + v.1. a. u’(x) = 2x + 3.b. g(x) = 2x 2 + 6x – 2 et g’(x) = 4x + 6.

x – 2 – 1 0 1 2 3u’ (x) – 1 1 3 5 7 9g’ (x) – 2 2 6 10 14 18

Conjecture : g’(x) = 2u’(x).c. h(x) = 5x 2 + 15x – 5 et h’(x) = 10x + 15.

x – 2 – 1 0 1 2 3u’ (x) – 1 1 3 5 7 9h’ (x) – 5 5 15 25 35 45

Conjecture : h’(x) = 5u’(x).

d. Conjecture : f ’(x) = ku’(x).2. a. p(x) = x 2 + 3x – 1 + 2x 2 – 4x + 5 = 3x 2 – x + 4.p’(x) = 6x – 1.b. v’(x) = 4x – 4.u’(x) + v’(x) = 2x + 3 + 4x – 4 = 6x – 1.c. Conjecture : p’(x) = u’(x) + v ’(x).

Activité Du signe de la dérivée aux variations de la fonction

3

Cette activité permet de rappeler et d’étendre aux fonctions polynômes de degré strictement supérieur à 3 les propriétés étudiées en classe de Première sur le sens de variation d’une fonction polynôme.L’utilisation d’un logiciel de géométrie dynamique permet de bien « visualiser » le signe de f ‘ (les parties de la courbe f ' sont colorées différemment selon le signe de f ’(x)) et le sens de variation de f (les parties de la courbe f sont colorées différemment selon le sens de variation de f ).Fichier associé sur www.bordas-indice.fr et sur le CD-Rom Professeur :03_TSTMG_activite3.ggb (GeoGebra).A. Sur le cahier1. Si pour tout réel x appartenant à un intervalle I,f ’(x) 0, alors f est croissante sur I.Si pour tout réel x appartenant à un intervalle I,f ’(x) 0, alors f est décroissante sur I.2. a. Sur les intervalles où la courbe f est colorée en rouge, f est décroissante.b. Sur les intervalles où la courbe f ’ est colorée en vert, f ’(x) est négatif.c. Sur]– ; x1] et sur [x2 ; x3], f ’(x) 0 et f est décroissante.d.

x – x1 x2 x3 +

f ’(x) – 0 + 0 – 0 +

f

B. Avec l’ordinateur1. Ouvrir le fichier 03_TSTMG_activite3.ggb (GeoGebra).Sélectionner le mode Déplacer, cliquer gauche sur le curseur pour le déplacer.Déplacer le curseur c de – 5 à – 1,6.On remarque que comme dans la partie A :sur ]– ; x1] et sur [x2 ; x3], f ’(x) 0 et f est décroissante ;sur [x1 ; x2] et sur [x3 ; + [, f ’(x) 0 et f est croissante.

20

Fichier associé sur www.bordas-indice.fr et sur le CD-Rom Professeur :03_TSTMG_activite4.ggb (GeoGebra).A. Sur le cahier1. f ’(x) = 3x 2 – 10x + 1.2. a . f ’(– 1) = 14.b. Par lecture, une équation de T est : y = 14x + 8.Le coefficient directeur de T est égal à 14.B. Avec l’ordinateur1. Ouvrir le fichier 03_TSTMG_activite4.ggb (GeoGebra).Sélectionner le mode Déplacer, cliquer gauche sur le curseur pour le déplacer et le positionner sur les différentes valeurs mises dans le tableau.

xA – 2 – 1 0 1 2 3 4 5f ’(xA) 33 14 1 – 6 – 7 – 2 9 26

Coefficientdirecteurde T

33 14 1 – 6 – 7 – 2 9 26

2. Le coefficient directeur de T semble être égal à f ’(xA).

2. Déplacer le curseur c de –1,5 à 5.

x – x1 +

f ’(x) – 0 +

f

3. Les propriétés rappelées dans la question A. 1 semblent encore vraies.

Activité Quand on approche la courbe en un point

4

Cette activité a pour objectif de rappeler comment calculer un nombre dérivé, comment lire le coefficient directeur d’une droite dont on connaît l’équation réduite et enfin de faire une conjecture sur l’expression du coefficient directeur de la tangente à la courbe en un point, en fonction du nombre dérivé en ce point.

E Exercicesu’ (x) = 1, v’ (x) = 0 et f ’(x) = 1.2. u(x) = x 2 et v (x) = x.u’ (x) = 2x, v’ (x) = 1 et f ’(x) = 2x +1.3. u(x) = x 3 et v (x) = x 2.u’ (x) = 3x 2, v’ (x) = 2x et f ’(x) = 3x 2 + 2x.10 1. u(x) = 5 et v (x) = 3x2.

u’ (x) = 0, v’ (x) = 6x et f ’(x) = 6x.2. u(x) = 2x 2 et v (x) = – x.u’ (x) = 4x, v’ (x) = – 1 et f ’(x) = 4x –1.3. u(x) = – x 2 et v (x) = 3x 3.u’ (x) = – 2x, v’ (x) = 9x 2 et f ’(x) = – 2x + 9x 2.11 f ’(x) = 3.12 f ’(x) = 4x – 5.13 Voir manuel page 251.14 Voir manuel page 251.15 f ’(x) = 5x 4 + 3x 2 + 1.16 f ’(x) = 6x 5 – 10x 4.17 Voir manuel page 251.18 1. f ’(x) = 12x 3 – 60x 2.

2. Pour tout réel x, 12x 2(x – 5) = 12x 3 – 60x 2

donc on a bien f ’(x) = 12x 2(x – 5).19 Sur [– 20 ; –2] et sur [1 ; 20], f ’(x) 0 donc f est décroissante

sur chacun de ces intervalles.Sur [– 2 ; 1], f ’(x) 0 donc f est croissante sur [– 2 ; 1].20 Sur [– 20 ; – 12] et sur [3 ; 10], f ’(x) 0 donc f est

décroissante sur chacun de ces intervalles.Sur [– 12 ; 3] et sur [10 ; 20], f ’(x) 0 donc f est croissante sur chacun de ces intervalles.21 f est croissante sur R.22 f est décroissante sur R.23 f ’(x) = 4x – 8.

P O U R D É M A R R E R1 1. n = 2 et f ’(x) = 2x.

2. n = 7 et f ’(x) = 7x 6.3. n = 11 et f ’(x) = 11x 10.2 1. n = 3 et f ’(x) = 3x 2.

2. n = 14 et f ’(x) = 14x 13.3. n = 20 et f ’(x) = 20x 19.3 1. u’ (x) = 1 et f ’(x) = 3.

2. u’ (x) = 2x et f ’(x) = 6x.3. u’ (x) = 3x 2 et f ’(x) = 9x 2.4. u’ (x) = 4x 3 et f ’(x) = 12x3.4 1. u’ (x) = 1 et f ’(x) = – 5.

2. u’ (x) = 2x et f ’(x) = – 10x.3. u’ (x) = 3x 2 et f ’(x) = – 15x 2.4. u’ (x) = 4x 3 et f ’(x) = – 20x3.5 1. u(x) = x.

u’ (x) = 1, f ’(x) = 2, g’ (x) = 17 et h’ (x) = – 0,01.2. u(x) = x 2.u’ (x) = 2x, f ’(x) = 10x, g’ (x) = 14x et h’ (x) = – 200x.3. u(x) = x 3.u’ (x) = 3x 2 , f ’(x) = 9x 2, g’ (x) = – 3x 2 et h’ (x) = 30x 2.6 1. f ’(x) = 0 et g’ (x) = 0.

2. f ’(x) = –50 et g’ (x) = 11.3. f ’(x) = 18x et g’ (x) = – 62x.7 Voir manuel page 251.8 1. u’ (x) = 0, v’ (x) = 3 et f ’(x) = 3.

2. u’ (x) = 10x, v’ (x) = 0 et f ’(x) = 10x.3. u’ (x) = 6x, v’ (x) = – 6x² et f ’(x) = 6x – 6x2.4. u’ (x) = 40x 4 , v’ (x) = 30 et f ’(x) = 40x 4 + 30.9 1. u(x) = x et v(x) = 2.

21Chapitre 3 Fonctions polynômes

2. T a pour équation y = 4x – 7.36 1. Graphiquement, l’équation a pour solutions – 2 et 1.

2. a. (x + 2)2 = x 2 + 4x + 4.b. Pour tout réel x,(x – 1)(x + 2)2 = (x – 1)(x 2 + 4x + 4) = x 3 + 3x 2 – 4.c. x 3 + 3x 2 – 4 = 0 équivaut à (x – 1)(x + 2)2 = 0, donc à x = 1 ou x = – 2.37

1. Graphiquement :a. les solutions sont environ 1 et –11 ;b. l’ensemble des solutions est ]– ; – 11] [1 ; + [.2. a. (x – 1)(x + 11) = x 2 + 10x –11.b. 3(x – 1)(x + 11) = 3(x 2 + 10x – 11) = 3x 2 + 30x – 33.c. • 3x 2 + 30x – 33 = 0 équivaut à 3(x – 1)(x + 11) = 0, donc à x = 1 ou x = – 11. • Tableau de signes de 3x 2 + 30x – 33 :

x – – 11 1 +

3x 2 + 30x – 33 + 0 – 0 +

S = ]– ; – 11] [1 ; + [.

P O U R S ’ E N T R A Î N E R38 1. f ’(x) = 17x 16.

2. f ’(x) = 23x 22.3. f ’(x) = 105x 104.39 • f (x) = – 0,1u(x) avec u(x) = x.

u’ (x) = 1 et f ’(x) = – 0,1.• g(x) = 1

3u(x) avec u(x) = x 2.

u’ (x) = 2x et g’(x) = 23 x.

• h(x) = 509

u(x) avec u(x) = x 3.

u’ (x) = 3x 2 et h’(x) = 50x2

3.

40 • f (x) = –350u(x) avec u(x) = x 2.u’ (x) = 2x et f ’(x) = – 700x.

• g(x) = 1003

u(x) avec u(x) = x 3.

u’ (x) = 3x 2 et g’(x) = 100x 2.• h(x) = 0,001u(x) avec u(x) = x 4.u’ (x) = 4x 3 et h’(x) = 0,004x 3.

41 • f (x) = 35

u(x) avec u(x) = x 4.

u’ (x) = 4x 3 donc f ’(x) = 125

x 3.

• g(x) = 110

u(x) avec u(x) = x 5.

u’ (x) = 5x 4 donc g ’(x) = 12

x 4.

• h(x) = 50u(x) avec u(x) = x 6.u’ (x) = 6x 5 donc h’(x) = 300x 5.

Sur ]– ; 2], f ’(x) 0 donc f est décroissante sur ]– ; 2].Sur [2 ; + [, f ’(x) 0 donc f est croissante sur [2 ; + [.24 f ’(x) = – 2x + 1.

Sur –` ; 12

⎤⎦

⎤⎦ , f ’(x) 0 donc f est croissante sur –` ; 1

2⎤⎦

⎤⎦ .

Sur 12

; + `⎡⎣

⎡⎣ , f ’(x) 0 donc f est décroissante sur 1

2; + `⎡

⎣⎡⎣ .

25 1. f ’(x) = 3x 2.Pour tout réel x, f ’(x) 0 donc f est croissante sur R.2. f ’(x) = 6x 2 + 3.Pour tout réel x, f ’(x) > 0 donc f est croissante sur R.3. f ’(x) = 5x 4.Pour tout réel x, f ’(x) 0 donc f est croissante sur R.26 1. f ’(x) = 3x 2 – 8x + 5.

2. a. Pour tout réel x,(3x – 5)(x – 1) = 3x 2 – 5x – 3x + 5 = 3x 2 – 8x + 5donc on a bien f ’(x) = (3x – 5)(x – 1).

b. Sur ]– ; 1] 53

; + `⎡⎣

⎡⎣ , f ’(x) 0 et sur 1 ; 5

3⎡⎣

⎤⎦ , f ’(x) 0.

3. f est croissante sur ]– ; 1] et sur 53

; + `⎡⎣

⎡⎣ , f est décroissante

sur 1 ; 53

⎡⎣

⎤⎦ .

27 1. f ’(x) = 3x 2 + 2x + 1. 2. a. D = – 8.b. Pour tout réel x, f ’(x) 0.3. Pour tout réel x, f ’(x) 0 donc f est croissante sur R.28 1.

x –10 0 20 40

f3 500

– 500

3 500

– 16 500

2. Sur l’intervalle [–10 ; 40], f admet un maximum égal à 3 500 pour x = – 10 et pour x = 20.f admet un minimum égal à – 16 500 pour x = 40.29 Voir manuel page 251.30 1., 2. et 3.

0

y

x1

1F

E

31 Une équation de T est : y = 2x – 8.32 Une équation de la tangente à la courbe au point

d’abscisse 2 est : y = 3x – 11.33 T1 a pour équation : y = 24x + 81.

T2 a pour équation : y = – 7.T3 a pour équation : y = 24x – 44.34 f ’(x) = 6x + 1.

Le coefficient directeur de la tangente est :a. f ’(3) = 19.b. f ’(5) = 31.c. f ’(–2) = – 11.35 1. T a pour équation y = – 3x + 7.

22

Sur ]– ; 2], f ’(x) 0 donc f est décroissante sur ]– ; 2].Sur [2 ; + [, f ’(x) 0 donc f est croissante sur [2 ; + [.

64 f ’(x) = – 15x 2 + 8x – 1. f ’(x) a pour racines 13 et 1

5.

Sur –` ; 15

⎤⎦

⎤⎦ et sur 1

3; + `⎡

⎣⎡⎣, f ’(x) 0 donc f est décroissante

sur chacun de ces intervalles.

Sur 15

; 13

⎡⎣

⎤⎦ , f ’(x) 0 donc f est croissante sur 1

5; 1

3⎡⎣

⎤⎦ .

65 f ’(x) = 3x 2 – 2x – 5. f ’(x) a pour racines –1 et 53

.

Sur ]– ; –1] et sur 53

; + `⎡⎣

⎡⎣, f ’(x) 0 donc f est croissante sur

chacun de ces intervalles.

Sur –1 ; 53

⎡⎣

⎤⎦ , f ’(x) 0 donc f est décroissante sur –1 ; 5

3⎡⎣

⎤⎦ .

66 f ’(x) = – 12x 2 + 12x – 3 = – 3(2x – 1)2.

Pour tout réel x, f ’(x) 0 donc f est décroissante sur R.67 f ’(x) = 3x 2 + 2x + 4. D = – 44.

Pour tout réel x, f ’(x) 0 donc f est croissante sur R.68 f ’(x) = – 6x 2 + 12x – 7. D = – 24.

Pour tout réel x, f ’(x) 0 donc f est décroissante sur R.69 Exercice résolu.70 1. f ’(x) = x 3 – 3x 2 – 10x.

En factorisant par x, on obtient f ’(x) = x(x 2 – 3x – 10).2. Le polynôme x 2 – 3x – 10 a pour racines – 2 et 5.3.

x –5 –2 0 5 7

x – – 0 + +

x 2 – 3x – 10 + 0 – – 0 +

f ’(x) – 0 + 0 – 0 +

4.

x – 5 – 2 0 5 7

f ’(x) – 0 + 0 – 0 +

f156,25

– 8

0

–93,75

12,25

71 1. f ’(x) = 4x 3 – 6x 2 = 2x 2(2x – 3).

Sur –` ; 32

⎤⎦

⎤⎦ , f ’(x) 0 donc f est décroissante sur –` ; 3

2⎤⎦

⎤⎦ .

Sur 32

; + `⎡⎣

⎡⎣, f ’(x) 0 donc f est croissante sur 3

2; + `⎡

⎣⎡⎣.

2. f ’(x) = 4x 3 – 400x = 4x(x 2 – 100).Sur ]– ; – 10] et sur [0 ; 10], f ’(x) 0 donc f est décroissante sur chacun de ces intervalles.Sur [–10 ; 0] et sur [10 ; + [, f ’(x) 0 donc f est croissante sur chacun de ces intervalles.3. f ’(x) = 2x 3 – 3x 2 – 5x = x(2x 2 – 3x – 5).Les racines du polynôme 2x 2 – 3x – 5 sont – 1 et 5

2.

Sur ]– ; –1] et sur 0 ; 52

⎡⎣

⎤⎦ , f ’(x) 0 donc f est décroissante sur

chacun de ces intervalles.

Sur [–1 ; 0] et sur 52

; + `⎡⎣

⎡⎣, f ’(x) 0 donc f est croissante sur

chacun de ces intervalles.72 Fichier associé sur www.bordas-indice.fr et sur le

CD-Rom Professeur :03_TSTMG_exercice72.alg (AlgoBox).

42 1. u’ (x) = 6x, v’ (x) = 10x 4 et w’ (x) = – 15x 4.2. f ’(x) = u’ (x) + v’ (x) = 6x + 10x 4,g’(x) = v’ (x) + w’ (x) = 10x 4 – 15x 4 = – 5x 4,et h’ (x) = u’ (x) + w’ (x) = 6x – 15x 4.3. g(x) = – x 5 donc g’(x) = – 5x 4.43 1. f ’(x) = 30x 2 – 4x.

2. f ’(x) = 2x 3 – 143

x + 5.

44 1. f ’(x) = 0,003 – 0,14x + 0,008x 3.2. f ’(x) = 3 000x 2 – 410x + 102.45 1. f ’(x) = 300(3x 2 – 8x) = 900x 2 – 2 400x.

2. f ’(x) = 0,05(4x 3 – 2x) = 0,2x 3 – 0,1x.46 1. f ’(x) = 100x + 3.

2. f ’(x) = 3x 2 – 10x – 13.47 Voir manuel page 251.48 Exercice résolu.49 1. f ’(x) = 3x 2 + 4x – 5 et g’(x) = 3f ’(x) = 9x 2 + 12x – 15.

2. f ’(x) = 300 – 1 500x 2 et g’(x) = 0,01f ’(x) = 3 – 15x 2.50 b(q) = 500q − (5q 2 − 100q + 200)

= – 5q 2 + 600q – 200.b’(q) = − 10q + 600.51 1. B(x) = 680x – (15x 3 − 120x 2 + 500x)

= – 15x 3 + 120x 2 + 180x.B’(x) = − 45x 2 + 240x + 180.2. CM(x) = 15x 2 –120x + 500CM’(x) = 30x – 120.52 1. f ’(x) = – 3x 2 – 3x + 18.

2. 3(2 – x)(x + 3) = 3(– x 2 – x + 6) = –3x 2 – 3x + 18donc on a bien f ’(x) = 3(2 – x)(x + 3).53 1. f ’(x) = 20x 3 – 6x 2 – 2x.

2. 2x(10x 2 – 3x –1) = 20x 3 – 6x 2 – 2xdonc on a bien f ’(x) =2x(10x 2 – 3x –1).54 1. h’(x) = 4x 3 et g’(x) = 4x 3.

2. a. D’après le cours, cette proposition est vraie.b. Proposition réciproque :« Si pour tout réel x, f ’(x) = 4x 3 alors pour tout réel x, f (x) = x 4 ».c. Cette nouvelle proposition est fausse :pour f telle que f (x) = x 4 + 5, f ’(x) = 4x 3 mais f (x) ≠ x 4.55 Faux, f ’(x) = 6x 2 + 6x.56 Vrai, f ’(x) = g’(x) = 6x 2 + 6x.57 Vrai, h ’(x) = 1

6f ’(x) = x 2 + x.

58 Vrai, k (x) = f (x) – g (x) = 11 donc k’(x) = 0.59 Voir manuel page 251.60 f est décroissante sur R.61 1. a. Lorsque a = 0,07, l’algorithme affiche « f est

croissante ».b. Lorsque a = 0, l’algorithme affiche « f est constante ».c. Lorsque a = – 0,1, l’algorithme affiche « f est décroissante ».2. Cet algorithme affiche le sens de variation d’une fonction affine, l’utilisateur ayant saisi le coefficient de x.62 f ’(x) = – 2x + 160.

Sur ]– ; 80], f ’(x) 0 donc f est croissante sur ]– ; 80].Sur [80 ; + [, f ’(x) 0 donc f est décroissante sur [80 ; + [.63 f ’(x) = 2x – 4.

23Chapitre 3 Fonctions polynômes

3.

x – 2 – 1 0 7 8

4x – – 0 + +

x + 1 – 0 + + +

x – 7 – – – 0 +

f ’(x) – 0 + 0 – 0 +

f28

– 1

4

–1 025

–892

4. Sur l’intervalle [– 2 ; 8], f admet un maximum égal à 28 et un minimum égal à – 1 025. 78 1. f ’(x) = x 3 + 3x 2 – 4x.

x(x 2 + 3x – 4) = x 3 + 3x 2 – 4x = f ’(x).2. Les racines du polynôme x 2 + 3x – 4 sont – 4 et 1.

x – 5 – 4 0 1 2

x – – 0 + +

x 2 + 3x – 4 + 0 – – 0 +

f ’(x) – 0 + 0 – 0 +

f–19,75

– 33

–1

–1,75

3

3. Sur l’intervalle [– 5 ; 2], f admet un maximum égal à 3 et un minimum égal à – 33.79 Fichiers associés sur www.bordas-indice.fr et sur le

CD-Rom Professeur :03_TSTMG_exercice79.xls (Excel 2003),03_TSTMG_exercice79.xlsx (Excel 2007),03_TSTMG_exercice79.ods (OpenOffice).1. a. Formule dans la cellule A3 : =A2+0,5 .

b. Formule dans la cellule B2 : =9*A2^4+28*A2^3 .c. Le minimum semble être égal à – 85,9375.2. a. f ’(x) = 36x 3 + 84x 2.b. f ’(x) = 12x 2(3x + 7).

x – 4 –73

0 2

f ’(x) – 0 + 0 +

f

512

–2 40127

0

368

c. Le minimum de f sur l’intervalle [– 4 ; 2] est –2 40127

(soit – 88,9 à 10–1 près).80 1. Le volume du rangement est x(12 – x) 2, soit

f (x) = x(x 2 − 24x + 144) = x 3 − 24x 2 + 144x.2. f ’(x) = 3x 2 – 48x + 144. f ’(x) a pour racines 4 et 12.

x 0 4 12

f ’(x) + 0 – 0

f0

256

0

3. Le rangement a un volume maximal pour x = 4.Ce volume maximal est 256 dm3.

1. B(x) et D(x) peuvent être factorisées par x.2. On complète par « d = 0 »3. Pour A(x) : a = 2, b = 3, c = 4 et d = 5.Pour B(x) : a = 1, b = 1, c = 1 et d = 0.Pour C(x) : a = – 2, b = 0, c = 7 et d = 1.Pour D(x) : a = 7, b = 3, c = 0 et d = 0.4. • AlgoBox

• Calculatrice Casio

• Calculatrice Texas

73 Faux. f est une fonction affine et le coefficient de x, égal à 0,1, est strictement positif donc f est croissante sur R.74 Vrai. g’(x) = – 4x + 4.

Sur ]– ; 1], g’(x) 0 donc g est croissante sur ]– ; 1] .Sur [1 ; + [, g’(x) 0 donc g est décroissante sur [1 ; + [.75 Vrai. h’(x) = – 6x 2 + 6x – 5. D = – 84.

Le coefficient de x 2, égal à – 6, est strictement négatif donc pour tout réel x, h’(x) 0 et h est décroissante sur R.76 Vrai. k’(x) = p’(x) = 8x 3 – 4.

k’ = p’ donc k’ et p’ sont de même signe et les fonctions k et pont les mêmes variations sur R.77 1. f ’(x) = 4x 3 – 24x 2 – 28x.

2. a. (x + 1)(x – 7) = x 2 – 6x – 7.b. 4x(x + 1)(x – 7) = 4x(x 2 – 6x – 7) = 4x 3 – 24x 2 – 28xdonc on a bien f ’(x) = 4x(x + 1)(x – 7).

24

et à g aux points d’abscisse 0 n’ont pas le même coefficient directeur.96 Vrai, car g’ (2) = 0.97 Vrai. g (0) = 0 et g’ (0) = − 12 donc une équation de la

tangente à g au point d’abscisse 0 est bien y = − 12x.98 1. Résolution graphique

a. Les solutions sont environ – 1 ; 0,5 et 1.b. L’ensemble des solutions est [– 1 ; 0,5] [1 ; + [.2. a. Pour tout réel x, (2x – 1)(x 2 – 1) = 2x 3 – 2x – x 2 + 1.b. 2x 3 – x 2 – 2x + 1 = 0 équivaut à (2x – 1)(x 2 – 1) = 0 donc à 2x – 1 = 0 ou x 2 – 1 = 0. S = {0,5 ; – 1 ; 1}.2x 3 – x 2 – 2x + 1 0 équivaut à (2x – 1)(x 2 – 1) 0.

x – –1 0,5 1 +

2x – 1 – – 0 + +

x 2 – 1 + 0 – – 0 +

(2x – 1)(x 2 – 1) – 0 + 0 – 0 +

S = [–1 ; 0,5] [1 ; + [.99 1. Résolution graphique

a. La solution est 3.b. L’ensemble des solutions est ]– ; 3].2. a. Pour tout réel x,(3 – x)(x 2 – 2x + 3) = 3x 2 – 6x + 9 – x 3 + 2x 2 – 3x

= – x 3 + 5x 2 – 9x + 9.b. – x 3 + 5x 2 – 9x + 9 = 0 équivaut à (3 – x)(x 2 – 2x + 3) = 0.x 2 – 2x + 3 = 0 n’a pas de solution puisque D 0 (D = – 8), donc S = {3}.– x 3 + 5x 2 – 9x + 9 0 équivaut à (3 –x)(x 2 – 2x + 3) 0.

x – 3 +

3 – x + 0 –

x 2 – 2x + 3 + +

(3 – x)(x 2 – 2x + 3) + 0 –

S = ]– ; 3].100 Exercice résolu.101 Fichiers associés sur www.bordas-indice.fr et sur le CD-Rom Professeur :03_TSTMG_exercice101.xls (Excel 2003),

4.

x 0 1 2 6 7 8 10 12

f (x) 0 121 200 216 175 128 40 0

5.

0

y

x5 10

100

200

6. Le volume est égal à 180 dm3 pour x ≈ 1,8 et x ≈ 6,9.81 Voir manuel page 251.82 Faux. f ’(x) = 2x – 6.

Sur [0 ; 3], f ’(x) 0 donc f est décroissante sur [0 ; 3].Sur [3 ; 4], f ’(x) 0 donc f est croissante sur [3 ; 4].f admet donc un minimum en 3.Ce minimum est f (3) = – 8.83 Vrai. g’(x) = – 6x 2 + 8x – 3. D = − 8.

Pour tout x, g’(x) 0 donc g est décroissante sur [− 1 ; 1] et le maximum de g sur [− 1 ; 1] est g (− 1) = 9.84 f ’(– 1) = 9,96, f ’(10) = – 60 et f ’(0,2) = – 1,99968.85 f ’(– 1) = 9 , f ’(10) = 50 301 et f ’(0,2) = 1,128.86 f ’(x) = 3x 2 − 2.

Une équation de la tangente est : y = 10x + 17.87 f ’(x) = – 4x 3 + 4x – 1.

Une équation de la tangente est : y = – x + 1.88 f ’(x) = – 10x 4 + 6x 2.

Une équation de la tangente est : y = – 4x – 46.89 Exercice résolu.90 1. Graphiquement, f (− 5) = 2 et f ’(−5) = 9

5.

Une équation de T1 est : y = 95

x + 11.

2. Graphiquement, f (3) = 10 et f ’(3) = –62

= – 3.

Une équation de T2 est : y = – 3x + 19.91 1. Graphiquement, f (− 1) = − 4 et f ’(−1) = 4.

Une équation de la tangente est : y = 4x.2. Graphiquement, f (0) = 0 et f ’(0) = 2.Une équation de la tangente est : y = 2x.92 Voir manuel page 251.93 Exercice résolu.94 On note C la variable qui prend la valeur du coefficient

directeur de la tangente.

a, b, c, t et C sont des réelsSaisir a, b, c et tC prend la valeur 2at + bAfficher C

95 Faux.f ’(0) = – 7 et g ’(0) = – 12 : f ’(0) ≠ g ’(0) donc les tangentes à f

25Chapitre 3 Fonctions polynômes

2.

x – 1 2,5 5

f ’(x) + 0 –

f– 106

– 93,75

– 100

3. Sur [– 1 ; 5], f admet un maximum de – 93,75 et un minimum de – 106.107 1. f ’(x) = 6x 2 + 18x.2. f ’(x) = 6x 2 + 18x = 6x(x + 3)

x – 4 – 3 0 1

f ’(x) + 0 – 0 +

f15

26

– 1

10

108 1. f ’(x) = 8x 3 + 12x.2. La tangente a pour équation y = 20x – 11.109 1. Graphiquement, les solutions sont environ 0, 2 et 3.

2. a. (3 – x)(x – 2) = – x 2 + 5x – 6.b. Pour tout réel x,x(3 – x)(x – 2) = x(– x 2 + 5x – 6) = – x 3 + 5 x 2 – 6x.c. –x 3 + 5x 2 – 6x = 0 équivaut à x(3 – x)(x – 2) = 0, donc à x = 0 ou x = 3 ou x = 2.

P O U R FA I R E L E P O I N TVoir manuel page 251. Les corrigés détaillés sont sur le site www.bordas-indice.fr

ACCO M PAG N E M E N T P E R S O N N A L I S É110 1. f ’(x) = 2x – 6.2. f ’(x) = – 15x 2 + 2x – 2.3. f ’(x) = 8x 3 – 3x 2 + 2.4. f ’(x) = x 3 + x + 1.111 1. f ’(x) = – 6x 2 + 2x – 3. D = – 68.Pour tout réel x, f ’(x) 0 donc f est décroissante sur R.2. f est croissante sur ]– ; – 1] et sur 5

9; + `⎡

⎣⎡⎣ et décroissante

sur –1 ; 59

⎡⎣

⎤⎦ .

3. f est décroissante sur –` ; 13

⎤⎦

⎤⎦ et sur [1 ; + [ et croissante

sur 13

; 1⎡⎣

⎤⎦ .

4. f ’(x) = 3x 2 + 6x + 3 = 3(x 2 + 2x + 1)= 3(x + 1) 2.Pour tout réel x, f ’(x) 0 : f est croissante sur R.

03_TSTMG_exercice101.xlsx (Excel 2007),03_TSTMG_exercice101.ods (OpenOffice).1. Dans la cellule D2 :

=0,5*B2^3-9*B2^2+48*B2-25 .f (x) = 0,5x 3 – 9x 2 + 48x –25.2. La cellule D12 contient la valeur de f (11), soit 79,5.3. Le cours de 2007 est 55 euros.D’après les résultats affichés, en 2013 l’action retrouve ce cours.4. a. f (x) – 55 = 0,5x 3 – 9x 2 + 48x –80 et (x 2 – 14x + 40)(0,5x – 2) = 0,5x 3 – 9x 2 + 48x – 80donc f (x) – 55 = (x 2 – 14x + 40)(0,5x – 2).b. f (x) – 55 = 0 équivaut à (x 2 – 14x + 40)(0,5x – 2) = 0.L’équation 0,5x – 2 = 0 a pour solution 4 et l’équationx 2 – 14x + 40 = 0 a pour solutions 4 et 10donc S = {4 ; 10}.c. f (x) – 55= 0 équivaut à f (x) = 55.D’après la question précédente, les solutions sont 4 et 10.Pour x = 4, c’est-à-dire en 2007 et pour x = 10, c’est-à-dire en 2013, le cours de l’action est de 55 euros.102 Fichier associé sur www.bordas-indice.fr et sur le CD-Rom Professeur :03_TSTMG_exercice102.xws (Xcas).1. – 2x 3 + 53x 2 – 246x = 0 équivaut à x(– 2x 2 + 53x – 246) = 0, donc à x = 0 ou – 2x 2 + 53x – 246 = 0.On résout l’équation – 2x 3 + 53x – 246 = 0.

= 841 d’où x1 = –53 – 29–4

= 20,5 et x2 = –53 + 29–4

= 6.

Les solutions sont bien 0 ; 6 et 412

.

2. – 2x 3 + 53x 2 – 246x 0 équivaut à x(– 2x 2 + 53x – 246) 0.

x – 0 6 20,5 + x – 0 + + +

– 2x 2 + 53x – 246 – – 0 + 0 –

produit + 0 – 0 + 0 –

S = ]– ; 0] [6 ; 20,5].3. L’entreprise réalise un bénéfice lorsque f (x) 0.Sur l’intervalle [1 ; 21], l’ensemble des solutions est [6 ; 20,5], donc l’entreprise réalise un bénéfice lorsqu’elle fabrique et vend entre 600 et 2 050 bijoux.103 Vrai.x 3 + 30x 2 – x = 30 équivaut à x 3 + 30x 2 – x – 30 = 0, soit à f (x) = 0.104 1. Vrai.(x 2 – 1)(x + 30) = x 3 + 30x 2 – x – 30 = f (x).2. Faux, dans R, les solutions sont – 1, 1 et – 30.3. Faux.

x – –30 –1 1 + x 2 – 1 + + 0 – 0 +

x + 30 – 0 + + +

f (x) – 0 + 0 – 0 +

S = ]– ; – 30[ ]– 1 ; 1[.105 f ’(x) = 4x, g’(x) = 3x 2 + 1 et h’(x) = 4x + 3x 2 + 1.106 1. f ’(x) = – 2x + 5.

26

c. Il n’y aucune modification lorsque x ≈ 84.d. Il y a une augmentation globale de 40 % lorsque x ≈ 21 ou x ≈ 73.e. L’augmentation est la plus forte pour x = 50.

C. Avec une fonction polynôme1. f (t ) = (1 + 2t + t 2)(1 – t 2) = – t 4 – 2t 3 + 2t + 1.2. f ’(t ) = – 4t 3 – 6t 2 + 2.3. – 4(t + 1)2(t – 0,5) = (t 2 + 2t + 1)(– 4t + 2)

= – 4t 3 – 6t 2 + 2et f ’(t ) = – 4t 3 – 6t 2 + 2 donc f ’(t ) = – 4(t + 1)2(t – 0,5).4.

x 0 0,5 1

f ’(t) + 0 –

f1

1,6875

0

5. L’augmentation est la plus forte pour t = 0,5, soit x = 50 : on retrouve le résultat de la question B. 5. e.

P O U R A P P R O F O N D I R112 1. Pour tout x appartenant à l’intervalle ]0 ; 10], CM (x) = 15x 2 – 120x + 500 donc C M’(x) = 30x – 120 = 30(x − 4).2. a.

x 0 4 10

CM’(x) – 0 +

CM 260

800

b. CM admet un minimum de 260 pour x = 4 : pour 4 kilomètres de tissu produits, le coût moyen de production est minimal et égal à 260 euros.Le coût total est alors égal à 1 040 euros.3. a.

0

y

x1 2 3 4 5 6 7 8 9 10

100

200

300

400

500

600

700

800

D

Si p = 200, le coût moyen est toujours supérieur à ce prix unitaire de 200 euros : l’entreprise ne peut pas réaliser de bénéfice.b. Pour que l’entreprise puisse espérer faire un bénéfice, le prix p doit être supérieur à 260 euros.

T R AVAU X P R AT I Q U E STP De la laine à foison

A. Modélisation du problème1. Le coefficient multiplicateur associé à une hausse de x % est

1+ x100( ) , celui associé à une baisse de x % est 1– x

100( ) .

Le coefficient multiplicateur associé à trois hausses successives de x % suivies d’une baisse de x % est donc égal à

1+ x100( )3 1– x

100( ) .

2. a. Il y a une augmentation de 38,24 %.b. Il y a une augmentation de 68,75 %.c. Il y a une augmentation d’environ 36,97 %.d. Il y a une baisse d’environ 5,03 %.

B. Avec un tableurFichiers associés sur www.bordas-indice.fr et sur le CD-Rom Professeur :03_TSTMG_TP_correction.xls (Excel 2003),03_TSTMG_TP_correction.xlsx (Excel 2007),03_TSTMG_TP_correction.ods (OpenOffice).1. Entrer dans la cellule A2 le nombre 0 .

Saisir dans la cellule A3 la formule =A2+1 et recopier cette formule vers le bas jusqu’à la cellule A102.2. a. et b. Sais i r dans la cel lule B2 la formule =(1+A2/100)^3*(1-A2/100) et recopier cette formule vers le

bas jusqu’à la cellule B102.3. a. La formule dans la cellule C2 est : =B2-1 .Les résultats correspondent au taux d’évolution global à l’issue des quatre modifications.b. Mettre la plage de cellules C2:C102 au format pourcentage.Saisir dans la cellule C2 la formule =B2-1 et recopier cette formule vers le bas jusqu’à la cellule C102.4. Sélectionner les cellules de la colonne A puis avec la touche Ctrl laissée enfoncée, celles de la colonne C.Avec Excel : Choisir Insertion et Nuage de points .Avec OpenOffice : Choisir Insertion puis Diagramme et XY .

5. a. Il y a une baisse globale lorsque x est supérieur à 84.b. Il y a une augmentation globale lorsque x est inférieur à 83.

27Chapitre 3 Fonctions polynômes

b. Conjecture : T semble au-dessous de f pour x – 0,5 et au-dessus de f ou confondue avec f pour x – 0,5.B. Avec un tableur1. a. La formule dans D2 est : = B2 – C2 .g(x) = f (x) – (6x – 2).b. En D5 : – 0,25 ; en D6 : 0 ; en D7 : – 0,5 ; en D8 : – 2,5.c. Conjecture sur le signe de g(x) :sur [– 1 ; – 0,5], g(x) 0 et sur [– 0,5 ; 2], g(x) 0.Conjecture sur la position de f par rapport à T :sur [– 1 ; – 0,5], g(x) 0 donc f (x) 6x – 2 et f est au-dessus de T ; sur [– 0,5 ; 2], g(x) 0 donc f (x) 6x – 2 et f est au-dessous de T.2. a. g(x) = f (x) – (6x – 2) = – x 3 + 1,5x 2 – 0,5.(x – 1)2(–x – 0,5) = (x 2 – 2x + 1)(– x – 0,5)

= –x 3 + 1,5x 2 – 0,5donc g(x) = (x – 1)2(– x – 0,5).b.

x – – 0,5 1 + – x – 0,5 + 0 – –

(x – 1)2 + + 0 +

g (x) + 0 – 0 –

Sur ]– ; – 0,5], g (x) 0 donc f est au-dessus de T.Sur [– 0,5 ; + [, g (x) ≤ 0 donc f est au-dessous de T.

C A P V E R S L E B AC

Zoom sur le tableur

Fichiers associés sur www.bordas-indice.fr et sur le CD-Rom Professeur :03_TSTMG_zoomtableur.xls (Excel 2003),03_TSTMG_zoomtableur.xlsx (Excel 2007),03_TSTMG_zoomtableur.ods (OpenOffice).

Sujet AFichiers associés sur www.bordas-indice.fr et sur le CD-Rom Professeur :03_TSTMG_sujetA.xls (Excel 2003),03_TSTMG_sujetA.xlsx (Excel 2007),03_TSTMG_sujetA.ods (OpenOffice).A. Étude des coûts donnés par le tableur1. Formule dans la cellule A3 : =A2+5 .

2. =A2^3/3-11*A2^2+100*A2+72 .

B. Étude graphique du bénéfice1. Le coût total de production est de 600 euros pour environ 22,5 kg de produit.2. a. R(x) = 60x.

113 A. Lectures graphiques1. Réponse b : f coupe deux fois l’axe des abscisses.2. Réponse c : f est strictement au-dessus de l’axe des abscisses sur cet intervalle.3. Réponse b : la tangente à f est horizontale aux points A et E d’abscisses respectives – 2 et 0.4. Réponse c : la fonction f est décroissante sur cet intervalle.5. Réponse c : f ’ (– 1) est le coefficient directeur de D, soit – 3.6. Réponse b : D a pour coefficient directeur – 3 et pour ordonnée à l’origine – 5 donc son équation est y = – 3x – 5.7. Réponse a : f est croissante sur [– 3 ; – 2] et sur [0 ; 1,5] donc f ’(x) 0 sur chacun de ces intervalles.f est décroissante sur [– 2 ; 0] donc f ’(x) 0 sur [– 2 ; 0].La courbe a est la seule représentant une fonction positive sur [– 3 ; –2] [0 ; 1,5] et négative sur [– 2 ; 0].

B. Preuves1. a. (x + 2) (x 2 + x – 2) = x 3 + 3x 2 – 4 et f (x) = x 3 + 3x 2 – 4donc f (x) = (x + 2) (x 2 + x – 2).b. f (x) = 0 équivaut à x + 2 = 0 ou x 2 + x – 2 = 0.L’équation x + 2 = 0 a pour solution – 2 et l’équationx 2 + x – 2 = 0 a pour solutions – 2 et 1 donc S = {– 2 ; 1}.c. f (x) 0 équivaut à (x + 2) (x 2 + x – 2) 0.

x – 3 – 2 1 1,5

x + 2 – 0 + +

x 2 + x – 2 + 0 – 0 +

f (x) – 0 – 0 +

S = ]1 ; 1,5].2. a. f ’(x) = 3x 2 + 6x = 3x(x + 2).b. f ’(x) = 0 équivaut à 3x(x + 2) = 0 et donc à x = 0 ou x = – 2. S = {– 2 ; 0}.c.

S = [– 2 ; 0].d. La courbe a. est la seule qui correspond au tableau de signes ci-dessus.3. a. f ’(– 1) = 3 – 6 = – 3.b. D a pour coefficient directeur f ’(– 1) = – 3 et passe par le point B(– 1 ; – 2) : une équation de D est y = – 3x – 5.114 Fichiers associés sur www.bordas-indice.fr et sur le CD-Rom Professeur :03_TSTMG_exercice114.xls (Excel 2003), 03_TSTMG_exercice114.xlsx (Excel 2007),03_TSTMG_exercice114.ods (OpenOffice).A. Avec une calculatrice1. T a pour équation y = 6x –2.2. a.

x – 3 – 2 0 1,5

x + 2 – 0 + +

3x – – 0 +

f ’(x) + 0 – 0 +

28

3.

0

y

x1 2 3 4 5 6 7 8 9 10

100 000

200 000

300 000

400 000

500 000

600 000 �

B. 1. Le chiffre d’affaires minimal a été de 187 500 euros en 2007.

2. 650 000 – 187 500187 500

×100 ≈ 246,7.

L’augmentation entre le chiffre d’affaires minimal et le chiffre d’affaires maximal a été d’environ 246,7 %.3. Graphiquement f (x) 400 000 pour x 8,5.À partir de 2011, le chiffre d’affaires a dépassé celui de 2002.

Sujet CFichiers associés sur www.bordas-indice.fr et sur le CD-Rom Professeur :03_TSTMG_sujetC.xls (Excel 2003),03_TSTMG_sujetC.xlsx (Excel 2007),03_TSTMG_sujetC.ods(OpenOffice).

1. a. Formule dans la cellule C1 : =B1+1 .

b. Formule dans la cellule B2 : =B1-$B$1 .

c. Formule dans la cellule H2 : =H1-$B$1 .d. La cellule G3 contient la formule :

=-0,01*G2^4+0,2*G2^3-2*G2^2+10*G2+3 .La valeur affichée est :– 0,01 × 54 + 0,2 × 53 – 2 × 52 + 10 × 5 + 3 = 21,75.2. a. f ’(x) = – 0,04x 3 + 0,6x 2 – 4x + 10 et0,04(x – 5)(– x 2 +10x – 50) = – 0,04x 3 + 0,6x 2 – 4x + 10donc f ’(x) = 0,04(x – 5)(–x 2 +10x – 50).b. Pour tout réel x, – x 2 +10x – 50 0 donc f ’(x) est du signe contraire de 0,04(x – 5).Sur [0 ; 5], f ’(x) 0 donc f est croissante sur [0 ; 5].Sur [5 ; 8], f ’(x) 0 donc f est décroissante sur [5 ; 8].c. f admet un maximum pour x = 5. Ce maximum est f (5) = 21,75. Le nombre maximal de ventes a été de 21 750 en 2010.

Sujet D1. a.

x 0 4 6

f’(x) – 0 +

f5

1

3

b.

0

y

x5

100

c. Graphiquement, la quantité (en kg) à produire et vendre pour réaliser un bénéfice doit appartenir à [6 ; 28].

C. Étude algébrique du bénéfice1.

La fonction B semble croissante sur [5 ; 20] et décroissante sur [20 ; 30].2. B’(x) = −x 2 + 22x – 40et − (x −2)(x −20) = (– x + 2)(x – 20) = – x 2 + 22x – 40donc B’(x) = −(x − 2)(x −20).3. Sur [5 ; 20], B’(x) 0 donc B est croissante sur [5 ; 20].Sur [20 ; 30], B’(x) 0 donc B est décroissante sur [20 ; 30].4. a. Pour réaliser le bénéfice maximal, il faut produire et vendre 20 kg de produit.b.

x 15 20 24

B678

B(20)

696

Le bénéfice minimal envisageable est de 678 euros.

Sujet BA. 1. a. f ’(x) = 1 000(3x 2 – 3x – 60) = 3 000(x 2 – x – 20).b. Le polynôme x 2 – x – 20 a deux racines : 5 et – 4. Sur [0 ; 5], f ’(x) 0 et sur [5 ; 10], f ’(x) 0.c.

x 0 5 10

f’(x) – 0 +

f400 000

187 500

650 000

2.

x 0 2 4 6 8 10f (x) 400 000 282 000 200 000 202 000 336 000 650 000

29Chapitre 3 Fonctions polynômes

2. Sur –2 ; 12

⎡⎣

⎤⎦, h ’(x) 0, donc h est croissante sur –2 ; 1

2⎡⎣

⎤⎦ .

Sur 12

; 1⎡⎣

⎤⎦ , h ’(x) 0, donc h est décroissante sur 1

2; 1⎡

⎣⎤⎦ .

3. Les variations de h sont celles de f4 : h est f4.

Sujet F1. Réponse c.2. Réponse c.3. Réponse a.4. Réponse a.5. Réponse a.6. Réponse a.

Sujet G1. Faux.2. Faux.3. Vrai.4. Vrai.5. Vrai.6. Vrai.7. Vrai.8. Faux.9. Vrai.10. Faux.

115 1. f ’(x) = – 4x + 8.f est croissante sur ]– ; 2] et décroissante sur [2 ; + [.2. Une équation de la tangente est : y = 4x + 1.116 1. f ’(x) = 3x 2 – 2x – 5 et (3x – 5)(x + 1) = 3x 2 – 2x – 5donc f ’(x) = (3x – 5)(x + 1).

2. Sur ]– ; – 1] et sur 53

; + `⎡⎣

⎡⎣, f ’(x) 0 donc f est croissante

sur chacun de ces intervalles.

Sur –1 ; 53

⎡⎣

⎤⎦ , f ’(x) 0 donc f est décroissante sur –1 ; 5

3⎡⎣

⎤⎦ .

117 1. f ’(x) = – 12x 3 + 24x 2.En factorisant par 12x 2, on obtient f ’(x) = 12x 2(2 – x).2.

x – 1 0 2 3

f ’(x) + 0 + 0 –

f

– 12

– 1

15

– 28

3. Sur [– 1 ; 3], f admet un maximum égal à 15 et un minimum égal à –28.

b. f ’(6) = 2 et f (6) = 3.Une équation de la tangente est : y = 2x – 9.c.

0

y

x1

1

2. g’(x) = – 10f ’(x) donc g ’(x) et f ’(x) sont de signes contraires.

x 0 4 6

g’(x) + 0 –

g– 50

– 10

– 30

Sujet EA. 1.

Fonction f1 f2 f3 f4

Tableau de signes b d c a

2.

Fonction f1 f2 f3 f4

Tableau de variation c b a d

3.

Fonction f1 f2 f3 f4

Signe de la dérivée c b d a

B. 1. g(x) = (1 – x)(x 2 + 2x + 1) = − x 3 − x 2 + x + 1.2. g ’(x) = – 3x 2 – 2x + 1.3. Les racines de g ’(x) sont 1

3 et – 1.

Sur [– 2 ; – 1] et sur 13

; 1⎡⎣

⎤⎦ , g ’(x) 0 donc g est décroissante sur

chacun de ces intervalles.

Sur –1 ; 13

⎡⎣

⎤⎦ , g ’(x) 0 donc g est croissante sur –1 ; 1

3⎡⎣

⎤⎦ .

4. Les variations de g sont celles de f1 : g est f1.

C. 1. h ’(x) = – 4x 3 – 6x 2 + 2et (x + 1)2(– 4x + 2) = (x 2 + 2x + 1)(– 4x + 2)

= – 4x 3 – 6x 2 + 2donc h ’(x) = (x + 1)2(– 4x + 2).

30

Le programme

4CHAPITRE

Fonctions rationnelles

L’objectif de ce chapitre est de découvrir la dérivée d’une nouvelle fonction de référence : la fonction inverse et d’apprendre à calculer la dérivée d’un quotient. Comme dans le chapitre précédent, les élèves seront amenés à utiliser la dérivation pour étudier le sens de variation d’une fonction et pour déterminer une équation d’une tangente à la courbe représentative de cette fonction.

– Dans le Avant de commencer, plusieurs exercices permettent de tester si l’élève sait déterminer le signe d’une expression sous la forme d’un quotient. C’est un acquis de seconde qui est indispensable lors de chaque étude de fonctions utilisant la dérivation. Un rappel figure en fin de livre page 225.

– Les deux activités sont très courtes afin de faire découvrir aux élèves les deux nouvelles formules de dérivation.Les Savoir-faire sont constitués d’exercices élémentaires, applications immédiates du cours.Après l’étude de chacun de ces Savoir-faire, l’élève pourra s’entraîner sur les exercices signalés.

– Les exercices sont progressifs et font appel à la capacité de l’élève à lire des graphiques, calculer des dérivées et étudier le signe de ces dérivées.De nombreux exercices proposent des problèmes d’optimisation dans le cadre de problème de coûts.

Les notions abordées dans le chapitre 4 • Fonction dérivée d’un quotient• Étude d’une fonction rationnelle

B Notre point de vue

Objectifs :• Étendre l’étude de la dérivation au cas des fonctions polynômes ou rationnelles.• Consolider l’utilisation des fonctions dans le cadre de résolutions de problèmes, en lien avec les enseignements technologiques.• Utiliser de façon complémentaire les différents outils de calcul et de représentation (à la main, à la calculatrice, au tableur…) et l’algorithmique.

Contenus Capacités attendues Commentaires

DérivationFonction dérivée de

x 1x

.• Connaître la fonction dérivée de x 1

x. L’étude des ensembles de définition et de

dérivation n’est pas un objectif du programme.

Fonction dérivée d’un quotient de fonctions.Application à l’étude des variations des fonctions.

Dans le cadre d’une résolution de problème :• déterminer la fonction dérivée d’une fonction rationnelle ;• étudier les variations et les extremums d’une fonction à partir du signe de sa fonction dérivée ;• déterminer une équation de la tangente en un point d’une courbe représentative ; tracer cette tangente.

On se limite à des fonctions simples.

Cette partie du programme se prête particulièrement à l’étude de situations issues des autres disciplines (résolutions graphiques ou numériques d’équations et d’inéquations, problèmes d’optimisation…).

A

31Chapitre 4 Fonctions rationnelles

Le QCM et les exercices proposés dans cette page permettent de tester les connaissances des élèves sur, d’une part, les calculs de dérivée des fonctions polynômes et leurs utilisations vues dans le chapitre 3, et, d’autre part les études de signe de quotients et les transformations d’écriture. En effet, lors de certaines études de fonctions rationnelles, les élèves pourront être amenés à transformer l’écriture de l’expression donnée.Voir manuel page 251 et le site www.bordas-indice.fr pour les corrigés détaillés.

C Avant de commencer

D ActivitésActivité Fonction dérivée de la fonction

inverse1

Fichier associé sur www.bordas-indice.fr et le CD-Rom Professeur :04_TSTMG_activite1.ggb (GeoGebra).L’objectif est de découvrir la dérivée de la fonction inverse à l’aide de quelques lectures graphiques de coefficient directeur de tangentes.1. a. Le coefficient directeur de la tangente à la courbe au point A d’abscisse 1 est égal à –1.Le coefficient directeur de la tangente à la courbe au point B d’abscisse 2 est égal à – 0,25.Le coefficient directeur de la tangente à la courbe au point C d’abscisse 0,5 est égal à – 4.b.

x 1 2 0,5f ’ (x) –1 – 0,25 – 4x2f ’ (x) –1 –1 –1

c. On remarque que tous les résultats sont égaux à –1.

d. On émet la conjecture que f ’ (x) = – 1x2

.

2. En déplaçant le point M à l’aide du logiciel, on peut lire la valeur de x et celle de f ’ (x) ; après plusieurs calculs de x2f ’ (x) on observe que l’on trouve à chaque fois –1 comme résultat.

On peut compléter le fichier fourni en utilisant le tableau de valeurs de GeoGebra.

Activité Fonction dérivée d’un quotient de deux fonctions

2

L’objectif est de découvrir, sur un exemple, la formule permettant de dériver le quotient de deux fonctions.

1. a. f est la somme des fonctions g et h définies sur ]0 ; + [ par g(x)= x et h(x)= 1

x ; on a g’(x) = 1 et h’(x)= – 1x2

.

b. f ’ (x) = 1 – 1x2

.

c. f ’ (x) = x2 −1x2

.

2. a. f (x) = x + 1x= x2 +1

x ; u(x)= x2 + 1 et v(x) = x donc u’ (x)= 2x

et v’(x)=1.

b. u’(x )v ’(x )

= 2x1

= 2x ; ce résultat n’est pas égal à celui obtenu

à la question 1. c.c. u’ (x) × v(x) = 2x(x) = 2x 2 et u(x) × v’ (x) = x 2 + 1donc u’ (x) × v(x) – u(x) × v’ (x) = x 2 – 1.d. En comparant les résultats, on voit que, sur cet exemple, la formule donnée permet d’obtenir f ’ (x).

E Exercices5 f ’ (x) = 14x et g ’ (x) = − 7

x2.

6 1. k = 11 et u(x)= 1x

.

2. f ’ (x) = − 11x2

.

7 f (x) = 1,5 × 1x donc f ’ (x) = –1,5 × 1

x2,

g (x) = – 2 × 1x donc g’(x) = 2

x2.

8 1. u(x) = 3x et v(x)= 12x

,

u’(x) = 3 et v’(x) = – 12x2

.

2. f ’ (x) = 3 – 12x2

.

9 1. u’(x) = 4 et v’(x) = 1.

P O U R D é M A R R E R1 1. f ’ (x) = – 1

x2.

2. g ’ (x) = – 1x2

.

2 f ’ (x) = 2x et g ’ (x) = – 1x2

.

3 1. u(x)=2x et v(x)= 1x

,

u’(x) = 2 et v’(x) = – 1x2

.

2. f ’ (x) = 2 – 1x2

.

4 f ’ (x) = 3 – 1x2

et g ’ (x) = 2x + 1x2

.

32

b. x 2 – 1 est positif sur ]– ; – 1], négatif sur [– 1 ; 1] et positif sur [1 ; + [ d’où le signe donné pour f ’ (x) sur [0,5 ; 6].2.

x 0,5 1 6

f’(x) – 0 +

f2,5

2

376

3. Le minimum de f est égal à 2.26 1. Le coefficient directeur de la tangente à la courbe au

point d’abscisse 0 est égal à 2.2. T : y = 2x.27 Voir page 252 du manuel.28 1.

10

A

T

y

x

1 1m = – 1

2. a. f ’ (x) = – 1x2 et f ’(1) = –1.

b. f ’(1) est le coefficient directeur de la tangente à la courbe au point d’abscisse 1.c. Voir le graphique ci-dessus.29 1.

10

y

x

1

T

2. a. T : y = 0,5x.b. Voir le graphique ci-dessus.30 1. le coefficient directeur de la tangente à la courbe au

point d’abscisse 2 est égal à – 2.2. f ’(2)= – 2.

P O U R S ’ E N T R A î N E R31 1. a. f ’ (x) = – 1

x2.

b. f ’(3) = – 19

.

c. Le nombre dérivé de la fonction f en 4 est égal à – 116

.

2. g ’ (x) = 4 + 1x2 et h’ (x) = 9 – 1

x2.

32 1. u’(x) = 2x, v’(x) = –3 et w’ (x) = – 1x2

.

2. f ’ (x) = 2x – 3 et g ’ (x) = 2x – 3 – 1x2

.

33 f ’ (q) = 63 – 1q2 et g’(q)= 4q – 1

q2.

34 1. a. f ’ (x) = – 1x2

.

b. f ’(2) = – 14

.

c. Le nombre dérivé de la fonction f en – 3 est égal à – 19

.

2. u’(x) × v(x) = 4(x + 1) = 4x + 4.3. u(x) × v’(x) = 4x.10 Voir page 252 du manuel.11 1. u(x) =2x et v(x) = x – 1,

u’(x) = 2 et v’(x) = 1.2. u’(x) v(x) = 2(x – 1) = 2x – 2,u(x) v’(x) = 2x.3. u’(x) v(x) – u(x) v’(x) = –2.12 1. u(x) = x + 3 et v(x) = x + 1, u’(x) = 1 et v’(x) = 1.

2. u’(x) v(x) = 1(x + 1) = x + 1,u(x) v’(x) = x + 3.3. u’(x) v(x) – u(x) v’(x) = – 2.13 1. f (x) est un quotient.

2. La formule de la dérivée d’un quotient.

3. f ’ (x) = –12x – 4( )2

.

14 1. f (x) est un quotient.2. La formule de la dérivée d’un quotient.

3. f ’ (x) = –12x – 4( )2

.

15 Voir page 252 du manuel.

16 f ’ (x) = –30 – 142x – 6( )2

= –442x – 6( )2

.

17 f ’ (x) = 62 – x( )2

.

18 La fonction f est croissante sur ]– ; – 4].19 La fonction f est décroissante sur ]3 ; + [.20

x – 3 1 4

f’(x) + 0 –

f– 2

3

– 3

21 1. f ’ (x) est positif comme quotient de deux termes positifs.2. La fonction f est croissante sur [1 ; 7].22 1. f ’ (x) est négatif comme quotient d’un terme négatif

par un terme positif.2. La fonction f est décroissante sur [– 1 ; 3].23 Voir page 252 du manuel.24 1. a. La fonction f est décroissante sur [– 4 ; 3].

b. f (– 4) = 0,5 et f (3) = – 3.c.

x – 4 3

f’(x) –

f0,5

– 3

2. a. f ’ (x) = –4x – 4( )2

.

b. f ’ (x) est négatif comme quotient d’un terme négatif par un terme positif donc la fonction f est décroissante sur [– 4 ; 3].25 1. a. f ’ (x) est un quotient dont le dénominateur est

toujours strictement positif donc le signe de f ’ (x) est celui de x 2 – 1.

33Chapitre 4 Fonctions rationnelles

On a f ’ (0) = – 7.46 Voir page 252 du manuel.47 1. f ’ (x) = g ’ (x) = –3

(x – 2)2.

2. a. La proposition est vraie.b. « Si deux fonctions ont leurs dérivées égales, alors ces deux fonctions sont égales ». c. Cette proposition est fausse ; les fonctions données en 1.fournissent un contre-exemple.48 1. u(x) = 2 et v(x) = 3x + 1 donc u’ (x) = 0 et v’ (x) = 3.

f ’ (x) = –6(3x +1)2

.

2. u(x) = 50 et v(x) = 4x – 1 donc u’ (x) = 0 et v’ (x) = 4.

f ’ (x) = –50(4x – 1)2

.

49 Exercice résolu.

50 1. f ’ (x) = 3(x2 + 2) – 6x2

(x2 + 2)2 = 6 – 3x2

(x2 + 2)2.

2. f ’ (x) = –100x(x2 +1)2

.

51 1. f ’ (x) = –3x2 – 2x – 3(x2 – 1)2

.

2. g ’ (x) = x2 – 6x – 5(x – 3)2

.

52 Vrai : f ’ (x) = g ’ (x) = 10(x + 7)2

.

53 Faux : f ’ (q) = 12 – 3q2

(q2 + 4)2.

54 Vrai : f ’ (x) = 4 et g ’ (x) = 4(x +1)2

donc f ’ (x) – g ’ (x) = 4 – 4(x +1)2 = 4x2 + 8x

(x +1)2.

55 1. f ’ (x) = 1 – 9x2 = x2 – 9

x2.

2. a. Tableaux de signes :

x – – 3 3 +

x 2 – 9 + 0 – 0 +

x 1 3 6

x 2 – 9 – 0 +

b. La fonction f est décroissante sur [1 ; 3] et croissante sur [3 ; 6].

56 1. f ’ (x) = 1(x – 3)2

.

2. f ’ (x) est strictement positif sur [– 5 ; 1].3. f est croissante sur [– 5 ; 1].

57 1. a. f ’ (x) = – kx2

.

b. Dans le cas où k = – 3, f ’ (x) = 3x2 donc f est croissante sur

]0 ; + [.2. a. Algorithme

Variable k est un réelEntrée Saisir kTraitement

Si k 0Alors afficher « f est croissante »Sinon

Si k 0 Alors afficher « f est décroissante » Sinon Afficher « f est constante »

Fin Si Fin Si

2. g ’ (x) = –150x2 et h ’ (x) = 75

x2.

35 1. f (x) = 13× 1x donc k = 1

3 et u(x)= 1

x.

2. f ’ (x) = – 13× 1x2 = – 1

3x2.

3. f ’ (1) = – 13 et f ’ (2) = – 1

12.

36 1. f (x) = 27× 1x donc k = 2

7 et u(x)= 1

x.

2. f ’ (x) = − 27× 1x2 = – 2

7x2.

3. f ’ (–1) = − 27

et f ’ (– 4) = − 156

.

37 Voir page 252 du manuel.

38 1. f ’ (x) = 75 – 5x2

.

2. f ’ (x) = 5 – 34x2

.

3. f ’ (x) = 4 + 2x2

.

39 1. g ’ (x)= – 11x2

.

2. a. La proposition est vraie.

b. « Si f ’ (x) = – 11x2 , alors f (x) = 11

x ».

c. Cette proposition est fausse puisque l’on pourrait avoir, par

exemple, f (x) = 11x

+ 3.

40 Fichiers associés sur www.bordas-indice.fr et sur le CD-Rom Professeur :04_TSTMG_exercice40.xlsx (Excel 2007),04_TSTMG_exercice40.xls (Excel 2003),04_TSTMG_exercice40.ods (OpenOffice).1. a. =14*A2-9/A2 .

b. f (x) = 14x – 9x

.

c. f (2) = 23,5 ; f (3) = 39. Ces résultats sont conformes à ceux affichés par le tableur.

2. a. f ’ (x) = 14 + 9x2

.

b. Dans la cellule C2, il faut saisir la formule =14 +9/A2^2

pour obtenir, par recopie vers le bas, les images des réels xpar la fonction f ’.41 1. u(x) = 3x 2 – x + 2 et k = 5 ; u’ (x) = 6x – 1 et v’ (x) = − 5

x2  ;

donc f ’ (x) = 6x – 1 – 5x2

.

2. u(x) = x 3 + x 2 + x et k = – 2 ; u’ (x) = 3x 2 + 2x + 1 et v’ (x) = 2x2

donc f ’ (x) = 3x 2 + 2x + 1 + 2x2

.

3. u(x) = x 4 + 3x et k = 100 ; u’ (x) = 4x 3 + 3 et v’ (x) = −100x2 donc

f ’ (x) = 4x 3 + 3 – 100x2

.

42 Exercice résolu.

43 f (x) = x – 2 + 9x donc u(x) = x – 2 et k = 9, f ’ (x) = 1 – 9

x2.

44 1. f ’ (x) = 12(3x + 6)2

.

2. f ’ (x) = 4(3x – 1)2

.

45 1. f ’ (x) = –7(2x – 1)2

.

On a f ’ (0) = – 7.

2. f ’ (x) = –7(x +1)2

.

34

1. f ’ (x) = 2x3 + 6x2

(2 + x )2 = 2x2 (x + 3)(2 + x )2

.

La fonction f est croissante sur [0 ; + [.2. a. Pour le réel k affiché par l’algorithme, f (k) est supérieur ou égal à 100.b. k = 11 car f (10) ≈ 83,33 et f (11) ≈ 102,38.3. Programme pour CASIO0 → k0 → FWhile F100k+1 → kk^3/(2+k) → FWhileEndk64 Vrai : f ’ (x) = 1– 1

x2 = x2 −1x2 ; f ’ (x) est positif sur ]1 ; + [

donc f est croissante sur ]1 ; + [.

65 Vrai : f ’ (x) = –4(2 – x )2

.

66 1. f ’ (x) = 4x(x2 +1)2

.

La fonction f est décroissante sur ]– ; 0] et croissante sur [0 ; +[.2. La fonction f admet un minimum en x = 0, égal à – 1.67 1. Allure de la courbe

10

y

x

100

2. a. C’M(x) = 2 – 2 000 000x2 = 2(x – 1 000)(x +1 000)

x2.

b. Sur [200 ; 1 000], la fonction C’M est négative et sur [1 000 ; 4 000] cette fonction C’M est positive.c.

x 200 1 000 4 000

C’M(x) – 0 +

CM

11 900

5 500

10 000

d. Le coût moyen minimal est obtenu au bout de 1 000 jours d’utilisation ; ce coût moyen journalier est égal à 5 500 euros. 68 Voir page 252 du manuel.69 Fichiers associés sur www.bordas-indice.fr et sur le

CD-Rom Professeur :04_TSTMG_exercice69.xlsx (Excel 2007),04_TSTMG_exercice69.xls (Excel 2003),04_TSTMG_exercice69.ods (OpenOffice).

1 a. c ’ (x) = 0,5 – 8x2 = 0,5x2 – 8

x2.

b. 0,5x 2 – 8 s’annule sur [0,1 ; 10] pour x = 4 ; cette expression est négative sur [0,1 ; 4] et positive sur [4 ; 10].

b. Lorsque l’on saisit 13 comme valeur de k, l’affichage est « f est décroissante ».c. Lorsque l’on saisit 0 comme valeur de k, l’affichage est « f est constante ».

58 1. f ’ (x) = 100(x +1)2 .

2. f ’ (x) est strictement positif sur [0 ; 5] donc f est croissante sur [0 ; 5].

x 0 5

f’(x) +

f0

2503

3.

10

y

x

1

4. Par lecture graphique, on trouve que la machine est rentable pour x appartenant à [4 ; 5].59 Voir page 252 du manuel.

60 1. f ’ (x) = 1– 1(x – 1)2 = (x – 1)2 – 1

(x −1)2 = x2 – 2x(x −1)2

.

2. x 2 – 2x = x(x – 2).Sur [2 ; 101], x et x – 2 sont positifs donc le produit x(x – 2) est positif .3.

x 2 101

f’(x) +

f2

100,01

61 Exercice résolu.62 1. D = (– 4)2 + 20 = 36 ; x1 = – 1 et x2 = 5.

x – – 1 5 +

x 2 – 4x – 5 + 0 – 0 +

g(x) est négatif sur [3 ; 5] et positif sur [5 ; 10].

2. a. f ’ (x) = (2x + 3)(x – 1) – (x2 + 3x – 1)(x – 2)2

donc f ’ (x) = x2 − 4x – 5(x – 2)2 = g(x )

(x – 2)2.

b. f ’ (x) est négatif sur [3 ; 5] et positif sur [5 ; 10].c.

x 3 5 10

f’(x) – 0 +

f17

13

1298

63 Fichier associé sur www.bordas-indice.fr et sur le CD-Rom Professeur :04_TSTMG_exercice63.alg (AlgoBox).

35Chapitre 4 Fonctions rationnelles

b. Le maximum de la fonction f est égal à – 3.

4.

– 1

0

y

x

– 1

A B

80 Faux : le coefficient directeur de la tangente à la courbe 

au point d’abscisse 1 est égal à 3.81 Vrai : f ’ (0)= 1 et f (0) = – 2.

82 1. f ’ (x) = 4x(x – 3) – 2x2

(x – 3)2 = 2x2 – 12x(x – 3)2

.

2. f ’ (x) est négatif sur [4 ; 6] et positif sur [6 ; 10].

3.

x 4 6 10

f’(x) – 0 +

f32

24

2007

83 1. f ’ (x) = –2(x – 3)2

.

2. T : y = – 225

x – 3925

.

84 1. f est décroissante sur ]0 ; 2] et croissante sur [2 ; + [.

2. f admet un minimum pour x = 2.

85 1. f (x) = x3 + x2 + 4

x2

donc f ’ (x) = (3x2 + 2x )x2 – 2x(x3 + x2 + 4)x4 = x3 – 8

x3.

2. a. x 2 + 2x + 4 est positif sur ]0 ; + [.

b. f est décroissante sur ]0 ; 2] et croissante sur [2 ; + [.

P O U R FA I R E L E P O I N TVoir page 252 du manuel. Les corrigés détaillés sont disponibles sur le site www.bordas-indice.fr.

ACCO M PAG N E M E N T P E R S O N N A L I S é86 1. f ’ (x) = 2x(x – 4) – x2

(x – 4)2 = x2 – 8x(x – 4)2

.

2. f ’ (x) = 10x(x – 3) – 5x2

(x – 3)2 = 5x2 – 30x(x – 3)2

.

3. f ’ (x) = 2x(x2 + 2) – 2x(x2 – 1)(x2 + 2)2 = 6x

(x2 + 2)2.

4. f ’ (x) = 1+ 3x2

.

c. La fonction c est décroissante sur [0,1 ; 4] et croissante sur [4 ; 10].2. a. La formule est =A2+0,1 .

b. La formule est =0,5*A2+8/A2 .

c. Dans la colonne D, on a calculé la différence p(x) – c(x). Lorsque l’entreprise produit et vend 0,4 hectolitre, elle fait un déficit moyen de 7,52 milliers d’euros environ par hectolitre.3. L’entreprise est bénéficiaire lorsque x est compris entre 0,7 et 9,3 hectolitres.

70 Faux : f ’ (x) = 2 – 2x2 = 2x2 – 2

x2 ;

f ’ (x) est positif sur ]- ; – 1] et négatif sur [– 1 ; 0[ donc la fonction admet un maximum en – 1 ; ce maximum est égal à – 3.

71 Vrai : g ’ (x) = 4x(x2 + 5)2 donc g ’ (x) est négatif sur ]– ; 0]

et positif sur [0 ; + [.La fonction g admet un minimum en x = 0.

72 1. f ’ (x) = 8(3x +1)2

.

2. a. Le coefficient directeur de la tangente à la courbe au point d’abscisse 1 est égal à f ’ (1) = 0,5.Une équation de la tangente T est y = 0,5x + 1,5.b.

10

y

x

1

A

T

73 1. a. Le coefficient directeur de T est 3.b. f ’ (0) = 3.c. T : y = 3x.2. f ’ (1) = 0.3. a. f ’ (x) = 3 – 3x2

(x2 +1)2.

b. f ’ (0) = 3 et f ’ (1) = 0.c. On retrouve les mêmes résultats que ceux obtenus par lecture graphique.74 f ’ (x) = 10 – 1

x2 donc f ’ (1)= 9.T : y = 9x + 2.75 Voir manuel page 252.

76 f ’ (x) = 7(x +1)2 donc f ’ (3)= 7

16.

T : y = 716

x – 116

.

77 Exercice résolu.

78 1. f ’ (x) = –x2 + 2xx4 = –x + 2

x3.

2. a. f ’ (x) = 0 équivaut à x = 2.b. La tangente à C est horizontale au point de coordonnées (2 ; 0,25).79 1. a. La courbe admet une tangente de coefficient

directeur égal à 0 au point d’abscisse – 2.b. La tangente est horizontale.2. Le coefficient directeur de la tangente à la courbe au point d’abscisse – 3 est égal à 0,75.3. a. La fonction f est croissante sur [– 4 ; – 2] et décroissante sur [– 2 ; – 1,5].

36

P O U R A P P R O F O N D I R88 1. a. f ’ (x) = 1

4– 4x2 = x2 – 16

4x2.

b.

x 1 4 20

f’(x) – 0 +

f5,25

3

6,2

2. a. U(q)= C(q)q

=4 + q + q2

4q

= 4q+1+ q

4.

b. U(q) = f (q) donc le coût moyen d’un millier de tonnes de produit lorsque q tonnes sont produites est minimal pour q = 4 .

89 1. a. C’(v) = 0,12v × v – (0,06v2 +150)v2 = 0,06v2 – 150

v2.

b. C est décroissante sur [20 ; 50] et croissante sur [50 ; 130].2. a. 50 kilomètres par heure.b. La consommation minimale est égale à C (50), soit 6 litres.90 1. En décembre 2013, le prix de la tablette est égal à

275 euros.On cherche y tel que 275 1–

y100( ) = 220.

Donc 1–y

100 = 0,8 donc y = 20.

2. a. On a P 1+ x100( ) 1–

y100( ) =P donc x et y sont tels que

1+ x100( ) 1–

y100( ) = 1.

b. 1–y

100= 1

1+ x100

donc 1– 100100 + x

= y100

donc y = 100xx +100

.

3. a. f ’ (x) = 100(x +100) – 100xx +100( )2

= 10 000x +100( )2

.

b.

x 0 100

f’(x) +

f0

50

4. On cherche x tel que 10 = 100xx +100 ; on trouve x = 100

9. Il faut

donc augmenter un produit d’environ 11,11 % pour que le prix de cet article redevienne lui-même après une diminution de 10 % du nouveau prix.91 1. On a x 2h = 13 500 donc h = 13 500

x2.

2. A(x) = 4xh + x 2 = 4x × 13 500x2 + x2 = 54 000

x + x2 .

3. A’(x) = – 54 000x2 + 2x = 2x3 – 54 000

x2.

Or 2(x – 30)(x 2 + 30x + 900) = 2x3 – 54 000 donc on retrouve bien l’expression de A’(x) donnée.4. L’aire est minimale lorsque x = 30 cm et h = 15 cm.92 1. Quand l’abonnement coûte 50 euros, on a 40 milliers

d’abonnements.2. a. Le nouveau prix est égal à 50,50 euros, on a alors 39 milliers d’abonnements.

5. f ’ (x) = 1+ 1(x +1)2

.

87 1. Le signe de f ’ (x) est celui de x(x – 8), c’est-à-dire celui de x – 8 sur [1 ; 2] ; f ’ (x) est négatif sur [1 ; 2] donc f est décroissante sur [1 ; 2].2. Le signe de f ’ (x) est celui de 5x(x – 6), c’est-à-dire celui de x – 6 sur [1 ; 2] ; f ’ (x) est négatif sur [1 ; 2] donc f est décroissante sur [1 ; 2].3. Le signe de f ’ (x) est celui de 6x, c’est-à-dire positif sur [1 ; 2] ; donc f est croissante sur [1 ; 2].4. f ’ (x) est positif sur [1 ; 2] donc f est croissante sur [1 ; 2].5. f ’ (x) est positif sur [1 ; 2] donc f est croissante sur [1 ; 2].

T R AVAU X P R AT I Q U E STP Coût total, coût moyen et coût marginal

L’objectif de ce TP est d’utiliser le tableur afin de déterminer la quantité à produire pour que le coût moyen soit minimal et de découvrir que le coût moyen est minimal lorsque le coût margi-nal est égal au coût moyen.Fichiers associés sur le site www.bordas-indice.fr et sur le CD-Rom Professeur :04_TSTMG_TP_correction.xlsx (Excel 2007)04_TSTMG_TP_correction.xls (Excel 2003)04_TSTMG_TP_correction.ods (OpenOffice)A. Étude du coût moyen1. et 2. Dans la cellule A3, on entre la formule =A2+1 .3. Dans la cellule B2, on entre la formule

=0.3*A2^3-4*A2^2+30*A2+18 .

4. a. Si x = 2, alors CM(2) = C(2)2 = 32,2.

b. Dans la cellule C3, on entre la formule =B3/A3 .5. a. Le coût moyen semble minimal pour x = 7.b. Une valeur approchée du coût minimal est 19 euros.6. En utilisant les indications placées dans l’aide du livre, on obtient le graphique donné dans le livre élèves.B. Étude du coût marginal1. C’(x) = 0,9x2 – 8x + 30.2. Cm(2) ≈ C’(2) donc Cm(2) ≈ 17,6.3. Dans la cellule D2, on entre la formule =0.9*A2^2-8*A2+30 .4. En sélectionnant la colonne D, on obtient un graphique proche de celui donné dans le livre élève.5. Les courbes représentatives du coût moyen et du coût marginal se coupent en un point d’abscisse environ égale à 7,6 ; une valeur approchée entière de cette abscisse peut être 8.C. Cas général

1. C’M(x) = C ’(x ) × x – C(x )x2

.

2. C’M(x) = 0 équivaut à C’(x) × x – C(x) = 0 équivaut à

C’(x) = C(x )x

, c’est-à-dire Cm(x) = CM(x).

3. Le coût moyen est minimal lorsque le coût moyen est égal au coût marginal.

37Chapitre 4 Fonctions rationnelles

B. Étude de la fonction recette1. R(75) = 5 640 et R(100) = 7 520 donc la droite passe par les points donnés et sa représentation graphique est tracée sur la figure de la question A. 4.2. Par résolution graphique de l’inéquation R(x) C(x), on trouve que l’entreprise est déficitaire lorsque x est compris entre 75 produits et 80 produits.C. Étude de la fonction coût moyen

1. f (x) = x2 – 120x + 9 216

x = x – 120 + 9 216x .

2. f ’ (x) = 1– 9 216x2 = x2 – 9 216

x2 = x – 96( ) x + 96( )x2

car 962 = 9 216.3. a. f ’ (x) est négatif sur [75 ; 96], et positif sur [96 ; 100].b.

x 75 96 100

f’(x) – 0 +

f77,88

72

72,16

c. Le coût moyen est minimal lors de la production de 96 articles ; ce coût minimal est de 72 euros par article.

Sujet CFichiers associés sur le site www.bordas-indice.fr et sur le CD-Rom Professeur :04_TSTMG_sujetC.xlsx (Excel 2007),04_TSTMG_sujetC.xls (Excel 2003),04_TSTMG_sujetC.ods (OpenOffice).1. a. La cellule F2 contient la formule =F1-$B1 .b. La formule saisie dans la cellule B3 est=(45*B2+15)/(3*B2+20) .

Le résultat affiché dans la cellule F3 est 6,094.

2. a. f ’ (x) = 8553x + 20( )2

.

b. La fonction f est croissante sur [0 ; 10].3. Avec l’éditeur de fonctions, on trouve que f(8) ≈ 8,523 et f (9) ≈ 8,936.En imaginant que la vente de cette boisson se poursuive ainsi, c’est à partir de l’année 2017 que la quantité de boissons vendues sera supérieure à 8,8 millions de bouteilles.

Sujet D

1. f ’ (x) = 80x2 – 1(40x2 +1 000)x2 = 40x2 – 1 000

x2.

2. Le signe de f ’ (x) sur [2 ; 10] est celui de 40x 2 – 1 000, c’est-à-dire celui de x – 5 car 40x 2 – 1 000 = 40(x – 5)(x + 5).Donc f ’ (x) est négatif sur [2 ; 5] et positif sur [5 ; 10].

x 2 5 10

f’(x) – 0 +

f580

400

500

3. Le coût de production est minimal pour cinq séjours ; ce coût minimal est égal à 400 euros.

b. Lorsque le prix initialement fixé à 50 euros augmente de 1 %, le nombre d’abonnements demandé baisse de 2,5 %.

3. a. f ’ (p) = –2 et E(p) = –2p140 – 2p

.

b. E(50)= – 2,5. Ainsi, lorsque le prix augmente de 1%, la quantité demandée baisse de 2,5 %, on retrouve le résultat de la question 2.b.. Cet abonnement est un bien élastique : sa demande dépend fortement du prix .

c. E’(p) = –280140 – 2p( )2

. La fonction E est décroissante.

C A P V E R S L E B AC

Zoom sur le tableur

Fichiers associés sur le site www.bordas-indice.fr et sur le CD-Rom Professeur :04_TSTMG_zoomtableur.xlsx (Excel 2007),04_TSTMG_ zoomtableur.xls (Excel 2003),04_TSTMG_ zoomtableur.ods (OpenOffice).

Sujet APartie A1. La réponse b est vraie car f (3) = 5

4.

2. La réponse c est vraie car f (–2) = 0.3. La réponse a est vraie car f ’ (x) = –1

x +1( )2.

Partie B1. La réponse c est vraie.2. La réponse b est vraie.3. La réponse a est vraie car, par lecture graphique, une équation de T est y = – 2x + 2.

Sujet BA. Étude de la fonction coût1. C’(x) = 2x – 120.2. C’(x) est positif sur [75 ; 100] .3.

x 75 100

C’(x) +

C5 841

7 216

4. y

x75 80 85 90 95 100

�M

38

b. R(x)= 110x.c. Voir graphique ci-dessus.d. 4 séjours.

93 1. f ’ (x) = 3(x2 +1) – 3x × 2xx2 +1( )2

= 3 – 3x2

x2 +1( )2.

f ’ (x) est négatif sur ]– ; – 1], positif sur [– 1 ; 1] et négatif sur [1 ; + [.2. La fonction f est décroissante sur ]– ; – 1], croissante sur [– 1 ; 1] et décroissante sur [1 ; + [.3. Une équation de la tangente à la courbe au point d’abscisse 0 est y = 3x.94 1. f ’ (x) = 9 – 1

x2 = 9x2 – 1x2

.

f est décroissante sur 0,1 ; 13

⎡⎣

⎤⎦ et croissante sur 1

3; 5⎡

⎣⎤⎦ .

2. f admet un minimum en 13 égal à 7.

3. Le coefficient directeur de la tangente à la courbe au point d’abscisse 1 est égal à f ’ (1), soit 8.

4. Courbe représentative

20

�f

(d)

y

x

100

5. a. • Le bénéfice net pour la vente de 3 séjours est égal à 330 – C(3) soit une perte d’environ 123,33 euros.• Le bénéfice net pour la vente de 7 séjours est égal à 770 – C(7) soit un bénéfice d’environ 346,14 euros.

39Chapitre 5 Statistiques

A Le programme

5CHAPITRE

Statistiques

Objectifs :• Consolider les acquis de la classe de Première sur la statistique à une variable.• Découvrir quelques notions sur la statistique à deux variables et la problématique de l'ajustement.

Contenus Capacités attendues Commentaires

Statistique descriptive à deux variablesÉtude de séries de données statistiques quantitatives à deux variables.Nuage de points.

• Représenter graphiquement un nuage de points associé à une série statistique à deux variables.

On accompagne ce travail d’un entretien des capacités sur les statistiques à une variable de la classe de Première.

Ajustement affine. • Trouver une fonction affine qui exprime de façon approchée y en fonction de x.• Utiliser un ajustement affine pour interpoler ou extrapoler.

L’ajustement affine est réalisé graphiquement ou par la méthode des moindres carrés à l’aide de la calculatrice ou du tableur.Aucun développement théorique n’est attendu. D’autres types d’ajustement peuvent être rencontrés dans des exemples.

Ce chapitre est scindé en deux parties distinctes :• la première reprend les notions statistiques vues dans les classes antérieures, c’est-à-dire la médiane, les quartiles, la moyenne et l’écart-type. Il s’agit, conformément au programme, de consolider les acquis de la classe de Première. Pour cela deux activités permettent de réintroduire ces notions.Dans l’activité 1, nous revoyons les notions de médiane et de quartiles en leur donnant une signification concrète.Dans l’activité 2, l’utilisation de la calculatrice permet de revoir la notion d’écart-type et son utilisation.Quatre savoir-faire sont associés à cette partie, deux sont consacrés à l’obtention des différents paramètres, et les deux autres à l’utilisation de ces paramètres statistiques ;• la seconde partie est nouvelle pour les élèves. Il s’agit de découvrir les notions de série statistique à deux variables et d’ajustement affine. L’activité 3 permet aux élèves de découvrir les notions de série statistiques à deux variables et de nuage de points associé.L’activité 4 a pour but d’introduire la notion de droite de régression par la méthode des moindres carrés. Aucune théorie sur cette droite n’étant bien sûr au programme, il nous a cependant semblé intéressant d’en faire observer les particularités. Dans cette activité, l’élève est essentiellement spectateur d’une activité menée par l’enseignant et qui permettra finalement de donner du sens à cette droite de régression.Deux pages sont consacrées spécifiquement à l’utilisation des TICE, les élèves pourront ainsi s’y reporter pour obtenir, à l’aide de la calculatrice ou du tableur, la moyenne et l’écart-type d’une série statistique simple ou pour obtenir l’équation réduite de la droite de régression par la méthode des moindres carrés d’une série statistique à deux variables.La page d’accompagnement personnalisé est consacrée à deux points essentiels du chapitre : la représentation du nuage de points d’une série statistique à deux variables, et l’obtention d’une équation de la droite de régression par la méthode des moindres carrés à la calculatrice.Le TP est également consacré à la régression linéaire par la méthode des moindres carrés, puisque nous établissons, à l’aide d’un ajustement, une relation entre la distance d’arrêt d’un véhicule et la vitesse de celui-ci.

B Notre point de vue

40

Enfin, les pages Cap vers le bac reprennent l’ensemble des notions rencontrées dans ce chapitre, nous avons en particulier intégré des exercices portant sur les statistiques à une variable pouvant faire l’objet de questions à l’examen.

Les notions abordées dans le chapitre 5 • Statistiques à une variable• Statistiques à deux variables• Ajustement affine

C Avant de commencerVoir manuel pages 252 et 253 et le site www.bordas-indice.fr pour les corrigés détaillés.

D ActivitésActivité Les vacances au soleil1

Fichier associé sur le site www.bordas-indice.fr et sur le CD-Rom Professeur :05_TSTMG_activite1.ggb (GeoGebra).Cette activité permet de revoir les notions de médiane, de quartiles et de diagramme en boite d’une série statistique à une variable. Les élèves sont amenés à déterminer ces paramètres pour une série, puis à en donner une interprétation.1.

Destination Djerba Malte Corse Croatie Grèce Martinique BrésilPrix du billet 280 380 670 770 790 1 050 1 490Nombre de billets vendus

105 150 250 100 120 130 145

Effectifs cumulés croissants

105 255 505 605 725 855 1 000

2. 1 000 est pair et 1 000 ÷ 2 = 500 donc la médiane est la demi-somme de la 500e et de la 501e valeur, ces deux valeurs étant égales à 670, la médiane de la série est 670.3. a. 1 000 ÷ 4 = 250 donc Q1 est la 250e valeur : Q1 = 380.b. 1 000 × 0,75 = 750 donc Q3 est la 750e valeur : Q3 = 1 050.c. Q3 – Q1 = 670.4.

300200 400 500 600 700 800 900 1 000 1 100 1 200 1 300 1 400 1 500

5. a. Vrai car Q1 = 380.b. Vrai car la médiane est 670.c. Vrai car comme Q3 = 1 050, au moins 75 % des billets coûtent moins de 1 050 euros, soit moins de 25 % coûtent plus de 1 050 euros.

Activité Contrôle qualité2Cette activité permet de revoir la notion d’écart-type. Pour percevoir l’intérêt de ce paramètre, nous déterminons la médiane, et les quartiles de deux séries, pour constater que ces paramètres ne suffisent pas à comparer les deux séries. L’écart-type est obtenu à la calculatrice conformément au programme.

1. Sous-traitant 1 : Q1 = 29,9 ; Me = 30 et Q3 = 30,2.Sous-traitant 2 : Q1 = 29,9 ; Me = 30 et Q3 = 30,2.2. Non, car les paramètres sont identiques pour les deux séries.3. Les valeurs du sous-traitant 1 semblent être plus proches du diamètre théorique, tandis que pour le sous-traitant 2 les valeurs semblent être plus dispersées.4. a.

Texas Casiob. Il s’agit de l’écart-type : pour le sous-traitant 1, σ1 ≈ 0,187 et pour le sous-traitant 2, σ2 ≈ 0,237. Cela confirme que le sous-traitant 1 a une production plus régulière.

Activité Plus haut, plus froid3Cette activité permet de découvrir les notions de nuage de points, de point moyen d’un nuage et d’ajustement affine d’une série statistique à deux variables.1. x1 = 0,4 et y1 = 8,5.2. a.

0,20

y

x

1

M1

M2

M3

M4

M5

M6

G

A

B

41Chapitre 5 Statistiques

Cette activité est une activité d’observation qui vise à introduire la notion de droite de régression par la méthode des moindres carrés en lui donnant un minimum de sens. L’élève pourra notamment observer que cette droite passe par le point moyen du nuage.

1. Le coefficient directeur d’une droite approchant de façon raisonnable le nuage de point est négatif.

2. E = 23,38 et la droite passe par le point G.

3. • Pour a = – 9, on obtient 11,94 pour valeur minimale de E et la droite passe par le point G.• Pour a = – 8, la valeur minimale de E est 4,42 et la droite passe par le point G.

4. La valeur minimale de E est 0,47.

5. y = –6,6x + 11,33.

b. M2(0,8 ; 6,5) correspond au relevé n° 2.c. Voir graphique obtenu en 2. a.d. Le nuage a une forme « allongée ».3. Non les autres points ne sont pas très éloignés de la droite (M1M6).4. a. x = 1,3 et y = 2,75.b. Voir graphique obtenu en 2. a.c. Oui.

Activité La méthode des moindres carrés4Fichier associé sur www.bordas-indice.fr et sur le CD-Rom Professeur :05_TSTMG_activite4.ggb (GeoGebra).

E Exercices8

9 x = 11,75 et σX ≈ 5,74 à 0,01 près.10 Voir manuel page 252.11 Comme σ2 � σ1 la classe de TSTMG1 a des résultats plus

hétérogènes.12 1. 8 tonnes.

2. 22 mètres.3. La charge maximale que peut lever la grue si la flèche mesure 28 mètres est 5 tonnes.13 1.

Année 2000 2002 2004 2006 2008 2010 2012

Rang de l’année : xi 0 2 4 6 8 10 12

Effectif : yi 144 147 152 156 158 160 161

2. x = 6 et y = 154 donc G(6 ; 154).3. M(13 ; 163).

14

10

y

x

10

15 L’ajustement affine est envisageable pour les nuages a.et d.

P O U R D É M A R R E R1 a. Q1 = 5 ; Me = 17 ; Q3 = 38.

b. Q1 = 24 ; Me = 55 ; Q3 = 62.2 Voir manuel page 252.3 1. 105 ÷ 2 = 52,5 donc la médiane est la 53e valeur.

2. 105 ÷ 4 = 26,25 donc le premier quartile est la 27e valeur.

3. 105 × 0,75 = 78,75 donc le troisième quartile est la 79e valeur.4 1.

Taille en cm 170 171 172 173 174 175 176 177 178 179 180 181 183

Effectif 3 2 5 4 3 3 1 4 2 1 3 1 1

Effectif cumulé 3 5 10 14 17 20 21 25 27 28 31 32 33

2. 33 ÷ 2 = 16,5 donc la médiane est la 17e valeur, c’est-à-dire

174.

3. a. 33 ÷ 4 = 8,25 donc le premier quartile est la 9e valeur.

b. Q1 = 172.

4. 33 × 0,75 = 24,75 donc Q3 est la 25e valeur soit 177.5 1. La médiane est 11, le premier quartile est 7, le troisième

quartile est 12.

2. Q3 – Q1 = 12 – 7 = 5.

3. 25 %.6

5 6 7 8 9 10 11 12 13 14 15 16 17 18 19

7

42

2. Fichier associé sur www.bordas-indice.fr et sur le CD-Rom Professeur :05_TSTMG_exercice26 (AlgoBox).

27 Exercice résolu.28 Faux : pour le lot A, Q3 = 40 donc c’est 75 % des tulipes du

lot A dont la masse est inférieure à 40 grammes.29 Vrai : pour le lot A, le maximum est 60 et pour le lot B le

troisième quartile est égal à 70.30 1. x = 3,3.

2. σx ≈ 1,9.31 Fichiers associés sur le site www.bordas-indice.fr et sur

le CD-Rom Professeur :05_TSTMG_exercice31.xls (Excel 2003),05_TSTMG_exercice31.xlsx (Excel 2007),05_TSTMG_exercice31.ods (OpenOffice).1. Dans la cellule K1 : =MOYENNE(A1:H2) .

2. Dans la cellule K2 : =ECARTYPEP(A1:H2) .32 1. Salaire moyen PME A : 2,3 milliers d’euros.

Salaire moyen PME B : 2,3 milliers d’euros.2. • Écart-type PME A : σA ≈ 1,65.• Écart-type PME B : σB ≈ 1.3. Les salaires moyens sont égaux mais l’écart-type de la série des salaires de la PME A est nettement supérieur à celui de la série des salaires de la PME B : il y a une répartition des salaires plus hétérogène pour la PME A.33 Faux : l’écart-type est toujours un nombre positif.34 Faux : par exemple si la série est « 2-8 », la moyenne est 5,

or si on double la première valeur on obtient la série « 4-8 » qui a pour moyenne 6.35

10

y

x

1

16 1. 4 000 visiteurs.2. C’est la neuvième année.17 Voir manuel page 253.18 1. y = –1,3 × 15 + 22,9 = 3,4.

2. 3 400 produits vendus par mois.19 1.

2. 1,8x – 0,7 = 24,5 ⇔ x = 14.20 Voir manuel page 253.

P O U R S ’ E N T R A Î N E R21 1. Me = 6 ; Q1 = 3 et Q3 = 7.

2.

1 2 3 4 5 6 7 8 9

3. Oui car Q1 = 3.22 1.

Pression artérielle 12 13 13,5 14 14,5 15 16 17 18

Effectifs 2 4 2 7 6 5 1 1 2

Effectifs cumulés 2 6 8 15 21 26 27 28 30

2. Me = 14,25 ; Q1 = 13,5 et Q3 = 15.3.

12,0 12,5 13,0 13,5 14,0 14,5 15,0 15,5 16,0 16,5 17,0 17,5 18,0

4. C’est parce que Q3 = 15.23 Voir manuel page 253.24 1. Oui car Q1 = 9 et Q3 = 13.

2. Les médianes sont égales mais l’écart interquartile est beaucoup plus important pour la promotion 2012. Cette promotion a un niveau plus hétérogène que la promotion 2013.25 1. Environ 50 %

2. Non, pour les deux intervalles il y a environ 25 % de l’effectif.3. Les saumons capturés sont probablement issus d’un élevage : leurs tailles sont globalement plus petites et moins dispersées que celles des saumons sauvages.26 1.

VariablesN, q et r sont des entiers.EntréeSaisir NTraitementq prend la valeur du quotient entier de N par 4r prend la valeur du reste de la division de N par 4Si r = 0 Alors afficher q Sinon afficher q + 1Fin Si

43Chapitre 5 Statistiques

2. G(4,5 ; 95)3. Voir graphique.4. 25 × 4,5 – 17,5 = 95 et 25 × 1 – 17,5 = 7,5.5. a. Pour 2018, x = 9 et 25 × 9 – 17,5 = 207,5.b. Voir graphique.44 Voir manuel page 253.45 Faux : comme les valeurs de y diminuent quand celles de

x augmentent, une droite d’ajustement convenable aura un coefficient directeur négatif.46 Vrai : – 0,91 × 7 + 51,29 = 44,92.47 y = 204x – 10,61.48 y = 0,5x + 849 Fichiers associés sur le site www.bordas-indice.fr et sur

le CD-Rom Professeur :05_TSTMG_exercice49.xls (Excel 2003),05_TSTMG_exercice49.xlsx (Excel 2007),05_TSTMG_exercice49.ods (OpenOffice).• En G2, on affiche le coefficient directeur de la droite d’ajustement de y en x par la méthode des moindres carrés.• En G3, on affiche l’ordonnée à l’origine de cette droite.50 1. y = 9,22x + 66,02.

2. a. 167,44 euros.b. En 2019.51 1.

10

y

x

10

2. y = – 12,6x + 227,7.3. a. – 12,6 × 7 + 227,7 = 139,5 donc A(7 ; 139,5) � (d).b. B (15 ; 38,7) appartient à (d).4. a. 108 acheteurs.b. 16,50 euros.52 Vrai : l’équation est obtenue avec la calculatrice.53 Faux car 52 vrai.54 1. Me = 38 ; Q1 = 37 ; Q3 = 39.

2.

35 36 37 38 39 40 41 42

3. 75 % des femmes ont une pointure plus petite que la totalité des hommes et environ la moitié des hommes ont une pointure plus élevée que toutes les femmes.L’écart interquartile pour la série des pointures des hommes est un peu plus faible que celui de la série des pointures des femmes, tandis que l’étendue est la même.Si la dispersion est sensiblement identique, les femmes ont une pointure globalement plus petite que les hommes.

36 1.

10

y

x

10

2. G(15 ; 53,8).37 Exercice résolu.38 Faux : aucune valeur de xi n’est égale à 77.39 Vrai : la moyenne de la série des xi est 19,5 et la moyenne

de la série des yi est 80,6.40 1.

10

y

x100

200 M6

M1

2. b. 235 milliers d’euros.c. En 2015.41 1.

10

y

x

1

G1

G2

2. G1(2,5 ; 2,45) et G2(6,5 ; 3,65).3. b. 47 000 euros.c. En 2019.42 Exercice résolu.43 1.

10

y

x10

100

200

A

O

44

Fichiers associés sur le site www.bordas-indice.fr et sur le CD-Rom Professeur :05_TSTMG_TP.xlsx (Excel 2007),05_TSTMG_TP.xls (Excel 2003),05_TSTMG_TP.ods (OpenOffice),05_TSTMG_ TP_correction.xlsx (Excel 2007),05_TSTMG_TP_correction.xls (Excel 2003),05_TSTMG_ TP_correction.ods (OpenOffice).A. Ajustement affine1. Voir fichier correction.2. a. =PENTE(B2:G2;B1:G1) .b. =ORDONNEE.ORIGINE(B2:G2;B1:G1) .c. y = 1,48v – 43,16.3. Pour v = 150, on obtient y =178,84.4. Non, car 224,3 est nettement supérieur à 178,84.B. Ajustement non affine1. =B1^2 .2. a. et b. Voir fichier correction3. Voir fichier correction.4. a. y = 0,01x – 0,47 et y = 0,01× 1502 – 0,47 = 224,53.b. À l’estimation de la distance d’arrêt pour une vitesse de 150 km/h.c. L’estimation est conforme au résultat donné à la question A. 4.C. Formules approchant y en fonction de v1. L’ajustement de y en x par la méthode des moindres carrés est donné par la relation y = 0,01x – 0,47. On a posé x = v 2 donc on obtient la relation y = 0,01v 2 – 0,47.2. Pour y =180, on obtient v 2 =18 047, soit v ≈ 134,34.

3. La méthode du code de la route conduit à l’estimation

y = v10( )2 soit y = v

2

100 donc à y = 0,01v 2.

Cette méthode donne des résultats comparables à ceux obtenus par la méthode précédente. La méthode donnée par le manuel du code de la route est en revanche beaucoup plus simple.

P O U R A P P R O F O N D I R60 1. y = – 3,25x + 344,5.

2. r (x) = x × y = –3,25x2 + 344,5x.3. a. r’(x) = – 6,5x + 344,5.b.

x 35 53 70

Signe de r’ + 0 –

r8 076,25

9 129,25

8 190,00

c. Prix 53 euros pour une recette de 9 129,25 euros.61 1. y = – 0,14x + 599,20.

2. a. B(x)= – 0,14x 2 + 599,20x – 50 000.b. B’(x)= – 0,28x + 599,20.c.

x 0 2 140 4 000

Signe de B’ + 0 –

B–50 000

591 144

106 800

Prix assurant le bénéfice maximal : 2 140 euros.

55 1. Nombre annuel moyen de visiteurs : 8 200.2. Écart-type : 314,63 à 0,01 près.56 1. et 2.

10

y

x200

250

300

3. 293 milliards d’euros.57 1. y = 7,6x + 204.

2. 7,6x + 204 � 500 équivaut à x � 38,94, soit x � 39 ce qui correspond à l’année 2045.

P O U R FA I R E L E P O I N TVoir manuel page 253. Les corrigés détaillés sont dispo-nibles sur le site www.bordas-indice.fr.

ACCO M PAG N E M E N T P E R S O N N A L I S É58

10

y

x

10

59 y = 5,4x + 51,9.

T R AVAU X P R AT I Q U E STP Distance d’arrêt d’un véhicule

L’objectif de ce TP est de déterminer, à l’aide du tableur, une re-lation entre la vitesse d’un véhicule et sa distance d’arrêt. Nous utiliserons deux méthodes pour obtenir un ajustement affine : en utilisant les fonctions du tableur, donnant le coefficient directeur et l’ordonnée à l’origine de la droite de régression par la méthode des moindres carrés d’une part, et en utilisant la courbe de ten-dance d’autre part. Après un changement de variable, nous obte-nons une relation que nous comparons avec celle donnée par le manuel du code de la route.

45Chapitre 5 Statistiques

2. Oui car le nuage a une forme « allongée ».3. G(4 ; 1 200).4. a. y = 80,4x + 878,6.5. a. Un peu moins de 1 700.b. 80,4 × 10 + 878,6 = 1 682,6c. Le 14e jour et 4 personnes seront refusées.

Sujet C1.

G

10

y

x1 600

1 700

2 000

2. a. G(4,5 ; 1 806,25).b. y = 44x + 1 608.c. Voir graphique.3. a. Estimation du salaire en 2015 : 2 090 euros.b. Non car 44 × 16 + 1 608 = 2 312.

Sujet DFichiers associés sur www.bordas-indice.fr et sur le CD-Rom Professeur :05_TSTMG_sujetD.xlsx (Excel 2007),05_TSTMG_sujetD.xls (Excel 2003),05_TSTMG_sujetD.ods (OpenOffice).1. Q1=230 , Me = 238 et Q3 = 250.

210 215 220 225 230 235 240 245 250 255 260

2. =MOYENNE(A1:K4) .3. Réponse d.4. La moyenne du deuxième échantillon est plus élevée et l’écart-type plus faible : la croissance des arbres sur la deuxième colline est plus homogène.

Sujet E1. a. Minimum : 6 ; maximum : 16.b. Q1 = 7 ; Me = 8 et Q3 = 10.c.

6 7 8 9 10 11 12 13 14 15 16

62 1. y = an + bn = a + bn .

2. Non un ajustement n’est pas justifié.3. a.

xi 1 0,5 0,33 0,25 0,2 0,16 0,14 0,12 0,11 0,1yi 1 504 809 540 438 375 320 290 254 231 212

b)

10

y

x

100

c. Oui car le nuage a une forme « allongée ».4. a. y = 1 426,05x + 82,32.b. a ≈ 82,32 et b ≈ 1 426,05.c. 153,62 euros.

C A P V E R S L E B AC

Zoom sur le tableur

Fichiers associés sur le site www.bordas-indice.fr et sur le CD-Rom Professeur :05_TSTMG_zoomtableur.xls (Excel 2003),05_TSTMG_ zoomtableur.xlsx (Excel 2007),05_TSTMG_ zoomtableur.ods (OpenOffice).

Sujet A1. Réponse b.2. Réponse c.3. Réponse b.4. Réponse a.

Sujet B1.

10

y

x

1 000

1 500

800

G

46

Sujet G1.

20

y

x

2

G1

G2

2. a. G1(16 ; 15,5) et G2(26 ; 22,5).b. (G1G2) a pour équation y = mx + p avec m = 22,5 – 15,5

26 – 16 = 0,7

puis 15,5 = 0,7 × 16 + p soit p = 15,5 – 0,7 × 16 = 4,3.3. a. 0,7 × 24 + 4,3 = 21,1. Soit 21 100 euros pour 24 ventes mensuelles.b. 21.

63 1.

028

29

30

1

y

x

2. y = 0,21x + 27,98.3. Voir graphique.4. 30,7 ans.64 1. Me = 7, Q1 = 6 et Q3 = 10.

2.

0 1 2 3 4 5 6 7 8 9 10 11 1312

3. x ≈ 7,51 et σx ≈ 2,85.

2. a. Q1 = 7 , Me = 9 et Q3 = 12.b.

5 6 7 8 9 10 11 12 13 14 15 16

c. Moyenne : 10,3 et σ ≈ 3,33.3. Pour les deux épreuves au moins 25 % des notes sont inférieures ou égales à 7, mais le troisième quartile et la médiane de la série 2 sont plus élevés que ceux de la série 1, l’écart interquartile pour la série 2 est donc plus élevé que celui de la série 1.Les notes de la série 2 sont beaucoup plus dispersées que celles de la série 1.4. a. Moyenne 11,6.b. Les résultats de l’épreuve 3 sont meilleurs et plus homogènes que ceux de l’épreuve 2.

Sujet F1. a.

10

y

x100

105

b. y = 0,85x + 99,39.c. Voir graphique.d. y = 0,9 × 8 + 99,4 = 106,6.2. a. f ’(x) = – 0,16x + 1,44.b.

x 1 9 15

Signe de f’ + 0 –

f100,06

105,18

102,30

c. f (9) = 105,18.d.

x 7 8 9 10 11 12 13 14f (x) 104,86 105,1 105,18 105,1 104,86 104,46 103,9 103,18

e.

10

y

x100

105�f

f. À partir de 2017.

47Chapitre 6 Probabilités conditionnelles

A Le programme

6CHAPITRE

Probabilités conditionnelles

Objectif : Découvrir la notion de conditionnement.

Contenus Capacités attendues Commentaires

ConditionnementConditionnement par un événement de probabilité non nulle.Notation PA(B).

• Construire un arbre pondéré en lien avec une situation donnée.• Exploiter la lecture d’un arbre pon-déré pour déterminer des probabilités.• Calculer la probabilité d’un évé-nement connaissant ses probabilités conditionnelles relatives à une parti-tion de l’Univers.

On représente une situation à l’aide d’un arbre pondéré ou d’un tableau.Un arbre pondéré correctement construit constitue une preuve.Le vocabulaire lié à la formule des probabilités totales n’est pas un attendu du programme mais la mise en œuvre de cette formule doit être maîtrisée.Cette partie du programme se prête particulièrement à l’étude de situations concrètes.

Nous avons placé dans ce chapitre la partie du programme relative au conditionnement. Les notions nouvelles concernant ce chapitre sont peu nombreuses : conditionnement par un événement de probabilité non nulle et arbres pondérés.La première séquence permet, à partir de données présentées en tableau, de découvrir la notion de probabilité conditionnelle et la formule permettant le calcul de ces probabilités. On retrouve, dans la première page de cours, la définition et les propriétés d’une probabilité conditionnelle ainsi que l’utilisation de tableaux à double entrée.La seconde séquence est consacrée aux arbres pondérés et à la formule des probabilités totales. La seconde page de cours explique comment construire et utiliser ces arbres pondérés.À noter que la notion d’arbres pondérés est déjà connue des élèves :– en classe de Troisième et de Seconde, on s’est intéressé à la succession de deux expériences (éventuellement trois), pas nécessairement identiques. Ces activités ont permis à l’élève de se familiariser avec les arbres pondérés construits intégralement ;– en Première, on s’est intéressé à la répétition d’une même expérience aléatoire, un certain nombre n de fois (ce nombre n pouvant éventuellement être grand), dans le cadre de la loi binomiale.Le contenu du programme de Terminale est dans le prolongement de ces apprentissages.Nous avons décidé de formaliser cette partie de cours en détaillant la méthode de construction d’un arbre pondéré, en proposant un peu de vocabulaire et en donnant quelques règles.Les capacités attendues étant : « Construire un arbre pondéré en lien avec une situation donnée », « Exploiter la lecture d’un arbre pondéré pour déterminer des probabilités » et « Calculer la probabilité d’un événement connaissant ses probabilités conditionnelles relatives à une partition de l’univers », ces capacités sont reprises dans les trois Savoir-faire de la séquence 2.Concernant l’accompagnement personnalisé, la page « Revoir les points essentiels » revient sur deux points importants : le calcul d’une probabilité conditionnelle et la construction d’un arbre pondéré.Le TP, quant à lui, propose l’étude de la fiabilité d’un test de dépistage et de l’évolution de cette fiabilité en fonction de la proportion d’individus « positifs » dans la population testée.

B Notre point de vue

48

Les notions abordées dans le chapitre 6• Probabilités conditionnelles• Arbres pondérés• Probabilités totales

C Avant de commencerIl s’agit, dans un premier temps, de réactiver les notions sur les probabilités vues dans les classes précédentes comme : calculer une probabilité, donner la probabilité d’une réunion d’événements, d’une intersection ou de l’événement contraire.Dans un deuxième temps, il s’agit de réinvestir les notions vues sur la représentation d’une situation par un arbre pondéré lors d’une répétition d’épreuves de Bernoulli.Voir manuel page 250 et le site www.bordas-indice.fr pour les corrigés détaillés.

D ActivitésActivité Cadres en entreprise1

Cette activité permet, à partir de données présentées en tableau, de découvrir la notion de probabilité conditionnelle et la formule permettant le calcul de ces probabilités. Les valeurs du tableau étant des effectifs, on commence par déterminer les probabilités correspondantes.1.

Femmes Hommes TotalCadres 15 17 32Autres employés 52 76 128Total 67 93 160

2. a. On a les événements :• –C : « la personne choisie n’est pas un cadre »,• F  C : « la personne choisie est une femme cadre »,• F  

–C : « la personne choisie est une femme non-cadre ».

b. P(F) = 67160

; P(C) = 32160 = 1

5 ; P(

–C) = 93

160 ;

P(F  C) = 15160

= 332

; P(F  –C) = 52

160 = 13

40.

3. a. PC(F) = 1532 b) P(F>C)

P(C)=

15160( )32

160( ) =1532

= PC(F).

4. a. PF(C) = 1567

b) P(C>F)P(F)

=15

160( )67

160( ) =1567

= PF(C).

5. 5267 = PF(

–C) : probabilité que la personne choisie ne soit pas

un cadre, sachant que c’est une femme.52

128 = P–

C(F) : probabilité que la personne choisie soit une

femme, sachant que ce n’est pas un cadre.

Activité Contrôle de fabrication2L’objectif de cette activité est d’apprendre à utiliser un tableau pour déterminer des probabilités, en particulier des probabilités conditionnelles. L’utilisation du tableur permet de réfléchir aux formules à mettre en œuvre pour calculer ces probabilités à partir des quelques valeurs connues.Fichiers associés sur www.bordas-indice.fr et sur le CD-Rom Professeur :06_TSTMG_activite2.ods (OpenOffice),06_TSTMG_activite2_correction.ods (OpenOffice),06_TSTMG_activite2.xls (Excel 2003),06_TSTMG_activite2_correction.xls (Excel 2003),06_TSTMG_activite2.xlsx (Excel 2007),06_TSTMG_activite2_correction.xlsx (Excel 2007).1. C’est la valeur de P(

–A).

2. a. « Le panneau est produit par la machine M1 et est défectueux ». Formule à placer en B2 : =B4*0,02 .b. 0,4 × 0,02 = 0,008.3. Formule à placer en C2 : =C4*0,03 .C’est la probabilité de choisir un panneau produit par la machine M2 qui est défectueux.4. a. P(F) = 0,4 × 0,02 + 0,6 × 0,03 = 0,026.b. Formule à placer en D2 : =B2+C2 .

5. PF(–A) = 0,018

0,026 ≈ 0,692.

Activité Contrat d’assurance3Cette activité donne l’occasion de lire et de compléter des probabilités sur un arbre pondéré. Ces informations sont ensuite utilisées pour déterminer la probabilité de certains événements.

49Chapitre 6 Probabilités conditionnelles

1. On a : PA(B) = 0,90 et P–A(B) = 0,85.2. a. P(A  B) = 0,6 × 0,9 = 0,54 ;P(

–A  B) = 0,4 × 0,85 = 0,34 .

b. P(B) = P(A  B) + P(–A  B), soit P(B) = 0,88.

c.

A–A Total

B 0,54 0,34 0,88–B 0,06 0,06 0,12

Total 0,6 0,4 1

3. a.

0,9

0,6

0,4

0,1

0,85

0,15

B

A

B

A

B

B

Événement

A�B

A�B

A�B

A�B

Probabilité

0,54

0,06

0,34

0,6

b. Chemins conduisant au choix souhaité : A  B et –A  B.

c. On fait la somme de tous les chemins conduisant à l’événement B.

1. 0,3 représente P(E). P(F) = 0,7.2. PF(A) = 0,2.

0,4

0,3

0,7

0,6

0,2

0,8

AE

A

AF

A

3. a. P(E  A) = 0,3 × 0,4 = 0,12.b. C’est l’événement F  A, P(F  A) = 0,14.c. P(A) = 0,12 + 0,14 = 0,26.4. a. P(E  

–A) = 0,18 et P(F  

–A) = 0,56.

b. P(–A) = 0,18 + 0,56 = 0,74.

Activité Orientations post-bac4Cette activité permet de découvrir la formule des probabilités totales grâce au calcul de la probabilité d’un événement : en utilisant un tableau de probabilité puis en utilisant un arbre pondéré.

E Exercices• PR(Q) = P(Q>R)

P(R)= 0,04

0,22= 2

11 : « probabilité que l’exercice

choisi soit un QCM sachant que c’est un exercice sur les probabilités ».7 1. P(A  B ) = P(A) × PA(B ) = 0,3 × 0,2 = 0,06.

2. PB(A) = P(A>B)P(B)

= 0,060,54

= 19

.

8 Voir manuel p. 253.9 1. a. P(C ) = 0,7 ; P(

–C) = 0,3 ; b. PC(V) = 0,8 ; c. P–C(V) = 0,1.

2. a. • C  V : « l’employé est un commercial qui possède une voiture de fonction ».• P(C  V) = 0,7 × 0,8 = 0,56.b. • –

C   V : « l’employé n’est pas un commercial qui possède une voiture de fonction ».• P(

–C  V) = 0,3 × 0,1 = 0,03.

10 1. 7082 400

= 59200

2. 1 1042 400

= 2350

3. 4322 400

= 950

4. 8281 104

= 34 5. 864

1 692= 24

4711 Voir manuel p. 253.12 1.

A B C TotalD 0,2 0,1 0,1 0,4–D 0,25 0,15 0,2 0,6

Total 0,45 0,25 0,3 1

2. P(B) = 0,25 ; P(B D) = 0,1 ; P(–D) = 0,6.

3. PB(D) = P(D>B)P(B)

= 0,10,25

= 0, 4 .

P –D(B) = P(D>B)

P(D)= 0,15

0,6= 0,25 .

P O U R D É M A R R E R1 a. P(C) : probabilité que la personne choisie soit en CDI ;

b) P(S) : probabilité que la personne choisie soit syndiquée ;c) P(C  S) : probabilité que la personne choisie soit en CDI et syndiquée ;d) PC(S) : probabilité que la personne choisie soit syndiquée sachant qu’elle est en CDI ;e) PS(C) : probabilité que la personne choisie soit en CDI sachant qu’elle est syndiquée.2 Voir manuel p. 253.3 1. « Parmi les femmes », PF(R) = 0,25.

2. « Un tiers des hommes », PH(R) = 13

.

3. « Chez les personnes retraitées », PR(F) = 0,45.4. « Lorsqu’on interroge un homme », PH(

–R) = 0,67.

5. « 55 % des personnes non retraitées », P–R(F) = 0,55.

4 1. PA(B) = P(B>A)P(A)

= 0,30,75

= 0, 4.

2. PB(A) = P(A>B)P(B)

= 0,30, 45

= 23

.

5 1. PD(C) = P(C>D)P(D)

= 0,10, 4

= 0,25 .

2. PC(D) = P(D>C)P(C)

= 0,10,25

= 0, 4 .

6 1. P(Q) = 0,1 ; P(R) = 0,22 ; P(Q  R) = 0,04.

2. • PQ(R) = P(R>Q)P(Q)

= 0,040,1

= 0, 4 : « probabilité que l’exercice

choisi soit une question sur les probabilités sachant que c’est un QCM ».

50

4. PB(T) = 0,630,84

= 0,75.

25 1. P(M) = 815

; P(T) = 515

= 13

.

2. • P(–M) = 7

15 : probabilité de tirer un numéro pair.

• P(M T) = 315

= 15

: probabilité de tirer un numéro impair et

multiple de trois.

• P(–MT) = 2

15 : probabilité de tirer un numéro pair et multiple

de trois.3. • PT(M) = 3

5 : probabilité de tirer un numéro impair sachant

qu’il est multiple de trois.

• P –M(T) = 27

: probabilité de tirer un multiple de trois sachant

que c’est un numéro pair.26 Exercice résolu.27 Voir manuel p. 253.28 P(A B) = 0,72 + 0,47 − 0,88 = 0,31 ;

PB(A) = P(A>B)P(B)

= 3147

≈ 0,66 ;

PA(B) = P(B>A)P(A)

= 3172

≈ 0,43.

29 P(C D) = P(C) × PC(D) = 0,3 × 0,2 = 0,06 ;P(C D) = P(C) + P(D) − P(C D) ;P(D) = 0,8 − 0,3 + 0,06 = 0,56 ;

PC(D) = P(C>D)P(C)

= 0,060,3

= 0,2.

30 1. • P(A) = 0,55 : il y a 55 % de chances que la pièce choisie provienne de la 1re machine.• PA(D) = 0,01 (resp. PB(D) = 0,02) : la pièce choisie provenant de la 1re machine (resp. la 2e machine), cette pièce a 1 % (resp. 2 %) de chances d’être défectueuse.2. P(B) = 0,45.3. P(A D) = 0,55 × 0,01 = 0,0055.C'est la probabilité que la pièce provienne de la première machine et qu’elle soit défectueuse.4. P(B D) = 0,45 × 0,02 = 0,009.31 1. P(N) = 0,36 ; P(B) = 1 − 0,36 = 0,64 ;

PN(M) = 23

; PB(M) = 0,875.

2. P(B M) = 0,64 × 0,875 = 0,56.C'est la probabilité que le client ait souscrit un forfait en boutique et avec mobile.

3. P(N M) = 0,36 × 23

= 0,24.

32 1. La proposition est vraie :

PA(B) = P(B>A)P(A)

= P(A>B)P(B)

= PB(A).

2. Réciproque : « Si A et B sont deux événements de probabilité non nulle tels PA(B) = PB(A), alors P(A) = P(B) ».33 On considère les événements : S : « obtenir un six » et

P : « obtenir un nombre pair ».P(S) = 1

6 ; P(P) = 12 et P(S P) = 1

6.

PP(S) = P(S>P)P(P)

= 13

.

34 Fichier associé sur www.bordas-indice.fr et sur le CD-Rom Professeur :06_TSTMG_exercice34.alg (AlgoBox).

13 1.

33 tours 45 tours TotalBon état 12 6 18Abîmés 8 9 17Total 20 15 35

2. P(T) = 2035

= 47 ; P(B) = 18

35 ; P(B T) = 1235 .

3. • PT(B) = P(B>T)P(T)

= 1220

= 0,6 : « probabilité de choisir un

disque en bon état sachant que le disque est un 33 tours ».

• PB(T) = P(B>T)P(B)

= 1218

= 23

: « probabilité de choisir un disque

33 tours sachant que le disque est en bon état ».14 1. 0,65 = P(B) ; 0,1 = PA(C) et 0,6 = PB(

–C).

2. P(A) = 0,35 ; PB(C) = 0,4 et PA(–C) = 0,9.

3. P(A C) = 0,35 × 0,1 = 0,035 ; P(B C) = 0,65 × 0,4 = 0,26.15 Voir manuel p. 253.16 1. 0,52

0,65

0,35

0,48

0,36

0,64

FE

F

F

FE

2. P(–E) = 0,35, PE(

–F) = 0,48 et P–E(

–F) = 0,64.

3. P(E F) = 0,65 × 0,52 = 0,338.De même : P(E

–F) = 0,312 ; P(

–E F) = 0,126 et P(

–E

–F) = 0,224.

P O U R s ’ E n t R A î n E R17 1. a. P(G) = 0,52 ; b. P(D) = 0,65 ;

c. P(F D) = 0,26 d. P(G D) = 0,39.2. a. 48 % des élèves sont des filles.b. 22 % des élèves sont des filles externes.c. 60 % des demi-pensionnaires sont des garçons.3. • PD(F) : probabilité de choisir une fille sachant que l’élève est demi-pensionnaire.• PG(D) : probabilité de choisir un demi-pensionnaire sachant que l’élève est un garçon.18 Voir manuel p. 253.

19 Faux : 1PA (B)

1 alors que PB(A) 1.

20 Vrai : si P(A B) = 0, on a = P(A>B)P(B)

= 0.

21 Vrai : PA(A) = P(A>A)P(A)

.

22 Faux : la probabilité est 0.23 1. P(S) = 0,75 ; P(C) = 0,5 et P(S C) = 0,3.

2. PC(S) = 0,30,5 = 0,6.

3. On veut PS(C) = 0,30,75 = 0,4.

24 1. P(B) = 0,84 ; P(T) = 0,75 et P(B T) = 0,63.

2. PT(B) = 0,630,75

= 0,84.

3. PT(–B) = 1 − 0,84 = 0,16.

51Chapitre 6 Probabilités conditionnelles

b. P(B E) = 0,06 ; P(E I) = 0,665.

3. PE(I) = 0,6650,671 ≈ 0,99.

4. P(C I) = 0,035.47 Exercice résolu.48 Fichiers associés sur www.bordas-indice.fr et sur le

CD-Rom Professeur :06_TSTMG_exercice48.ods (OpenOffice),06_TSTMG_exercice48.xls (Excel 2003),06_TSTMG_exercice48.xlsx (Excel 2007).1. En D2 : P(B) ; en B3 : P(A

–B).

2. =D2−B2 .

3. =B2/B4 ou =B2/B$4 pour pouvoir effectuer la recopie de la formule.4. P–A(B).49 Vrai.

Tableau complété pour les exercices 49 à 51

A–A Total

B 0,32 0,32 0,64–B 0,16 0,2 0,36

Total 0,48 0,52 150 Vrai.

51 Faux : PA(B) = 0,320, 48

= 23

.

52 1. 0,2

0,6

0,4

0,8

0,7

0,3

BA

B

B

BA

2. P(–A) = 0,4 ; P–A(B) = 0,7.

3. P(A B) = 0,6 × 0,2 = 0,12 ;P(

–A B) = 0,4 × 0,7 = 0,28.

53 1. P(F) = 0,6 ; PF(M) = 0,45 et PG(M) = 0,35.2.

0,45

0,6

0,4

0,55

0,35

0,65

MF

G

M

M

M

3. a. P(F M) = 0,6 × 0,45 = 0,27 ;b. P(G M) = 0,4 × 0,35 = 0,14.54 1. P(V) = 0,2 ; PV(D) = 0,96 et P–V(D) = 0,05.

2.0,96

0,2

0,8

0,04

0,05

0,95

DV

D

V

D

D

1. L’algorithme calcule PA(B) connaissant P(A) et P(A B).2. Voir rubrique « Algorithmique » pp. 228 à 231.35 Fichier associé sur www.bordas-indice.fr et sur le

CD-Rom Professeur :06_TSTMG_exercice35.alg (AlgoBox).1. Afficher « Valeur de P(A) »

Saisir XAfficher « Valeur de PA(B) »Saisir YC prend la valeur X*YAfficher C

2. Voir rubrique « Algorithmique » pp. 228 à 231.36 Vrai : P(A B) = 0,4 × 0,3 = 0,12.

37 Faux : PB(A) = 0, 40,5

= 0,8.

38 Vrai : P(A) 1, soit 1P(A)

1 d’où P(A>B)P(A)

P(A B).

39 Voir manuel p. 253.40 1. P(X 20) ≈ 0,952 et P(X 10) = 1 − P(X 9) ≈ 0,960.

2. P(10 X 20) = P(X 20) − P(X 9) ≈ 0,912.

PX 10(X 20) = P(10 < X < 20)P(X > 10)

≈ 0,9120,960

≈ 0,950.

41 1. X suit une loi binomiale de paramètres 15 et 0,2.2. P(X 1) = 1 − P(X = 0) ≈ 0,9648. 3. P(X 5) = P(X 4) ≈ 0,8358.

4. PX 1(X 5) = P(1< X < 4)P(X > 1)

≈ 0,80060,9648 ≈ 0,8298.

42 Vrai : P(X 5) ≈ 0,9666 ;P(2 X 5) = P(X 5) − P(X 2) ≈ 0,4295 ;

PX 5(X 2) ≈ 0, 42950,9666

≈ 0,4443.

43 1.

S–S Total

T 0,24 0,45 0,69–T 0,13 0,18 0,31

Total 0,37 0,63 1

2. PT(S) = 0,240,69

= 823

≈ 0,35.

3. PS(T) = 0,240,37

= 2437

≈ 0,65.

44 1. P(–A) = 0,3 et P(

–B) = 0,5.

2. a. b. c. d. et e.

A–A Total

B 0,3 0,2 0,5–B 0,4 0,1 0,5

Total 0,7 0,3 1

PA(B) = 37

; PA(–B) = 4

7 ; PB(A) = 3

5 ; PB(

–A) = 2

5 et P–A(

–B) = 1

3.

45 Voir manuel p. 253.46 1.

B I TotalC 0,294 0,035 0,329E 0,06 0,665 0,671

Total 0,3 0,7 1

2. a. PC(B) = 0,2940,329

= 4247 ≈ 0,8936.

PB(C) = 0,2940,3

= 4950

= 0,98.

52

2. P(D) = 0,025.

3. PD(C) = 425 = 0,16, soit 16 %.

62 1. PA(N) = 0,7. 70 % des vêtements provenant du fournis-seur A sont vendus à un prix normal.2. 0,7

0,4

0,6

0,3

0,6

0,4

NA

BN

R

R

3. a. « Le vêtement provient du fournisseur A et est vendu à un prix normal ».P(N A) = 0,28.b. P(N B) = 0,36.c. P(N) = P(N A) + P(N B) = 0,64.

4. PN(A) = 0,280,64

= 716

= 0,4375.

P O U R fA i R E l E P O i n tIl s’agit, dans le cas de situations mettant en jeu des probabilités conditionnelles, de tester la maîtrise du calcul de probabilités et l’utilisation de tableaux ou d’arbres pondérés.Voir manuel p. 254. Les corrigés détaillés sont disponibles sur le site www.bordas-indice.fr.

AccO M PAg n E M E n t P E R s O n n A l i s ÉL’objectif est de donner une méthode d’analyse à l’élève afin qu’il comprenne l’énoncé auquel il a à faire et qu’il sache répondre aux différentes questions proposées.La présentation est donnée sous forme de question/ réponse du type « que dois-je faire dans telle situation… »63 On considère les événements F : « l’élève est une fille » et

E : « l’élève est externe ».On a P(F) = 0,6 ; P(E) = 0,4 et PE(F) = 0,5.Ainsi :P(E F) = 0,4 × 0,5 = 0,2 et PF(E) = P(E>F)

P(F)= 0,2

0,6= 1

3.

64 1. et 2. 0,74

0,52

0,48

0,26

0,79

0,21

MF

H

M

M

M

P(M) = 0,52 × 0,74 + 0,48 × 0,79 = 0,764.

3. P(V D) = 0,2 × 0,96 = 0,192.4. P(D) = P(V D) + P(

–V D) = 0,2 × 0,96 + 0,8 × 0,05 = 0,232.

55 Voir manuel p. 253.56 1. P(F) = 0,4 ; P(M) = 0,3 ; P(D) = 0,3.

PF(R) = 0,95 ; PM(R) = 0,6 ; PD(R) = 0,4.D’où l’arbre :

0,95

0,4

0,3

0,3

0,05

0,6

0,4

0,4

0,6

RF

M

D

R

R

R

R

R2. a. P(D R) = 0,3 × 0,4 = 0,12.b. P(F

–R) = 0,4 × 0,05 = 0,02.

c. P(R) = 0,4 × 0,95 + 0,3 × 0,6 + 0,3 × 0,4 = 0,68.

3. P–R(M) = P(R>M)P(R)

= 0,3 × 0, 41– 0,68

= 38 = 0,375.

4. PR(F) = P(F>R)P(R)

= 0, 4 × 0,950,68

= 1934

≈ 0,56.

La sœur de Pierre a environ 56 % de chances d’avoir raison.5. Soit X le nombre de grilles réussies par Pierre.X suit une loi binomiale de paramètres 8 et 0,68.On veut : P(X 4) ≈ 0,925.57 Vrai : P(

–A

–B) = 0,6 × 0,75 = 0,45.

58 Faux : P(–B) = 0,4 × 0,35 + 0,6 × 0,25 = 0,29.

59 Vrai : PB(A) = P(A>B)P(B)

= 0, 4 × 0,651– 0,29

≈ 0,37.

60 1. • PM(T) : probabilité que le test soit positif sachant que la personne est malade.• P–M(

–T) : probabilité que le test soit négatif sachant que la

personne n’est pas malade.Arbre illustrant la situation :

0,95

0,005

0,995

0,05

0,05

0,95

TM

T

M

T

T

2. a. P(M T) = 0,005 × 0,95 = 0,00475.P(

–M T) = 0,995 × 0,05 = 0,04975.

b. P(T) = 0,00475 + 0,04975 = 0,0545.

3. PT(M) = 0,004750,0545

≈ 0,087, soit environ 8,7%.

61 1.

Machine A B C TotalDéfectueux 0,012 0,009 0,004 0,025Non défectueux 0,588 0,291 0,096 0,975Total 0,6 0,3 0,1 1

53Chapitre 6 Probabilités conditionnelles

P O U R A P P R O f O n D i R65 On considère les événements :

– E : « la personne rencontrée est étrangère » ;– S : « la personne rencontrée compte rester plus d’une semaine ».L’arbre pondéré ci-dessous traduit la situation :

0,7

0,32

0,68

0,3

0,3

0,7

SE

S

S

SE

P(S) = 0,32 × 0,7 + 0,68 × 0,3 = 0,428.

PS(E) = 0,32 × 0,70, 428

≈ 0,523, soit environ 52,3 %.

66 Notons respectivement A, B et C les événements : « Avoir voté pour le candidat A, B ou C », F et H les événements : « Être une femme » ou « Être un homme ».L’arbre pondéré ci-dessous traduit la situation :

0,5

0,5

0,2

0,3

0,5

0,3

0,7

0,4

0,6

H

F

H

F

H

F

H

F

A

B

C

P(F) = 0,5 × 0,5 + 0,3 × 0,7 + 0,2 × 0,6 soit P(F) = 0,58.67 1. 4

7

37

35

25

13

23

BA

B

B

BA

2. Remplacer :

T1 prend la valeur d’un entier aléatoire entre 1 et 5Si T1 3

par

T1 prend la valeur d’un entier aléatoire entre 1 et 8Si T1 5

3. Remplacer :

T2 prend la valeur d’un entier aléatoire entre 1 et 7Si T2 4

par

T2 prend la valeur d’un entier aléatoire entre 1 et 10Si T2 6

t R AvAU X P R At i Q U E sTP Fiabilité d’un test de dépistage

Fichiers associés sur www.bordas-indice.fr et sur le CD-Rom Professeur :06_TSTMG_TP.ods (OpenOffice),06_TSTMG_TP_correction.ods (OpenOffice),06_TSTMG_TP.xls (Excel 2003),06_TSTMG_TP_correction.xls (Excel 2003),06_TSTMG_TP.xlsx (Excel 2007),06_TSTMG_TP_correction.xlsx (Excel 2007).Ce TP propose d’étudier la fiabilité d’un test de dépistage de produits dopants et d’observer, à l’aide d’un tableur, l’évolution de cette fiabilité en fonction de la proportion d’individus « positifs » dans la population testée.A. Simulations avec un tableur1. =A2+1 .2. a. La formule renvoie la valeur 1 avec une probabilité p (égale au contenu de la cellule G1) et la valeur 0 sinon.3. On peut dire que le sportif est dopé et que son test est positif.4. a. =SOMME(B2:B2501) .b. =G3/2500 .5. En G6 : =SOMME(C2:C2501) ;en G7 : =G6/2500 ;en G9 : =SOMME(D2:D2501) ;en G10 : =G9/2500 ;6. a. Formule en G13 : =G9/G6 .Exemple de tableau que l’on peut obtenir :

p 0,01 0,05 0,1 0,2 0,5 0,8 0,95 0,99d 0,186 0,55 0,75 0,851 0,969 0.994 0.998 1

b. Quand p augmente, d augmente aussi.c. Quand la valeur de p est faible, le test n’est pas fiable.Par exemple, pour une proportion de 1 % de dopés dans la population ; parmi les sportifs pour lesquels on a obtenu un test positif, moins de 19 % étaient réellement dopés.B. Étude d’un cas particulier1. a. P(D) = 0,05 ; PD(T) = 0,94 et P–D(T) = 0,04.b. 0,94

0,05

0,95

0,06

0,04

0,96

TD

T

N

ND

P(T) = 0,9p + 0,04.2. P(D T) = 0,05 × 0,94 = 0,047 ; P(

–D T) = 0,95 × 0,04 = 0,038.

Ainsi P(T) = 0,085.3. a. PT(D) = 0,047

0,085 ≈ 0,553.

b. Le résultat est proche de la valeur simulée : 0,55.

4. a. PN(–D) = P(N>D)

P(N)= 0,95 × 0,96

1– 0,085 ≈ 0,997.

b. Le test n’est pas très efficace pour déterminer si un sportif est dopé (un sportif déclaré positif n'a que 55,3 % de chances d’être dopé). Il est en revanche beaucoup plus efficace pour déterminer si un sportif est non-dopé (un sportif déclaré négatif a 99,7 % de chances d’être non-dopé).

54

70 1. Probabilité : 18

.

2. L’arbre pondéré ci-dessous traduit la situation :14

18

p

1– p

RT

R

R

RT

– Probabilité de tirer un roi :P(R) = 1

4p + 1

81− p( ) = 1

8p + 1

8.

3. Si p = 0, on retrouve P(R) = 18

. Si p = 1, la probabilité de tirer un roi est 1

4.

4. PR(T) = 2pp +1

= T(p).

5. Quand p augmente, T(p) augmente en se rapprochant de 1.

c A P v E R s l E B Ac

Zoom sur le tableur

Fichiers associés sur le site www.bordas-indice.fr et sur le CD-Rom Professeur :06_TSTMG_zoomtableur.xls (Excel 2003),06_TSTMG_zoomtableur.xlsx (Excel 2007),06_TSTMG_zoomtableur.ods (OpenOffice).

Sujet AArbre complété

0,8

0,4

0,6

0,2

0,3

0,7

VR

V

R

V

V

1. P(R V) = 0,4 × 0,8 = 0,32 : réponse a.2. P(V) = 0,4 × 0,8 + 0,6 × 0,3 = 0,5 : réponse d.3. P–R(

–V) = 0,7 : réponse b.

4. PV(R) = P(V>R)( )P(V)

: réponse a.

Sujet B1. Soit A l'événement : « La pièce prélevée a une masse inadéquate ». P(A) = 25

250= 1

10.

2. Soit B l'événement : « La pièce prélevée a une masse trop lourde ».Parmi les 25 pièces qui ont une masse inadéquate, on en compte 10 qui sont trop lourdes donc PA(B) = 10

25= 2

5.

Et remplacerT2 prend la valeur d’un entier aléatoire entre 1 et 3Si T2 1

parT2 prend la valeur d’un entier aléatoire entre 1 et 10Si T2 3

68 1. On considère les événements :– S : « le questionnaire est celui d’un élève de Seconde »,– P : « le questionnaire est celui d’un élève de Première »,– T : « le questionnaire est celui d’un élève de Terminale » et– R : « le questionnaire est celui d’un élève utilisant régulièrement Internet ».2.

S P T TotalR 0,38 0,315 0,175 0,87–R 0,02 0,035 0,075 0,13

Total 0,4 0,35 0,25 1

2. P(S R) = 0,38.3. P(

–R) = 0,13.

4. PR(P) = P(P>R)P(R)

= 0,3150,87

= 2158

≈ 0,362.

69 Si Sarah commence à jouer contre sa mère :

0,5 Oui

Oui

Non

Non

Oui

0,15

0,15

0,15

Non

Non

Non

0,5

0,5

0,6

0,4

0,6

0,4

0,5

0,5

0,5

0,5

0,5

0,5

0,5

G3G2

G1

P1

P2

G2

P2

P3

G3

P3

G3

P3

G3

P3

Partie 1Mère

Partie 2Père

Partie 3Mère

Réussite Probabilité

Sa probabilité de réussite est alors 0,45.Si Sarah commence à jouer contre son père :

0,6 Oui

Oui

Non

Non

Oui

Non

Non

Non

Partie 1Père

Partie 2Mère

Partie 3Père

Réussite

0,6

0,4

0,5

0,5

0,5

0,5

0,4

0,6

0,4

0,6

0,4

0,6

0,4

Probabilité

0,18

0,12

0,12

G3G2

G1

P1

P2

G2

P2

P3

G3

P3

G3

P3

G3

P3

Sa probabilité de réussite est alors 0,42.Sarah a intérêt à commencer avec sa mère comme premier partenaire.

55Chapitre 6 Probabilités conditionnelles

2. P(S1 G) = 0,75 × 0,92 = 0,69.

3. Pour réussir le second service, il faut avoir raté le premier :

P(–S1 S2 G) = 0,25 × 0,96 × 0,7 = 0,168.

4. P(G) = 0,69 + 0,168 = 0,858.

5. PG(S1) = 0,690,858

≈ 0,804.

6. 0,8584 ≈ 0,542.

Sujet F

Partie A

1. P(F) = 7201 200

= 0,6 ; P(T) = 6181 200

= 0,515 et P(–T) = 0,485.

2. « L’employé est une femme voyageant en train ».

P(F T) = 4681 200

= 0,39.

3. P –T(F) = 196 + 56462 +120

≈ 0,433.

Partie B

1.0,8

0,4

0,6

0,2

0,8

0,2

EU

D

E

E

E

2. P(D E) = 0,6 × 0,8 = 0,48.

3. a. Coût (C) 100 130 180 210Probabilité 0,12 0,48 0,08 0,32

b. P(C 200) = 1 − P(C = 210) = 0,68.

Sujet G

Correctif : il faut lire « 92 % des dossiers entraînent des frais de réparation et 16 % des frais de dommage corporel ».

1. P(M) = 0,92 ; P(C) = 0,16 et PM(C) = 0,12.

2. a. P(M C) = 0,92 × 0,12 = 0,1104.

b. Probabilité que « le dossier choisi entraîne des frais

de réparation matérielle mais pas de frais de dommages

corporels » : P(M –C) = 0,92 − 0,1104 = 0,8096.

c. PC(M) = 0,11040,16

= 0,69.

3. Soit V l’événement : « l’accident est dû à un excès de vitesse ».

P(V) = 0,45 et PV(C) = 0,30.

PC(V) = 0, 45 × 0,300,16

≈ 0,844.

Sujet H

1. P(C) = 1 − 15− 1

2= 3

10.

Sujet CNotons A1 l’événement : « Le favori gagne son 1er match » et A2

l’événement : « Le favori gagne son second match ».L’arbre suivant illustre la situation :

0,9

0,10,9

0,1A2

A2

A1

A1

1. Réponse b.2. Réponse b (0,9 × 0,9).3. Réponse c.4. Réponse a (0,1 + 0,1 × 0,9).

Sujet D1.

Défectueuses En bon état TotalUsine de Bordeaux 160 3 200 3 360Usine de Grenoble 66 1 200 1 266Usine de Lille 154 3 500 3 654Total 380 7 900 8 280

2. a. P(B) = 3 3608 280 ≈ 0,406.

b. P(D) = 3808 280

≈ 0,046.

c. « L’alarme provient de l’usine de Bordeaux et elle est défectueuse ».

P(B D) = 1608 280

≈ 0,019.

d. PB(D) = 1603 360

≈ 0,048.

e. On a : PB(D) = 1603 360

≈ 0,048 ; PG(D) = 661 266

≈ 0,052 et

PL(D) = 1543 654

≈ 0,042.

La probabilité de choisir une alarme défectueuse est la plus faible lorsque celle-ci provient de l'usine de Lille, puisque des trois nombres PB(D), PG(D) et PL(D), c'est ce dernier qui est le plus petit.On en déduit que c'est l'usine de Lille qui semble la plus efficace en terme de qualité de production.

Sujet E1.

0,92

0,75

0,25

0,08

0,96

0,04

0,7

0,3

1

G

G

G

G

GS1

S2

S1

S2

56

72 1. P(A) = 0,25 ; PA(B) = 0,7 et P–A(B) = 0,4.2.

0,7

0,25

0,75

0,3

0,4

0,6

BA

B

B

BA

3. P(–A) = 0,75 ; PA(

–B) = 0,3.

4. P(A B) = 0,25 × 0,7 = 0,175 et P(–A B) = 0,75 × 0,4 = 0,3.

P(B) = 0,175 + 0,3 = 0,475.

5. PB(A) = 0,1750, 475 ≈ 0,368.

73 1.

A–A Total

B 0,45 0,05 0,5–B 0,15 0,35 0,5

Total 0,6 0,4 1

2. PA(B) = 0, 450,6

= 0,75 ; P–A(B) = 0,050, 4

= 0,125.

3. 0,75

0,6

0,4

0,25

0,125

0,875

BA

B

B

BA

2. 0,25

0,2

0,3

0,5

0,75

0,4

0,6

23

13

VA

B

C

V

V

V

V

V

3. P(V) = 0,2 × 0,25 + 0,5 × 0,4 + 0,3 × 13

= 0,45.

4. PV(A) = 0,2 × 0,250, 45

= 19

≈ 0,11.

5. a. X suit une loi binomiale de paramètres 5 et 0,45.b. P(X = 2) ≈ 0,34 ; P(X 4) ≈ 0,87.c. P(X = 2) : probabilité que deux clients parmi les cinq choisis-sent le vin.P(X 4) : probabilité que moins de quatre clients parmi les cinq choisissent le vin.

71 1. P(A) = 0,6 ; P(B) = 0,7 et P(A B) = 0,5.2.

A–A Total

B 0,5 0,2 0,7–B 0,1 0,2 0,3

Total 0,6 0,4 1

3. P(A –B) = 0,1 et P(

–A B) = 0,2.

4. PB(–A) = 0,2

0,7= 2

7 ≈ 0,286 ; P–A(B) = 0,2

0, 4= 1

2 = 0,5.

57Chapitre 7 Loi normale

A Le programme

7CHAPITRE

Loi normale

Ce chapitre est consacré à la loi normale, premier exemple de loi continue.Au vu des connaissances des élèves en probabilités et en analyse, il n’était possible de définir cette loi qu’à partir de la courbe de sa fonction de densité.• Nous l’avons donc définie à partir de l’observation (dans l’activité 1) de la courbe « en cloche » qui approche les diagrammes en bâtons représentant une loi binomiale (étudiée en classe de Première) lorsque le paramètre n de la loi binomiale devient « très grand ». Ce choix a été motivé par le commentaire du programme et le chapitre suivant (chapitre 8), les intervalles de fluctuation d’une fréquence étant introduits à partir de l’approximation d’une loi binomiale par une loi normale de même espérance et d’écart-type np(1 – p).Par la suite, très peu d’exercices font intervenir une variable aléatoire suivant une loi binomiale remplacée par une variable suivant une loi normale, puisque les documents Ressources « Statistiques et probabilité pour la classe terminale générale et technologique » précisent que cette pratique ne doit être mise en œuvre que dans les séries STI2D-STL.• Dans l’activité 2, l’observation d’une série statistique permet de visualiser « la courbe en cloche » qui approche les diagrammes en bâtons des fréquences lorsque l’effectif de la série devient très grand, et d’introduire l’espérance et l’écart-type en lien avec la moyenne et l’écart-type de la série statistique.L’utilisation d’un logiciel de géométrie permet de faire une conjecture sur le calcul d’une probabilité comme aire d’un domaine sous la courbe « en cloche ».Le programme ne recommandant aucune méthode de calcul des probabilités P(X � k) et P(X � k) avec les calculatrices, nous avons choisi d’utiliser les approximations respectives P(– 1099 � X � k) et P(k � X � 1099). Les exercices 33 et 34 permettent cependant d’aborder le calcul de ces probabilités tel qu’il est réalisé dans les séries générales.

B Notre point de vue

Contenus Capacités attendues Commentaires

Loi normaleLoi normale d’espérance μ et d’écart-type σ.

• Utiliser une calculatrice ou un tableur pour calculer une probabilité dans le cadre d’une loi normale.

La loi normale peut être introduite à partir de l’observation, à l’aide d’un logiciel, de la loi binomiale.Les élèves doivent connaître l'allure de la courbe de densité, ainsi que sa symétrie. L'expression de la densité de la loi normale n'est pas un attendu du programme.Des exemples issus des autres disciplines montrent que la loi normale permet de modéliser des situations concrètes.

Intervalle de fluctuation d’une variable aléatoire suivant une loi normale.

• Connaître et interpréter graphiquement une valeur approchée de la probabilité de l’événement {X � [μ – 2σ, μ + 2σ]} lorsque X suit la loi normale d’espérance μ et d’écart-type σ.

On fait ainsi percevoir l’information apportée par la valeur de l’écart-type.Seul l’intervalle de fluctuation « 2σ » au seuil approximatif de 95 % est un attendu. L’intervalle « 1,96σ » ainsi que des exemples d’autres seuils peuvent être mentionnés.

58

Les exercices sont variés : calcul de probabilités avec calculatrice ou tableur, interprétation d’aires sous la « courbe d’une loi normale », détermination de l’intervalle de fluctuation « 2 σ », détermination du réel k tel que P(X � k) = a(a étant un réel donné entre 0 et 1)… les énoncés étant le plus souvent issus de situations concrètes.• Le TP est un problème permettant de sensibiliser les élèves au fait que toutes les variables aléatoires ne suivent pas une loi normale ! et qu’il existe des critères permettant d’en décider.Les sujets Cap vers le bac sont en général inspirés de sujets de BTS, modifiés afin d’être conformes aux objectifs du programme.

Les notions abordées dans le chapitre 7 • Loi normale d’espérance μ et d’écart-type σ• Intervalle de fluctuation « 2 σ »

C Avant de commencerVoir manuel page 254 et le site www.bordas-indice.fr pour les corrigés détaillés.

D ActivitésActivité Une loi binomiale qui devient

« normale »1

Fichiers associés sur www.bordas-indice.fr et sur le CD-Rom Professeur :07_TSTMG_activite1.xls (Excel 2003),07_TSTMG_activite1.xlsx (Excel 2007),07_TSTMG_activite1.ods (OpenOffice).Cette activité a pour objectif d’introduire la loi normale à partir de la loi binomiale.Les élèves ont étudié la loi binomiale de paramètres n et p en classe de Première. Ils connaissent l’espérance mathématique d’une variable aléatoire suivant une telle loi mais ne connaissent pas son écart-type. Nous nous limitons donc dans cette activité à faire observer la forme des diagrammes représentant une loi binomiale lorsque n devient « très grand » et à conjecturer le lien entre l’axe de symétrie des courbes « en cloche » qui se dessinent et l’espérance mathématique de la loi binomiale.1. a. On répète cinq fois de suite de manière indépendante la même expérience qui consiste à lancer un dé à quatre faces et à observer si on obtient la face « 3 ».La probabilité d’obtenir la face « 3 » étant égale à 1

4, la variable

aléatoire X correspondant au nombre de fois où on obtient la face « 3 » suit la loi binomiale de paramètres n = 5 et p = 0,25.b. E(X) = np = 5 × 0,25 = 1,25.2. Ouvrir le fichier 07_TSTMG_activite1.xls,07_TSTMG_activite1.xlsx ou 07_TSTMG_activite1.ods.a. Au bas de l’écran, cliquer gauche sur la feuille de 1 à 30 .– En présence de l’ascenseur : pour augmenter les valeurs

de n, cliquer gauche sur la flèche . Pour les diminuer, cliquer gauche sur la flèche .– En l’absence de l’ascenseur : entrer différentes valeurs de n(1 ; 2 ; 3 ; … ; 30) dans la cellule E2.

On observe que le diagramme prend la forme d’une cloche.b. Au bas de l’écran, cliquer gauche sur la feuille de 30 à 500

et comme dans la question a., faire varier les valeurs de n(30 ; 40 ; 50 ; … ; 500).On peut conjecturer que l’axe de symétrie de la courbe « en cloche » est la droite d’équation x = E(X).

Activité Quand un diagramme prend la forme d’une cloche

2

Fichiers associés sur www.bordas-indice.fr et sur le CD-Rom Professeur :07_TSTMG_activite2.xls (Excel 2003),07_TSTMG_activite2.xlsx (Excel 2007),07_TSTMG_activite2.ods (OpenOffice),07_TSTMG_activite2.ggb (GeoGebra).Cette activité a pour objectif d’introduire la loi normale et ses paramètres (espérance et écart-type) à partir de l’observation de séries statistiques.L’utilisation d’un tableur permet d’observer que lorsque l’effectif total devient très grand, le diagramme en bâtons des effectifs prend la forme d’une cloche et que la moyenne et l’écart-type se stabilisent. L’utilisation d’un logiciel de géométrie dynamique permet ensuite de comparer les probabilités calculées à partir des effectifs de la série et les aires sous la courbe « en cloche » qui ap-proche le diagramme des fréquences de la série.Comme nous approchons une série discrète par une loi continue, il fallait appliquer « la correction de continuité ». C’est pour cette raison que nous demandons de calculer la probabilité que la batterie ait une autonomie comprise entre 9,5 heures et 13,5 heures (et non entre 10 heures et 13 heures)…1. Ouvrir le fichier 07_TSTMG_activite2.xls,07_TSTMG_activite2.xlsx ou 07_TSTMG_activite2.ods.– Au bas de l’écran, cliquer gauche successivement sur les feuilles 25 batteries , 50 batteries , … 10 000 batteries .

59Chapitre 7 Loi normale

3. Ouvrir le fichier 07_TSTMG_activite2.ggb.a. Sélectionner le mode Déplacer, cliquer gauche sur les curseurs a et b pour les déplacer et les positionner sur les différentes valeurs de a et de b.Pour a = 9,5 et b = 13,5, l’aire affichée est 0,6677.Pour a = 8,5 et b = 11,5, l’aire affichée est 0,3612.Pour a = 14,5 et b = 16,5, l’aire affichée est 0,0934.Pour a = 0 et b = 24, l’aire affichée est 1.Ces aires sont très proches des valeurs des probabilités calculées dans la question 2.b. µ − 2σ = 8 et µ + 2σ = 16.Pour a = 8 et b = 16, l’aire affichée est 0,9545.

On observe que lorsque l’effectif devient très grand, le diagramme en bâtons des effectifs prend la forme d’une cloche et la moyenne et l’écart-type se stabilisent : la moyenne devient très proche de 12 et l’écart-type de 2.2. Au bas de l’écran, cliquer gauche sur la feuille

avec la « courbe en cloche » .La probabilité que l’autonomie (en heures) soit comprise entre :

a. 9,5 et 13,5 est 1 136 +1 759 + 2 007 +1 81310 000

, soit 0,6715 ;

b. 8,5 et 11,5 est 670 +1 136 +1 75910 000

, soit 0,3565 ;

c. 14,5 et 16,5 est 678 + 24810 000

, soit 0,0926 ;

d. 0 et 24 est 10 00010 000

, soit 1.

E Exercices14 Voir page 254 du manuel.15 1. µ − 2σ = 70 et µ + 2σ = 110.

2. I = [70 ; 110] .16 I = [49 ; 51] .17 1. µ − 2σ = 68 et µ + 2σ = 92.

2. À 0,01 près, l’aire de la partie colorée est 0,95.18 a = 900 et b = 1 100.

P O U R S ’ E N T R A Î N E R19 Voir page 254 du manuel.20 a. Faux. b. Faux.

c. Vrai. d. Vrai.e. Faux. f. Vrai.21 a. P(69 � X � 71) ≈ 0,0958 à 10−4 près.

b. P(X �74,5) ≈ 0,5442 à 10−4 près.c. P(X � 83) ≈ 0,9623 à 10−4 près.22 Voir page 254 du manuel.23 1. P(X � 26) est l’aire du domaine coloré sur le graphique

donc P(X � 26) ≈ 0,7475 à 10−4 près.2. a. À 10−4 près, l’aire sous la courbe de la partie non colorée est 1 – 0,7475, soit 0,2525.b. P(X � 26) ≈ 0,2525 à 10−4 près.3. Avec une calculatrice, on retrouve bien ce résultat.24 1. On peut en déduire les probabilités :

• P(X � 140) ≈ 0,3085 à 10−4 près ;• P(X � 170) ≈ 0,1587 à 10−4 près.2. a. L’aire sous la courbe de la partie non colorée est1 – (0,3085 + 0,1587), soit 0,5328.b. P(140 � X � 170) ≈ 0,5328 à 10−4 près.3. Avec une calculatrice, on retrouve bien ce résultat.25 a. P(X � 1 220) ≈ 0,7881.

b. P(X � 1 220) ≈ 1 – 0,7881, soit P(X � 1 220) ≈ 0,2119.c. P(X � 1 220) ≈ 0,2119.26 a. P(X � 450) ≈ 0,3121.

P O U R D É M A R R E R1 1. a. P(X � 25) = 0,5 b. P(X � 25) = 0,5

c. P(X � 25) = 0,5 d. P(X � 25) = 0,52 a. P(X � 3) = 0,5 b. P(X � 3) = 0,5

c. P(X � 3) = 0,5 d. P(X � 3) = 0,53 1. µ = 20 et σ = 3.

2. a. Le résultat affiché est 0,6997 à 10–4 près.b. Ce résultat correspond à P(15 � X � 22).4 1. µ = 120 et σ = 15.

2. a. Le résultat affiché est 0,1931 à 10–4 près.b. Ce résultat correspond à P(X � 107).5 1. µ = 40 et σ = 4.

2. a. Le résultat affiché est 0,8944 à 10–4 près.b. Ce résultat correspond à P(X � 35).6 Voir page 254 du manuel.7 1. P(X � 310) ≈ 0,7475 à 10−4 près

et P(X � 340) ≈ 0,9088 à 10−4 près.2. Avec une calculatrice, on retrouve bien ces résultats.8 1. P(−1099 � X � 38,5) ≈ 0,9599 à 10−4 près.

2. P(X � 38,5) ≈ 0,9599 à 10−4 près.9 1. P(680 � X � 1099) ≈ 0,7161 à 10−4 près.

2. P(X � 680) ≈ 0,7161 à 10−4 près.10 Fichiers associés sur www.bordas-indice.fr et sur le

CD-Rom Professeur :07_TSTMG_exercice10.xls (Excel 2003),07_TSTMG_ exercice10.xlsx (Excel 2007),07_TSTMG_ exercice10.ods (OpenOffice).1. a. La formule saisie dans la cellule B2 est

=LOI.NORMALE(B1;30;3;1) . b. µ = 30 et σ = 3.2. P(X � 27) ≈ 0,1587 à 10−4 prèset P(X � 29) ≈ 0,3694 à 10−4 près.11 P(100 − 2 × 5 � X � 100 + 2 × 5) ≈ 0,95 à 10−2 près.12 P(20 − 2 × 3 � X � 20 + 2 × 3) ≈ 0,95 à 10−2 près.13 1. µ − 2σ = 61 et µ + 2σ = 69.

2. P(61 � X � 69) ≈ 0,95 à 10−2 près.

60

b. B prend la valeur P(X � 11,5), soit 0,3694.c. La valeur affichée en sortie est 0,3694 – 0,0912, soit 0,2782.2. Cet algorithme affiche P(a � X � b), a et b ayant été entrés par l’utilisateur.33 1. a. et b.

10 12 14 16 18 20 22c. P(X � 16) = 0,5.2. a.

10 12 14 16 18 20 22b. P(X � 18) = P(X � 16) + P(16 � X � 18) doncP(X � 18) = 0,5 + P(16 � X � 18).c. P(16 � X � 18) ≈ 0,3413 à 10−4 près.P(X � 18) ≈ 0,5 + 0,3413 , soit P(X � 18) ≈ 0,8413 à 10−4 près.3. a.

10 12 14 16 18 20 22

P(X � 15) est l’aire du domaine coloré sur le graphique ci-dessus : cette aire est la différence entre l’aire du domaine hachuré et l’aire du domaine hachuré donc :P(X � 15) = P(X � 16) – P(15 � X � 16), soitP(X � 15) = 0,5 – P(15 � X � 16).b. P(15 � X � 16) ≈ 0,1915 à 10−4 près.P(X � 15) ≈ 0,5 – 0,1915, soit P(X � 15) ≈ 0,3085 à 10−4 près.34 1. P(X � 34) = 0,5.

2. a. P(X � 31) = P(31 � X � 34) + P(X � 34)donc P(X � 31) = 0,5 + P(31 � X � 34).b. P(31 � X � 34) ≈ 0,2257 à 10−4 près.P(X � 31) ≈ 0,5 + 0,2257, soit P(X � 31) ≈ 0,7257 à 10−4 près.3. a.

34 38 42 4630262218 50P(X � 38) est l’aire du domaine coloré sur le graphique ci-dessus : cette aire est la différence entre l’aire du domaine hachuré et l’aire du domaine hachuré donc :

P(X � 38) = 0,5 – P(34 � X � 38).b. P(34 � X � 38) ≈ 0,2881 à 10−4 près.P(X � 38) ≈ 0,5 – 0,2881, soit P(X � 38) ≈ 0,2119 à 10−4 près.35 Vrai : avec la calculatrice, on obtient bien :

P(19 � X � 27,3) ≈ 0,78 à 0,01 près.

b. P(X � 450) ≈ 1 – 0,3121, soit P(X � 450) ≈ 0,6879.c. P(X � 450) ≈ 0,6879.27 Voir page 254 du manuel.28 Les valeurs affichées sont celles de P(X � 195), P(X � 196),

…, P(X � 200), soit dans l’ordre (à 10−4 près) : 0,4207 ; 0,4364 ; 0,4522 ; 0,4681 ; 0,4840 ; 0,5.Fichier associé sur www.bordas-indice.fr et sur le CD-Rom Professeur :07_TSTMG_exercice28.alg (AlgoBox).

29 Fichiers associés sur www.bordas-indice.fr et sur le CD-Rom Professeur :07_TSTMG_exercice29.xls (Excel 2003),07_TSTMG_ exercice29.xlsx (Excel 2007),07_TSTMG_ exercice29.ods (OpenOffice),07_TSTMG_exercice29_correction.xls (Excel 2003),07_TSTMG_exercice29_correction.xlsx (Excel 2007),07_TSTMG_exercice29_correction.ods (OpenOffice).1. a. On peut saisir =LOI.NORMALE(B2;700;50;1) .b. On peut saisir =B4-B3 .c. On peut saisir =1-B3 .2. a. et b.

30 Fichiers associés sur www.bordas-indice.fr et sur le CD-Rom Professeur :07_TSTMG_exercice30.xls (Excel 2003),07_TSTMG_ exercice30.xlsx (Excel 2007),07_TSTMG_ exercice30.ods (OpenOffice).1. a. La valeur affichée est 0,8043 – 0,2839, soit 0,5204.b. Cette valeur correspond à P(36 � X � 46).2. a. La valeur affichée est 1 – 0,2839, soit 0,7161.b. Cette valeur correspond à P(X � 36).31 Exercice résolu.32 1. a. A prend la valeur P(X � 10), soit 0,0912.

61Chapitre 7 Loi normale

b. μ = 11 et σ = 4.2. a. Le réel k tel que P(X � k) = 0,3 est 8,9 à 0,01 près.b. On peut estimer que 30 % des candidats ont obtenu une moyenne inférieure à 8,9.3. a. P(X � b) = 0,6.b. Le réel b tel que P(X � b) = 0,4 est 12,01, à 0,01 près.c. On peut estimer que 40 % des candidats ont obtenu une moyenne supérieure à 12,01.49 Exercice résolu.50 1. a. k ≈ 2,5 à 0,1 près.

b. 25 % des kangourous n’ont pas dépassé 2,5 mètres.2. a. P(X � 3,9) ≈ 0,10.b. 10 % des kangourous ont dépassé 3,9 mètres.51 1. a. k ≈ 89,9 à 0,1 près.

b. Le quart des individus a un QI inférieur à 89,9.2. a. P(X � b) = 0,95.b. b ≈ 124,7 à 10–1 près.c. 5 % des individus ont un QI supérieur à 124,7.52 Vrai : avec la calculatrice, on obtient P(X � 1) ≈ 0,06 à

0,01 près.53 Faux : P(X � 1) ≈ 0,94 à 0,01 près, environ 94 % des poulets

se vendront bien.54 Vrai : P(X � 1,21) ≈ 0,20 à 0,01 près, environ 20 % des

poulets pèsent moins de 1,21 kg.55 Vrai : P(X � 1,6) ≈ 0,68 à 0,01 près, environ 68 % des

poulets pèsent moins de 1,6 kg.56 I = [21 ; 23].57 I = [98,4 ; 101,6].58 Voir manuel page 254.59 σ = 5.60 1. La courbe étant symétrique par rapport à la droite

d’équation x = 40,

P(X � 35) = P(X � 45) = 1− P(35 < X < 45)2

≈ 0,052

.

P(X � 35) ≈ 0,02 et P(X � 45) ≈ 0,02 à 0,01 près par défaut.2. 2σ = 5 donc σ = 2,5.61 1. I = [47,4 ; 52,6].

2. σ = 0,45.62 Voir manuel p. 254.63 1. La probabilité qu’une face soit conforme est :

P(129 � L1 � 131), soit 0,85 à 10–2 près.2. σ = 0,5.64 Vrai : P(90 � X � 130) = P(110 – 2 × 10 � X � 110 + 2 × 10)

donc P(90 � X � 130) ≈ 0,95 à 0,01 près.

65 Faux : P(X � 90) ≈ 0,052

donc P(X � 90) ≈ 0,02 à 0,01 près par défaut.66 Vrai : [90 ; 130] = [µ − 2σ ; µ + 2σ] donc [90 ; 130] contient

environ 95 % des valeurs prises par X.67 Faux : [100 ; 120] est inclus dans [90 ; 130] qui contient

environ 95 % des valeurs prises par X donc [100 ; 120] contient moins de 95 % des valeurs prises par X.68 Vrai : [92 ; 128] est inclus dans [90 ; 130] qui contient

environ 95 % des valeurs prises par X donc [92 ; 128] contient moins de 95 % des valeurs prises par X.

36 Vrai : comme 26,2 est l’espérance de la loi de X,P(X � 26,2) = P(X � 26,2) = 0,5.37 Vrai : P(X � 27) = P(X � 26,2) + P(26,2 � X � 27)

= 0,5 + P(26,2 � X � 27).38 Faux : P(X � 27) = 0,5 – P(26,2 � X � 27)

et P(26,2 � X � 27) ≠ 0 donc P(X � 27) � 0,5.39 Vrai : P(X � 23) = 1 – P(X � 23) = 1 – P(X � 23).40 Vrai : la symétrie de la courbe de la loi de X par rapport à

la droite d’équation x = 26,2 entraîne :P(X � 25) = P(X � 27,4).P(X � 25) = P(X � 27,4) ≈ 0,1957 à 10−4 près.41 1. a. P(X � 242) ≈ 0,0912 à 10−4 près.

b. Environ 91 sachets ont une masse inférieure à 242 mg.2. a. Cette probabilité est P(239 � X � 261), soit 0,9332 à 10−4

près.b. 1 – 0,9332 = 0,0668. La probabilité est 0,0668 à 10−4 près.42 1. P(Y � 1 200) ) ≈ 0,8849 à 10−4 près. La probabilité est

0,8849 à 10−4 près.2. a. Cette probabilité est P(600 � Y � 1 500), soit 0,7742 à 10−4 près.b. 1 – 0,7742 = 0,2258. Cette probabilité est 0,2258 à 10−4 près.43 Voir page 254 du manuel.44 1. a. Cette probabilité est P(T � 76), soit 0,0044 à 10−4 près.

b. Cette probabilité est P(T � 124), soit 0,0146 à 10−4 près.c. Cette probabilité est P(T � 76) + P(T � 124), soit 0,019 à 10−4 près.2.

Taille US Taille en chiffre Proportion d’hommesXS ou S 1 ou 2 8,09 %

M 3 23,38 %

L 4 34,72 %

XL ou XXL 5 ou 6 31,90 %

45 Exercice résolu.46 1. a. On répète 6 fois de manière identique et

indépendante l’épreuve qui consiste à faire match nul ou non.

La probabilité qu’il y ait match nul étant égale à 13 , la variable

aléatoire X comptant le nombre de matchs nuls suit la loi

binomiale de paramètres n = 6 et p = 13

.

b. P(X = 4), soit 0,0823 à 10−4 près.2. a. Cette probabilité est P(Y � 30), soit 0,0668 à 10−4 près.b. Cette probabilité est P(Y � 10), soit 0,0002 à 10−4 près.47 1. a. Cette probabilité est P(15 � X � 21), soit 0,6827 à

10−4 près.b. Cette probabilité est P(X � 25), soit 0,0098 à 10−4 près.2. P(X � 20) ≈ 0,2525 à 10−4 près.La probabilité est 0,2525 à 10−4 près.48 Fichiers associés sur www.bordas-indice.fr et sur le

CD-Rom Professeur :07_TSTMG_exercice48.xls (Excel 2003),07_TSTMG_ exercice48.xlsx (Excel 2007),07_TSTMG_ exercice48.ods (OpenOffice).1. a. =LOI.NORMALE.INVERSE(B1;11;4) .

62

Modifier les Étiquettes de l’axe horizontal (entrer la plage de cellules A2:A12) et supprimer Pointure dans Entrées de légende.Avec OpenOffice : Choisir l’icône Diagramme puis Colonne .Dans l’étape 2 de l’Assistant de diagramme, cocherPremière colonne comme étiquette puis Terminer .

Le critère (2) est vérifié.2. a. Les cellules G1 et G2 ont été respectivement nommées m et s.Cela permet d’écrire dans les formules m à la place de $G$1 et s à la place de $G$2.Dans la cellule G5, on peut saisir la formule =m-G4*s .

b. Dans la cellule G6, on peut saisir la formule =m+G4*s .

c. Saisir dans la cellule G5 la formule =m-G4*s et dans la cellule G6 la formule =m+G4*s .Recopier ces cellules vers la droite jusqu’à la colonne I.3. a. La formule dans la cellule C2 est

=SI(ET($A2<=G$5;$A2<=G$6);$B2;0) .b. Si le contenu de la cellule A2 est compris entre la valeur contenue dans G5 (c’est-à-dire m – s) et la valeur contenue dans G6 (c’est-à-dire m + s), la valeur affichée dans la cellule C2 est celle contenue dans B2, sinon elle est égale à 0. Recopiée vers le bas, cette formule permet d’afficher les ventes pour les pointures appartenant à I1 (et 0 pour les pointures n’appartenant pas à I1).c. Recopier la cellule C2 vers le bas jusqu’à la cellule C12, puis recopier la zone de cellules C2:C12 vers la droite jusqu’à la colonne E.4. Saisir dans la cellule C14 la formule = SOMME(C2:C12) .Mettre la zone de cellules C15:E15 au format pourcentage. Saisir dans la cellule C15 la formule = C14/$B14 .Recopier les cellules C14 et C15 vers la droite jusqu’à la colon-ne E.5.

Le critère (3) n’est pas vérifié : on ne peut pas approcher cette série par une loi normale.

69 a. P(90 � X � 100) ≈ 0,587 à 10–3 près.b. P(X � 96) ≈ 0,552 à 10–3 près.c. P(X � 80) ≈ 0,994 à 10–3 près.70 1. P(58,5 � Z � 59,5) ≈ 0,678 à 10–3 près.

2. La probabilité qu’une machine soit jugée inutilisable est P(Z � 60), soit 0,024 à 10–3 près.71 1. I = [83 ; 127].

2. a. P(83 � X � 127) ≈ 0,95 à 0,01 près.b. P(X � 83) ≈ 0,02 à 0,01 près par défaut.c. P(X � 127) ≈ 0,02 à 0,01 près par défaut.72 σ = 0,1.

P O U R FA I R E L E P O I N TVoir manuel page 254. Les corrigés détaillés sont sur le site www.bordas-indice.fr.

ACCO M PAG N E M E N T P E R S O N N A L I S E73 P(72 � X � 75) ≈ 0,1859 à 10–4 près.

P(X � 81) ≈ 0,9861 à 10–4 près.P(X � 68) ≈ 0,6554 à 10–4 près.74 P(847 � X � 962) ≈ 0,5751 à 10–4 près.

P(X � 1 000) ≈ 0,8413 à 10–4 près.P(X � 880) ≈ 0,9192 à 10–4 près.75 P(11,5 � X � 11,9) ≈ 0,1587 à 10–4 près.

P(X � 11,7) ≈ 0,0013 à 10–4 près.P(X � 12,1) ≈ 0,1587 à 10–4 près.76 I = [7 ; 13]. 77 I = [98 ; 202].

T R AVAU X P R AT I Q U E STP Modèle « normal » de gestion de stock

Fichiers associés sur www.bordas-indice.fr et sur le CD-Rom Professeur :07_TSTMG_TP.xls (Excel 2003),07_TSTMG_TP.xlsx (Excel 2007),07_TSTMG_TP.ods (OpenOffice),07_TSTMG_TP_correction.xls (Excel 2003),07_TSTMG_TP_correction.xlsx (Excel 2007),07_TSTMG_TP_correction.ods (OpenOffice).A. Critère (1) avec une calculatriceLa moyenne est égale à 40,075 et la médiane est égale à 40 : le critère (1) est vérifié.B. Autres critères avec un tableur1. – Ouvrir le fichier 07_TSTMG_TP.xls,07_TSTMG_TP.xlsx ou 07_TSTMG_TP.ods.– Sélectionner la plage de cellules A1:B12.Avec Excel : Choisir Insertion , Colonne et Histogramme groupé .Cliquer droit sur le diagramme et choisirSélectionner des données… .

63Chapitre 7 Loi normale

07_TSTMG_ exercice80.ods (OpenOffice),07_TSTMG_exercice80_correction.xls (Excel 2003),07_TSTMG_exercice80_correction.xlsx (Excel 2007),07_TSTMG_exercice80_correction.ods (OpenOffice).1. a. D’après la partie A,P(163 – h � T � 163 + h) = 1 – 2P(T � 163 – h).P(163 – h � T � 163 + h) = p doncp = 1 – 2P(T � 163 – h) et P(T �163 – h) =

1− p2

.

b. P(163 – h � T � 163 + h) = 1 – 2P(T � 163 – h)= 1 – 2P(T � 163 + h).

P(163 – h � T � 163 + h) = p doncp = 1 – 2P(T � 163 + h) doncp = 1 – 2(1 – P(T � 163 + h)) et P(T � 163 + h) =

1+ p2

.

2. a. Elle affiche le réel k tel que P(T � k) = 1− p

2 , p étant la valeur contenue dans la cellule B3.b. On peut saisir dans la cellule B5 la formule

=LOI.NORMALE.INVERSE((1+B3)/2;$B$1;$E$1) .c. Saisir dans la cellule B5 la formule donnée ci-dessus et recopier les cellules B4 et B5 jusqu’à la colonne J.

C A P V E R S L E B AC

Zoom sur le tableur

Fichiers associés sur www.bordas-indice.fr et sur le CD-Rom Professeur :07_TSTMG_zoomtableur.xls (Excel 2003),07_TSTMG_zoomtableur.xlsx (Excel 2007),07_TSTMG_ zoomtableur.ods (OpenOffice).

Sujet A1. La moyenne est 17 degrés à 0,1 près.L’écart-type est 1,5 degrés à 0,1 près.2. a. Cette probabilité est P(X � 18), soit 0,2525 à 10–4 près.b. Cette probabilité est P(15 � X � 18), soit 0,6563 à 10–4 près.c. Cette probabilité est P(X � 15), soit 0,0912 à 10–4 près.3. a. La valeur affectée à la variable A est P(X � 15), soit 0,0912 à 10–4 près.La valeur affectée à la variable B est P(X � 18), soit 0,7475 à 10–4 près.b. Cette valeur est P(X � 18) – P(X � 15), soit P(15 � X � 18). Cela correspond à la probabilité que la maison soit dans la catégorie 2.c. On remplace la dernière ligne de l’algorithme par :« Afficher 1 – B ».

P O U R A P P R O F O N D I R78 1. Cette probabilité est P(Z � 90), soit 0,91 à 10−2 près.

2. a. • Pour n = 5 : S5 suit la loi normale d’espérance 350 et d’écart-type 15 5 . p5 = P(S5 � 500), soit 4 × 10–6 à 10–6 près.• Pour n = 6 : S6 suit la loi normale d’espérance 420 et d’écart-type 15 6 . p6 = P(S6 � 500), soit 0,015 à 10–3 près.b. La probabilité Sn de surcharge dépasse 0,0001 pour n = 6 : le nombre maximal de personnes autorisées à emprunter l’ascenseur est n = 5.79 1. a. Cette probabilité est P(468 � X � 520), soit 0,95 à

0,01 près.b. P(468 � X � 500) ≈ 0,50 à 0,01 près.c. P(500 � X � 520) ≈ 0,45 à 0,01 près.2. a. k1 ≈ 479 à l’unité près.b. P(X � k2) = 0,01 donc P(X � k2) = 0,99.Avec la calculatrice, on obtient : k2 ≈ 528 à l’unité près.c. Les masses appartiennent à l’intervalle [479 ; 528].3. a. • Pour le premier critère, d’après la question 1. a., environ 95 % des bocaux sont bien remplis.• Pour le second critère, 5 % des bocaux ne sont pas bien remplis : les 4 % les moins remplis et les 1 % les plus remplis. Donc 95 % des bocaux sont bien remplis.L’affirmation A1 est vraie pour les deux critères.• Pour le premier critère, d’après les questions 1. b. et 1. c., P(468 � X � 500) ≠ P(500 � X � 520).• Pour le second critère,P(479 � X � 500) ≈ 0,5 – 0,04, soit 0,46 à 0,01 près etP(500 � X � 528) ≈ 0,5 – 0,01, soit 0,49 à 0,01 prèsdonc P(479 � X � 500) ≠ P(500 � X � 528).L’affirmation A2 est fausse pour les deux critères.b. L’intervalle de centre 500 contenant environ 95 % des valeurs prises par X est [500 – 2 × 12 ; 500 + 2 × 12].Critère vérifiant les affirmations A1 et A2 : « un bocal est bien rempli s’il contient entre 476 grammes et 524 grammes d’abricots ».80 A. Avec une calculatrice

1. La courbe de la loi de T est symétrique par rapport à la droite d’équation x = 163 donc :P(T � 163 – h) = P(T � 163 + h).P(163 – h � T � 163 + h) = 1 – P(T � 163 – h) – P(T � 163 + h).Comme P(T � 163 – h) = P(T � 163 + h),P(163 – h � T � 163 + h) = 1 – 2P(T � 163 – h).2. a. P(163 – h � T � 163 + h) = 0,68donc 0,68 = 1 – 2P(T � 163 – h) et P(T �163 – h) = 0,16.b. Avec la calculatrice, on obtient 163 – h ≈ 159 à 0,1 près, donc h ≈ 4. On constate que h = σ.3. 163 + h = 167. Environ 68 % des femmes françaises mesurent entre 159 cm et 167 cm.B. Avec un tableurFichiers associés sur www.bordas-indice.fr et sur le CD-Rom Professeur :07_TSTMG_exercice80.xls (Excel 2003),07_TSTMG_ exercice80.xlsx (Excel 2007),

64

2. a. Cette probabilité est P(S � 35), soit 0,04 à 0,01 près.b. Cette probabilité est P(S � 40), soit 0,81 à 0,01 près.

Sujet EFichiers associés sur www.bordas-indice.fr et sur le CD-Rom Professeur :07_TSTMG_sujetE.xls (Excel 2003),07_TSTMG_ sujetE.xlsx (Excel 2007),07_TSTMG_ sujetE.ods (OpenOffice).Partie A1.

9,90 9,92 9,94 9,96 9,95 10,00 10,02 10,04 10,06 10,080

5

10

15

20

25

30

2. La moyenne est 9,999 mètres à 10–3 près.L’écart-type est 0,03 mètre à 10–3 près.Partie Ba. La probabilité qu’un rouleau ait une longueur comprise entre 9,95 mètres et 10,05 mètres est P(9,95 � X � 10,05), soit 0,9044 à 10−4 près.b. La probabilité qu’un rouleau ait une longueur inférieure à 9,95 mètres est P(X � 9,95), soit 0,0478 à 10−4 près.c. La probabilité qu’un rouleau ait une longueur supérieure à 10,05 mètres est P(X � 10,05), soit 0,0478 à 10−4 près.Partie C1. On peut saisir dans la cellule A3 la formule =A2+0,02 .

2. a. La formule saisie dans la cellule E2 est =B3 .b. La valeur affichée dans la cellule E2 est celle contenue dans la cellule B3, soit 0,0478 .3. On peut saisir dans la cellule E3 la formule =B8-B3 .

4. a. On peut saisir dans la cellule E4 la formule =1-B8 .

b. La valeur affichée dans la cellule E4 est 0,0478 .

Sujet F1. Réponse b.2. Réponse a.3. Réponse c.4. Réponse c.5. Réponse a.6. Réponse b.

Sujet GFichiers associés sur www.bordas-indice.fr et sur le CD-Rom Professeur :07_TSTMG_sujetG.xls (Excel 2003),

Sujet B1. a. Cette probabilité est P(56,2 � X � 57,2), soit 0,9044 à 10–4

près.b. Cette probabilité est 1 – P(56,2 � X � 57,2), soit 0,0956 à 10–4 près.2. a.

c prend la valeur m – 2sd prend la valeur m + 2s

b. I = [56,1 ; 57,3].Les plateaux dont la largeur appartient à I ne sont pas tous conformes puisque leur largeur peut être inférieure à 56,2 cm ou supérieure à 57,2 cm.3. σ = 0,25.

Sujet CA. Avec un arbre pondéré1.

0,007

0,4

0,6

0,993

0,012

0,988

DA

B

D

D

D

2. P(A � D) = 0,4 × 0,007 = 0,0028.P(B � D) = 0,6 × 0,012 = 0,0072.3. P(D) = P(A � D) + P(B � D) = 0,01. La probabilité qu’une pièce présente un défaut de surface est 0,01.B. Avec une loi binomiale1. On renouvelle 20 fois de manière indépendante la même épreuve à deux issues consistant à prélever une pièce et à observer si elle a un défaut de surface ou non. La probabilité que la pièce ait un défaut de surface étant égale à 0,01, la variable aléatoire X suit la loi binomiale de paramètres n = 20 et p = 0,01.2. P(X = 2) ≈ 0,0159 à 10−4 près.La probabilité que dans le lot il y ait exactement deux pièces ayant un défaut de surface est 0,0159 à 10−4 près.3. P(X � 1) ≈ 0,9831 à 10−4 près.C. Avec une loi normale1. Cette probabilité est P(Y � 10), soit 0,3809 à 10−4 près.2. Cette probabilité est P(Y � 20), soit 0,0032 à 10−4 près.

Sujet D1. a. On renouvelle 15 fois de manière indépendante la même épreuve à deux issues consistant à choisir le nom d’une personne de l’entreprise et à noter s’il s’agit d’une personne ayant suivi le stage ou non. La probabilité qu’une personne ait suivi le stage étant égale à 0,3, la variable aléatoire X suit la loi binomiale de paramètres n = 15 et p = 0,3.b. Cette probabilité est P(X = 5), soit 0,21 à 0,01 près.c. Cette probabilité est P(X � 1), soit 0,04 à 0,01 près.

65Chapitre 7 Loi normale

2. Cette probabilité est P(A).P(A) = P(H � A) + P(F � A) = 0,3 × 0,9212 + 0,7 × 0,1276 = 0,36568.

81 1. a. P(700 � X � 720) ≈ 0,17 à 0,01 près.b. P(X � 750) ≈ 0,75 à 0,01 près.c. P(X � 705) ≈ 0,63 à 0,01 près.2. I = [630 ; 810] .82 1. Cette probabilité est P(1,4 � X � 1,6), soit 0,85 à 0,01

près.2. Cette probabilité est 1 – P(1,4 � X � 1,6), soit 0,15 à 0,01 près.3. L’écart-type aurait dû être égal à 0,05.83 1. P(Y � 104) ≈ 0,98 à 10–2 près.

2. a. La probabilité qu’un sachet soit rejeté est1 – P(104 � X � 136), soit 0,05 à 10–2 près.b. 1 200 × 0,05 = 60. Dans un lot de 1 200 sachets, environ 60 sachets sont rejetés.

07_TSTMG_ sujetG.xlsx (Excel 2007),07_TSTMG_ sujetG.ods (OpenOffice).A. Avec un tableur1. a. =LOI.NORMALE(B4;C2;C3;1) .b. Les réponses justes sont :p1 = P(X � 40 000) et p4 = P(X � 40 000).c. =1-C4 .

d. 0,9212 .2. a. On peut saisir dans la cellule D4 la formule =LOI.NORMALE(B4;D2;D3;1)

et dans la cellule D5 la formule =1-D4 .

b. La valeur affichée dans D4 est 0,8724 .

Celle affichée dans D5 est 0,1276 .B. Avec un arbre pondéré1. 0,9212

0,3

0,7

0,0788

0,1276

0,8724

AH

A

A

AH

66

Ce chapitre traite les parties « Échantillonnage » et « Estimation » du programme de probabilités de la classe. L’intervalle de fluctuation asymptotique est donné sans démonstration en classe de Terminale STMG : on découvre cet intervalle sur un exemple, à partir de la loi binomiale, dans la première activité. Conformément au programme, cet intervalle de fluctuation n’est donné dans le cours qu’au seuil de confiance 0,95. Nous avons utilisé le mot « asymptotique », alors qu’il n’est pas indiqué dans le programme, afin de le différencier de l’intervalle de fluctuation vu en classe de Première avec la loi binomiale.La seconde activité proposée introduit la notion d’intervalle de confiance sur un exemple, en utilisant l’outil tableur, comme l’indique le programme.Les exercices sont pour la plupart élémentaires : les élèves doivent surtout savoir déterminer un intervalle de fluctuation et un intervalle de confiance. L’accompagnement personnalisé du chapitre revient d’ailleurs sur ces deux points. Le TP revient sur l’élection présidentielle de 2002 : l’utilisation du tableur et des notions du chapitre permet de mieux comprendre « l’erreur » des sondages de cette année-là.

Les notions abordées dans le chapitre 8 • Échantillonnage• Prise de décision sur un échantillon• Estimation par intervalle d’une proportion

B Notre point de vue

Objectif : Consolider les connaissances acquises dans le domaine de l’échantillonnage et aborder l’estimation par la détermination d’un intervalle de confiance pour une proportion.

Contenus Capacités attendues Commentaires

Échantillonnage et prise de décisionIntervalle de fluctuation d’une fréquence.

Prise de décision.

• Connaître un intervalle de fluctuation à au moins 95 % d’une fréquence d’un échan-tillon de taille n :

p – 1n

, p + 1n

⎡⎣⎢

⎤⎦⎥

lorsque la proportion p dans la population est connue.

• Exploiter un tel intervalle pour rejeter ou non une hypothèse sur une proportion.

On peut faire observer qu’en approchant la loi binomiale par la loi normale de même espérance et d’écart-type p(1 – p)n , on est conduit à l’intervalle

p –1,96 p(1 – p)n

, p +1,96 p(1 – p)n

⎡⎣⎢

⎤⎦⎥

qui est inclus dans p – 1n

, p + 1n

⎡⎣⎢

⎤⎦⎥

.

Le vocabulaire des tests (test d’hypothèse, hypothèse nulle, risque de première espèce) est hors programme.

EstimationIntervalle de confiance d’une proportion.

• Estimer une proportion inconnue par l’intervalle :

f – 1n

, f + 1n

⎡⎣⎢

⎤⎦⎥

où f est la fréquence obtenue sur un échan-tillon de taille n.

Cet intervalle contient la proportion dans au moins 95 % des cas pour n grand, ce qui peut être illustré par simulation.La notion de niveau de confiance ne fait pas l’objet de développements.

Le programmeA

8CHAPITRE

Échantillonnage et estimation

67Chapitre 8 Échantillonnage et estimation

C Avant de commencerLes notions abordées dans ces exercices permettent de réactiver les notions utiles pour ce chapitre, en particulier celles sur la loi binomiale déjà abordées en classe de Première STMG, et celles concernant la loi normale vue dans le chapitre précédent.Voir manuel page 254 et le site www.bordas-indice.fr pour les corrections détaillées.

D ActivitésActivité Les joueurs de pétanque1

Cette activité a pour but d’introduire la notion d’intervalle de fluctuation asymptotique dans le cadre d’un exemple.Pour cela, on utilise la loi binomiale, puis une approximation de cette loi binomiale par une loi normale : ce travail est d’abord effectué dans la partie A sur des valeurs numériques, avec un échantillon de taille 400, puis dans la partie B avec un échantillon de taille n. Les difficultés techniques de la partie B pourront être diminuées en faisant le parallèle avec les questions de la partie A. Pour conclure dans chacune des parties, il n’est pas nécessaire de calculer avec la loi normale, car on utilise l’intervalle « 2σ ».Fichier associé sur le site www.bordas-indice.fr et sur le CD-Rom Professeur : 08_TSTMG_activite1.ggb (GeoGebra).Partie A1. L’épreuve consistant à choisir une personne de 15 ans ou plus dans la population française se répète 400 fois de façon identique et indépendante, et elle conduit à deux issues : « la personne joue à la pétanque », avec la probabilité 0,1, et « la personne ne joue pas à la pétanque », avec la probabilité 0,9.Ainsi, la variable aléatoire X comptant le nombre de joueurs de pétanque dans cet échantillon suit la loi binomiale de paramètres n = 400 et p = 0,1.Son espérance mathématique est : E(X) = 40.2. μ = n × p = 40.σ = n × p × (1– p) = 36 = 6 .3. a. C’est un intervalle « 2σ », doncP(Y [μ - 2σ ; μ + 2σ]) ≈ 0,95.b. μ – 2σ = 28 et μ + 2σ = 52.

4. F = 1400 Y.

P(0,07 F 0,13) = P(28 Y 52) ≈ 0,95 en utilisant la question 3.Partie B1. X suit la loi binomiale de paramètres n et p = 0,1 donc E(X) = 0,1 × n.2. μ = np = 0,1n.σ = np(1– p) = 0,1× 0,9n = 0,09n = 0,3 n .3. P(Y [0,1n – 0,6 n ; 0,1n + 0,6 n ]) = P(Y [μ – 2σ ; μ + 2σ]) ≈ 0,95.4. a. P(F J) = P(0,1n – 0,6 n Y 0,1n + 0,6 n ) ≈ 0,95.

b. J est inclus dans I car 0,6n

< 1n

.

Donc : P(F I) P(F J) et ainsi la probabilité que F appartienne à I est au moins 0,95.

c. L’intervalle I est un intervalle de fluctuation de F avec un seuil de confiance d’au moins 95 %.

Activité Estimation de la proportion de myopes

2

Cette activité permet de découvrir la notion d’intervalle de confiance.Pour cela, on suppose connue la proportion p, alors que dans le problème de l’estimation, on ne connaît pas p et on veut l’estimer.

On forme alors des intervalles de la forme f − 1n

; f + 1n

⎡⎣⎢

⎤⎦⎥ , où

f est la fréquence observée dans un échantillon de taille n, et onexamine combien de ces intervalles contiennent p. On va se rendre compte en multipliant les simulations avec le tableur qu’il y en a environ 95 %, ce qui permet de définir la notion d’intervalle de confiance.Fichiers associés sur le site www.bordas-indice.fr et sur le CD-Rom Professeur :08_TSTMG_activite2.xlsx (Excel 2007),08_TSTMG_activite2.xls (Excel 2003),08_TSTMG_activite2.ods (OpenOffice).1. a. Cette formule simule le choix d’un myope ou d’un « non-myope » dans la population.Si le résultat est 1, on a choisi une personne myope, et si le résultat est 0, on a choisi une personne non-myope.b. La fréquence de myopes dans cet échantillon de taille 100 est égale au quotient du nombre de myopes par 100 : le nombre de myopes étant donné par le nombre de « 1 », il suffit de faire la somme de ces « 1 » pour l’obtenir, ce qui justifie la formule.2. Correctif : la formule en A106 est

=SI(ET(A104<=0,39;A103>=0,39);1;0)

et non pas =SI(ET(A104<=0,39;A105>=0,39);1;0)

La formule =SI(ET(A104<=0,39;A103>=0,39);1;0) renvoie 1

lorsque f + 1n

0,39 et f – 1n

0,39, c’est-à-dire lorsque

0,39 f − 1n

; f + 1n

⎡⎣⎢

⎤⎦⎥ , ou dit autrement lorsque la pro-

portion p appartient à l’intervalle f − 1n

; f + 1n

⎡⎣⎢

⎤⎦⎥ .

Pour chacun des échantillons formés en colonne, on obtient

ainsi 1 si p appartient à f − 1n

; f + 1n

⎡⎣⎢

⎤⎦⎥ et 0 sinon.

En faisant la somme de tous ces nombres, on obtient le nombre de « 1 » à partir de ces 100 échantillons : on obtient ainsi le nombre d’échantillons tels que l’intervalle f − 1

n; f + 1

n⎡⎣⎢

⎤⎦⎥

contient la proportion p.

68

b. On constate la même chose qu’à la question précédente, mais avec des intervalles d’amplitude plus petite.c. Tous ces intervalles ne contiennent pas p : il suffit pour cela d’observer les intervalles pour lesquels on a obtenu « 0 » dans la ligne 106.

La formule saisie en G108 divise ce nombre par 100 : on obtientdonc la fréquence des intervalles de la forme f − 1

n; f + 1

n⎡⎣⎢

⎤⎦⎥

contenant p.3. a. On constate que la fréquence des intervalles contenant pest dans la plupart des cas supérieure ou égale à 0,95.

E Exercices21 1. f = 40

50 = 0,8.

2. Intervalle de confiance pour p : [0,65 ; 0,95].22 Fréquence observée : f = 54

400 = 0,135.

Intervalle de confiance pour p : [0,085 ; 0,185].23 1. f = 114

150 = 0,76.

2. Intervalle de confiance pour p : [0,67 ; 0,85].

P o u r s ’ e n t r a î n e r24 1. [0,03 ; 0,23].

2. Environ 250.25 1. [0,503 ; 0,843].

2. Environ 100 classes.26 a. [0,12 ; 0,32].

b. [0,188 ; 0,252].27 Fichier associé sur le site www.bordas-indice.fr et sur le

CD-Rom Professeur : 08_TSTMG_exercice27.alg (AlgoBox).1. On complète les deux lignes incomplètes de la façon suivante :n prend la valeur 100 × i ;b prend la valeur p + 1

n.

2. Pour i = 1 : les valeurs affichées sont 0,1 et 0,3.Pour i = 2 : 0,12 et 0,28.Pour i = 3 : 0,14 et 0,26.Ces valeurs s’affichent successivement.3. Pour cela, il faut remplacer la ligne « Pour i allant de 1 à 3 » par la ligne « Pour i allant de 1 à 10 ».28 Voir manuel p. 254.29 1. [0,77 ; 0,97].

2. P(F 0,95) ≈ 0,004.30 Exercice résolu.

31 1. 0,9 − 1n

; 0,9 + 1n

⎡⎣⎢

⎤⎦⎥

.

2. On résout: 2n

0,1. La taille minimale de l’échantillon est 401.

32 1. [0,046 ; 0,136].2. P(F 0,06) ≈ 0,009.

33 Vrai : p p − 1n

; p + 1n

⎡⎣⎢

⎤⎦⎥ .

34 Vrai : L’amplitude de l’intervalle de fluctuation pour unéchantillon de taille n est 2

n.

Si la taille passe de n à 2n, l’amplitude de l’intervalle defluctuation devient 2

2n: ce nombre est bien plus petit que

2n

car 2n n.

P o u r D É M a r r e r1 1. p – 1

n = 0,4 et p + 1

n = 0,6.

2. L’intervalle est [0,4 ; 0,6].2 Voir manuel p. 254.3 a. [0,18 ; 0,22].

b. [0,16875 ; 0,23125].4 a. [0,08 ; 0,12].

b. [0,06875 ; 0,13125].5 [0,6375 ; 0,7625].6 [0,166 ; 0,214].7 [0,201 ; 0,259].8 [0,684 ; 0,748].9 [0,257 ; 0,283].10 [0,498 ; 0,562].11 1. On peut prendre p = 0,5.

2. [0,455 ; 0,545].12 Voir manuel p. 254.13 [0,588 ; 0,652].14 [0,558 ; 0,842].15 1. La fréquence observée dans l’échantillon est :

f = 8981 600

≈ 0,561.

f appartient à [0,555 ; 0,605], donc le maire accepte l’hypothèse au seuil 95 %.2. La fréquence observée dans l’échantillon est :

f ’ = 1 0031 600

≈ 0,627.

f ’ n’appartient pas à [0,555 ; 0,605], donc le maire rejette l’hypothèse au seuil 95 %.16 Voir manuel p. 254.17 1. L’intervalle de fluctuation est :

0,07 − 1320

; 0,07 + 1320

⎡⎣⎢

⎤⎦⎥ ≈ [0,014 ; 0,126].

2. Règle de décision : si la fréquence observée de pommes non conformes appartient à [0,014 ; 0,126], on accepte l’hypothèsep = 0,07 au seuil 95 %, sinon on la rejette.

3. a. f = 9320 ≈ 0,028.

b. On accepte le lot au seuil de confiance 95 %.

18 1. f – 1n

= 0,75 et f + 1n

= 0,85.

2. Intervalle de confiance de p : [0,75 ; 0,85].

19 1. f = 63350

= 0,18.

2. [0,12 ; 0,24].20 Voir manuel p. 254.

Correctif : à la dernière ligne de la page 203, il faut lire « …, alors p appartiendrait à environ 95 % des intervalles de confiance calculés à partir de ces échantillons. »

69Chapitre 8 Échantillonnage et estimation

4. La fréquence observée est 0,98 : elle n’appartient pas à I, donc on rejette l’hypothèse faite au seuil 0,95.40 Voir manuel p. 254.41 Faux : La fréquence observée est f = 163

800 = 0,20375.

Puisque f n’appartient pas à [0,131 ; 0,203], on rejette l’hypothèse que le dé est bien équilibré au seuil 0,95.42 Vrai : L’intervalle de fluctuation est le même si l’on

considère la face numéro 1 ou la face numéro 6. Ici, la fréquence

observée est f ’ = 110800

= 0,1375.

Donc, on accepte l’hypothèse que le dé est bien équilibré au seuil 0,95.43 1. [0,79 ; 0,99].

2. C’est faux.44 1. [0,659 ; 0,741].

2. Oui, c’est possible.45 Voir manuel p. 254.46 [0,427 ; 0,491].47 Fichier associé sur le site www.bordas-indice.fr et sur

le CD-Rom Professeur :08_TSTMG_exercice47.alg (AlgoBox).1. L’algorithme affiche en sortie : a = 0,2 et b = 0,4.2. Cet algorithme détermine un intervalle de confiance à 95 % pour une fréquence observée f d’un caractère dans un échantillon de taille n.3. ProgrammesCalculatrices Texas

Calculatrices Casio

Logiciel AlgoBox

48 Fichiers associés sur le site www.bordas-indice.fr et sur le CD-Rom Professeur :08_TSTMG_exercice48.xls (Excel 2003),08_TSTMG_exercice48.xlsx (Excel 2007),08_TSTMG_exercice48.ods (OpenOffice).1. On saisit en C3 : =C2+50 .

2. On saisit en D2 : =A$2-1/RACINE(C2) .

On saisit en E2 : =A$2+1/RACINE(C2) .49 Exercice résolu.50 1. I = [0,326 ; 0,486].

2. J = [0,387 ; 0,518].

35 1. [0,106 ; 0,174].2. Règle de décision : « Si la fréquence observée des personnes pratiquant le ski dans un échantillon de taille 900 appartient à [0,106 ; 0,174], alors on accepte l’hypothèse p = 0,14 au seuil 95 %, sinon on la rejette. »3. La fréquence observée est 100

900 ≈ 0,111. Donc, on accepte

l’hypothèse.36 1. X suit la loi binomiale de paramètres n = 100 et p = 0,03.

2. Un intervalle de fluctuation à 95 % de la fréquence des Français préférant le rap dans les échantillons de taille 100 est [0 ; 0,07].Remarque : dans ce cas, on ne peut pas utiliser l’intervalle de fluctuation asymptotique, car une condition d’utilisation de cet intervalle n’est pas remplie : n × p = 3, donc n × p 5.3. Règle de décision : « Si la fréquence observée des personnes préférant le rap dans un échantillon de taille 100 appartient à [0 ; 0,07], alors on accepte l’hypothèse p = 0,03 au seuil de confiance 95 %, sinon on la rejette. »

4. La fréquence observée est 6100

= 0,06.

On accepte l’hypothèse au seuil 95 %.37 1. [0,389 ; 0,471].

2. Règle de décision : « Si la fréquence observée des Français de 15 ans et plus n’allant jamais au cinéma dans un échantillon de taille 600 appartient à [0,389 ; 0,471], alors on accepte l’hypothèse p = 0,43 au seuil 95 %, sinon on la rejette. »3. La fréquence observée est 289

600 ≈ 0,482. Donc, on rejette

l’hypothèse.38 Fichiers associés sur le site www.bordas-indice.fr et sur

le CD-Rom Professeur :08_TSTMG_exercice38.xls (Excel 2003),08_TSTMG_exercice38.xlsx (Excel 2007),08_TSTMG_exercice38.ods (OpenOffice).1. On obtient 0 ou 1 de façon équiprobable.2. [0,4 ; 0,6].3. Règle de décision : « Si la fréquence observée du résultat 0 dans un échantillon de 100 nombres appartient à [0,4 ; 0,6], alors on accepte l’hypothèse p = 0,5 au seuil 95 %, sinon on la rejette. »Remarque : le fait que le générateur de nombres aléatoires du tableur fonctionne bien se traduit par l’hypothèse « p = 0,5 ».3. La fréquence observée est 58

100 = 0,58. Donc, on accepte

l’hypothèse.39 1. X suit la loi binomiale de paramètres n = 500 et p = 0,993.

2. a. P(X 491) ≈ 0,010 et P(X 492) ≈ 0,026 donc a = 492.P(X 499) ≈ 0,970 et P(X 500) = 1 donc b = 500.b. Intervalle de fluctuation à 95 % : [0,984 ; 1].Remarque : on ne peut pas utiliser ici l’intervalle de fluctuation asymptotique car les conditions de son utilisation ne sont pas toutes réunies. En effet :n × (1 – p) = 500 × 0,007 = 3,5 et ainsi n(1 – p) 5.3. Règle de décision : « Si la fréquence observée des retraités ayant un téléviseur dans un échantillon de taille 500 appartient à l’intervalle I = [0,984 ; 1], alors on accepte l’hypothèse p = 0,993 au seuil 95 %, sinon on la rejette. »

70

de confiance.b. Ces intervalles de confiance se recouvrent deux à deux partiellement, donc on ne peut pas prévoir l’ordre des trois candidats lors de l’élection.B. Fluctuation des sondages1. On saisit en A2 la formule : =ALEA.ENTRE.BORNES(1;10000) .2. a. X = 3 607 car 1 988 + 1 618 = 3 606 et Y = 5 293, car 3 606 + 1 686 = 5 292.b. On saisit en B2 la formule :

=SI(A2<1989;1;SI(A2<3607;2;SI(A2<5293;3;4))) .

3. a. La formule =NB.SI(B2:B1001;1) compte le nombre de « 1 » dans la plage de cellules B2 :B1001 : on obtient ainsi le nombre de votes pour le candidat J . Chirac dans l’échantillon de 1 000 personnes. En divisant par 1000, on obtient alors la fréquence de votes pour J. Chirac.

b. On saisit en F4 la formule =NB.SI(B2:B1001;2)/1000 , et en

G4 la formule =NB.SI(B2:B1001;3)/1000 .

c. On saisit en E2 la formule =E4-1/RACINE(1000) , puis en E3 la

formule =E4+1/RACINE(1000) . On recopie ensuite ces formules

vers la droite.4. On représente ces trois intervalles de confiance sur un même graphique en sélectionnant la plage de cellules E1:G4.5. a. Pour simuler plusieurs sondages de taille 1 000, on utilise la touche F9 sur Excel et la combinaison de touches Ctrl + Maj + F9 sur OpenOffice.b. On ne peut donc pas prévoir l’ordre des candidats le jour de l’élection au vu de ces simulations.6. Deux simulations représentées ci–dessous montrent que des échantillons semblables (de même taille) conduisent à des résultats très différents sur l’ordre des candidats :

J. Chirac L. Jospin J.-M. Le Pen0,100

0,150

0,200

0,250

J. Chirac L. Jospin J.-M. Le Pen0,100

0,150

0,200

0,250

La critique principale que l’on peut faire à ce sondage, c’est qu’il donne des résultats figés, alors que l’on aurait dû donner des fourchettes d’intentions de vote pour chacun des candidats.

3. L’intersection des intervalles I et J n’est pas vide, donc on peut conclure, au seuil de 95 %, que ces deux commerciaux ont la même efficacité.51 Faux : p peut appartenir à l’intervalle de confiance ou

non ; on sait seulement qu’il appartient à environ 95 % des intervalles de confiance issus d’échantillons différents.52 • Intervalle de fluctuation de la proportion des carpes :

[0,466 ; 0,784].• Intervalle de fluctuation de la proportion des tanches : [0,008 ; 0,326].53 [0,868 ; 0,932].54 [0,69 ; 0,85].55 L’intervalle de fluctuation asymptotique à 95 % est

[0,45 ; 0,55], donc on peut estimer le nombre d’échantillons à 5

100 × 1 000 = 50.

P o u r fa i r e l e P o i n tVoir manuel page 254. Les corrigés détaillés sont disponibles sur le site www.bordas-indice.fr.

acco m Pag n e m e n t P e r s o n n a l i s é56 [0,06 ; 0,16].57 [0,5 ; 0,7].58 [0,612 ; 0,688].59 [0,01 ; 0,23].

t r aVau X P r at i Q u e sTP Sondages à risques

Fichiers associés sur le site www.indice-bordas.fr et sur le CD-Rom Professeur :08_TSTMG_TP.xls (Excel 2003),08_TSTMG_TP.xlsx (Excel 2007),08_TSTMG_TP.ods (OpenOffice),08_TSTMG_TP_correction.xls (Excel 2003),08_TSTMG_ TP_correction.xlsx (Excel 2007),08_TSTMG_ TP_correction.ods (OpenOffice).L’objectif de ce TP est de justifier la phrase de Michel Lejeune, et ainsi de prendre conscience que des résultats de sondage ne peuvent pas être résumés uniquement par des nombres, mais par des intervalles. Le tableur, et plus particulièrement la simulation de plusieurs sondages de taille 1 000 vont illustrer ce phénomène.A. Intervalles de confiance1. • Pour J. Chirac, f = 0,18 ; n = 1000, donc un intervalle de confiance au seuil 0,95 est : I = [0,148 ; 0,212].• Pour L. Jospin, f = 0,17 ; n = 1 000, donc un intervalle de confiance au seuil 0,95 est : J = [0,138 ; 0,202].• Pour J.-M. Le Pen, f = 0,145 ; n = 1 000, donc un intervalle de confiance au seuil 0,95 est : K = [0,113 ; 0,177].2. a. Non, car les pourcentages de voix obtenues par ces trois candidats au 1er tour appartiennent bien à ces trois intervalles

71Chapitre 8 Échantillonnage et estimation

08_TSTMG_ zoomtableur.xlsx (Excel 2007),08_TSTMG_ zoomtableur.ods (OpenOffice).

Sujet AFichiers associés sur le site www.bordas-indice.fr et sur le CD-Rom Professeur :08_TSTMG_sujetA.xls (Excel 2003),08_TSTMG_ sujetA.xlsx (Excel 2007),08_TSTMG_ sujetA.ods (OpenOffice).1. Réponse d.2. Réponse c.3. Réponse c.4. Réponse b.

Sujet B1. a. I = [0,377 ; 0,503].b. I’ = [0,167 ; 0,293].c. La fréquence des femmes dans l’échantillon est :

f = 102255 = 0,4.

La fréquence des compétiteurs dans l’échantillon est :

f ’ = 65255

≈ 0,255.

Puisque f appartient à I et f ’ appartient à I’, l’échantillon est bien représentatif de la population étudiée.2. a. [0,486 ; 0,612].b. La borne inférieure étant inférieure à 0,5, le comité directeur ne peut pas prendre la décision de rénover le restaurant.

3. a. 0,549 − 1n

; 0,549 + 1n

⎡⎣⎢

⎤⎦⎥ .

b. On résout : 0,549 – 1n

≈ 0,5, ce qui donne n 10,049

et n 417.La plus petite valeur de n pour laquelle la décision de rénovation sera prise est 417.

Sujet C1. a. I1 = [0,45 ; 0,55].b. I2 = [0,48 ; 0,52].c. Règle de décision : « Si la fréquence observée de femmes dans un échantillon de 400 salariés de l’entreprise Femmor appartient à [0,45 ; 0,55], alors on accepte l’hypothèse p = 0,5 dans l’entreprise Femmor au seuil 95 %, sinon on la rejette. »d. Règle de décision : « Si la fréquence observée de femmes dans un échantillon de 2 500 salariés de l’entreprise Machard appartient à [0,48 ; 0,52], alors on accepte l’hypothèse p = 0,5 dans l’entreprise Machard au seuil 95 %, sinon on la rejette. »e. La fréquence observée dans l’entreprise Femmor est :

f1 = 182400

= 0,455.

Puisque f1 appartient à I1, on peut dire que Femmor respecte la parité au seuil de confiance 95 %.La fréquence observée dans l’entreprise Machard est :

f2 = 1 1752 500

= 0,47.

Puisque f2 n’appartient pas à I2, on peut dire que Machard ne respecte pas la parité au seuil de confiance 95 %.

P o u r a P P r o f o n D i r60 1. [0,03 ; 0,23].

2. La proportion observée est 0,19 : elle est inférieure à 0,23, donc aucune investigation plus complète ne sera mise en place.3. Soit n la taille de l’échantillon. Alors, un intervalle de fluctuation asymptotique au seuil de confiance 95 % est :

0,13 − 1n

; 0,13 + 1n

⎡⎣⎢

⎤⎦⎥ .

On veut avoir : 0,19 0,13 + 1n

.

Ceci s’écrit : 1n

0,06 et n 1

0,062.

On en déduit : n 278.Il faut prendre au moins 278 sujets.61 Fichier associé sur le site www.bordas-indice.fr et sur

le CD-Rom Professeur :08_TSTMG_exercice61.dfw (Derive).1. X suit la loi binomiale de paramètres n et p = 0,9.2. a. La probabilité de surbooking est :

P(X 300) ≈ 0,0002.b. La probabilité de surbooking est :

P(X 300) ≈ 0,840.3. a. 0,9 − 1

n; 0,9 + 1

n⎡⎣⎢

⎤⎦⎥ .

b. On doit avoir : n 0,9 + 1n( ) 300.

Ceci s’écrit : 0,9n + n 300.c. D’après le logiciel, la valeur maximale de n est 313.62 • Un intervalle de confiance au seuil de 95 % de la

proportion des ampoules défectueuses dans l’entreprise A est :

0,6 − 1100

; 0,6 + 1100

⎡⎣⎢

⎤⎦⎥ = [0,5 ; 0,7].

• Un intervalle de confiance au seuil de 95 % de la proportion des ampoules défectueuses dans l’entreprise B est :

156200

− 1200

; 156200

− 1200

⎡⎣⎢

⎤⎦⎥ ≈ [0,709 ; 0,851].

L’intersection de ces deux intervalles n’est pas vide, donc on peut conclure que la différence entre les durées de vie des ampoules fabriquées par ces entreprises n’est pas significative au seuil de confiance 95 %.63 1. [0,424 ; 0,600].

2. La règle de décision est : « Si la fréquence observée de garçons dans un échantillon de 132 enfants appartient à [0,424 ; 0,600], alors on accepte l’hypothèse p = 0,512 au seuil 95 %, sinon on la rejette. »

Ici, la fréquence est : 46132

≈ 0,348.

On rejette donc l’hypothèse p = 0,512 : la différence observée avec la proportion de naissances de garçons au Canada est significative.

C a P v e r s l e B aC

Zoom sur le tableur

Fichiers associés sur le site www.bordas-indice.fr et sur le CD-Rom Professeur :08_TSTMG_zoomtableur.xls (Excel 2003),

72

ampoule de type A », avec la probabilité 14

, et « obtenir une ampoule de type B ». Donc, la variable aléatoire X égale au nombre d’ampoules de type A dans cet échantillon suit la loi binomiale de paramètres n = 100 et p = 1

4.

b. P(X = 25) ≈ 0,092.c. P(X 25) = P(X 24) ≈ 0,462.2. a. I = [0,15 ; 0,35].b. Règle de décision : « Si la fréquence observée d’ampoules de type A dans un échantillon de 100 ampoules appartient à l’intervalle I, alors on accepte l’hypothèse p = 0,25 au seuil 95 %, sinon on la rejette. »c. La fréquence observée est : f = 38

100 = 0,38.

Puisque f n’appartient pas à I, on rejette l’hypothèse. La répartition prévue des ampoules n’est pas respectée au seuil de confiance 0,95.

Sujet G1. a. I = [0,518 ; 0,582].b. Règle de décision : « Si la fréquence observée des électeurs désirant voter pour Mme Marin dans un échantillon de taille 1 000 appartient à I, alors on accepte l’hypothèse p = 0,55 au seuil 95 %, sinon on la rejette. »c. La fréquence observée est : f = 520

1 000 = 0,52. Puisque fappartient à I, on accepte l’hypothèse de Mme Marin au seuil de confiance 95 %.2. a. P(F 0,5) ≈ 0,525.b. P(F 0,52) ≈ 0,095.

64 [0,054 ; 0,279].65 [0,558 ; 0,842].66 [0,522 ; 0,589].

2. a. Arbre pondéré :

0,47

0,455

0,545

0,53

0,47

0,53

MF

M

M

MF

b. P(F M) = 0,455 × 0,47 ≈ 0,214.c. P(

–F M) + P(F

–M) ≈ 0,497.

Sujet D1. I = [0 ; 0,062].2. J = [0,018 ; 0,082].3. I J n’est pas vide, donc la différence n’est pas significative au seuil 95 %.

Sujet E1. [0,06 ; 0,26].2. Règle de décision : « Si la fréquence observée d’hypertendus dans un échantillon de 100 habitants de la région appartient à l’intervalle I = [0,06 ; 0,26], alors on accepte l’hypothèse p = 0,16 au seuil 95 %, sinon on la rejette. »3. Puisque f appartient à I, on accepte l’hypothèse p = 0,16 au seuil de confiance 95 %.

Sujet F1. a. L’épreuve consistant à prélever une ampoule parmi 100 ampoules se répète 100 fois de façon identique et indépendante  : elle conduit à deux issues, « obtenir une

73Ensembles – Raisonnement logique

Ensembles Raisonnement logique

1 E = {–2 ; – 1 ; 0 ; 1 ; 2 ; 3 ; 4 ; 5 ; 6 ; 7}a. Faux ; b. Vrai ; c. Vrai ; d. Faux ; e. Faux ; f. Faux.

2 F= { 0 ;4 ;8 ;12} . On n’a pas E � F.

3 a. Vrai ; b. Faux ; c. Faux ; d. Vrai.

4 a. 3, 4, 7, 6, 13, 9, 17 sont des entiers impairs ouinférieur à 10.b. 2, 3, 9, 21 sont des entiers multiples de 3 ou inférieurs à 10.c. 14, 28, 42 sont des entiers divisibles par 7 et par 2.

5 a. I � J = [2,5 ; 4] et I � J = ]0 ; 7].b. I � J = [– 2 ; 8] et I � J = ]– 4 ; + �[.c. I � J = [– 1 ; 6[ et I � J = [– 3 ; 8].

6 a. ]3 ; + �[ � ]–� ; – 2].b. ]5 ; + � [ � ]– � ; – 1].

7 1. A � B = { 0 ; 1 ; 2 ; 3 ; 4 ; 5 ; 7 ; 11} ;A � B = {1 ; 3} ;B � C = {0 ; 7};B � C = {0 ; 1 ; 2 ; 3 ; 4 ; 7 ; 11 ; 13}.2. A � B � C = ∅A � B � C = {0 ; 1 ; 2 ; 3 ; 4 ; 5 ; 7 ; 11 ; 13}.

8 1. Faux. Contre-exemple : x = 8.2. Vrai : x = 1,5.3. Faux. Contre-exemple : x = – 4.4. Vrai : la fonction « carré » sur l’intervalle [– 3 ; – 1].5. Vrai : par exemple a = 0 et b = 1.6. Faux. Contre-exemple : a = 1 et b = 1.7. Faux. Contre-exemple : x = – 1.

9 1. Cet énoncé est vrai.Énoncé réciproque : « Si le pourcentage des � lles dans une classe est de 50 %, alors il y a autant de garçons que de � lles dans cette classe ». Cet énoncé est vrai. Il y a donc équivalence.2. Cet énoncé est faux.Énoncé réciproque : « Si x = 2, alors x2 = 4 ». Cet énoncé est vrai. Il n’y a donc pas équivalence.

3. Cet énoncé est vrai.Énoncé réciproque : « Si x2 � 4, alors x � – 2 ». Cet énoncé est faux. Il n’y a donc pas équivalence.4. Cet énoncé est vrai.Énoncé réciproque : « Si la droite (d) passe par le point A(– 1 ; 1) alors la droite (d) a pour équation y = 2x + 3 ».Cet énoncé est faux. Il n’y a donc pas équivalence.5. Cet énoncé est faux.Énoncé réciproque : « Si f est une fonction constante sur �, alors f (1) = 2 et f (3) = 2 ».Cet énoncé est faux. Il n’y a pas équivalence.6. Cet énoncé est vrai.Énoncé réciproque : « Si une équation du second degré a deux solutions dans �, alors son discriminant est égal à 5 ». Cet énoncé est faux. Il n’y a pas équivalence.7. Cet énoncé est vrai.Énoncé réciproque : « Si le salaire annuel d’un employé est 24 000 euros, alors son salaire mensuel moyen est 2 000 euros. ». Cet énoncé est vrai. Il y a donc équivalence.

10 1. (E2) : « Si x 2 ≠ 25, alors x ≠ 5 » ; (E3) : « Si x 2 = 25, alors x = 5 ».Les énoncés (E1) et (E2) sont vrais, (E3) est faux.2. (E2) : « Si la droite (d1) n’a pas pour équation y = – 5x + 7, alors (d1) n’est pas parallèle à la droite (d2) d’équation y = – 5x + 3 ».(E3) : « Si la droite (d1) a pour équation y = – 5x + 7, alors (d1) est parallèle à la droite (d2) d’équation y = – 5x + 3 ».Les énoncés (E1) et (E2) sont faux ; (E3) est vrai.3. (E2) : « Si deux notes ne sont pas toutes les deux égales à 12, alors la moyenne de ces deux notes n’est pas égale à 12 » ;(E3) : « Si deux notes sont égales à 12, alors la moyenne de ces deux notes est égale à 12 ».Les énoncés (E1) et (E2) sont faux ; (E3) est vrai.

74

Fonctionnalités de base du tableur

Fichiers associés sur le site www.bordas-indice.fr et sur le CD-Rom Professeur :TSTMG_� cheslogiciels.xls (Excel 2003)TSTMG_� cheslogiciels.xlsx (Excel 2007)TSTMG_� cheslogiciels.ods (OpenO� ce)TSTMG_� cheslogiciels_corrections.xls (Excel 2003)TSTMG_� cheslogiciels_corrections.xlsx (Excel 2007)TSTMG_� cheslogiciels_corrections.ods (OpenO� ce)

Outils les plus utilisés1 Exercice résolu.

2 Dans la cellule A2, on saisit la formule =A1+2 , puis on recopie vers le bas jusqu’à la cellule A50.

3 Dans la cellule B1, on saisit la formule =A1+25 , puis on recopie vers la droite jusqu’à la cellule CC1.

4 1. On entre 2008 dans la cellule A2, puis dans la cellule A3 on saisit la formule =A2+1 que l’on recopie vers le bas jusqu’à la cellule A20.2. Dans la cellule B3, on saisit la formule =B2+650 .3. Dans la cellule C3, on saisit la formule =C2*1,03 .4. Dans la cellule D2, on saisit la formule =B2/C2 .5. Voir � chier corrigé.

5 1. Formule en B6 : =B4*B5/100 . Cela correspond à la consommation journalière du véhicule en ville lorsque la consommation pour 100 kilomètres est saisie en B4 et la distance parcourue par jour est saisie en B5.2. En B8 : =B6+C6 .En B9 : =B8*B2 .En B10 : =B9*B1 .3. Voir � chier corrigé4. Le budget carburant serait de 831 euros.

Fonctions d’un tableur6 Exercice résolu.

7 a. La formule =Max(A1:C5) détermine le plus grand nombre de la zone de cellules A1:C5 : il s’agit ici de 5.b. La formule = MIN(A1:B6) détermine le plus petit nombre de la zone de cellules A1:B6 : il s’agit ici de 2.c. La formule =SOMME(A1:A7) calcule la somme des nombres situés dans la zone de cellules A1:A7 : il s’agit ici de 17.

d. La formule =MOYENNE(A2:C6) calcule la somme moyenne des nombres situés dans la zone de cellules A2:C6 : il s’agit ici de 2.

8 a. =ALEA.ENTRE.BORNES(1;6) .b. =ALEA.ENTRE.BORNES(1;2013) .

9 1. On entre 0 dans la cellule A2, on saisit la formule =A2+1 dans la cellule A3, puis on recopie cette formule vers le bas jusqu’à la cellule A32.2. Dans la cellule B2, on saisit la formule =LOI.BINOMIALE(A2;30;0,3;0) .

10 1. En G2, on peut saisir la formule =MIN(B2:E7) .En G3, on peut saisir la formule =MAX(B2:E7) .En G4, on peut saisir la formule =MOYENNE(B2:E7) .2. Voir � chier corrigé.

11 1. En H1, on peut saisir la formule =B1−D1 .2. Dans la cellule C3, on peut saisir la formule =B3+4 .3. Dans la cellule M3, on peut saisir la formule =H1−SOMME(B3:L3) .

4. Voir � chier corrigé. On peut observer que la dernière mensualité sera d’un montant de 40 €.

12 1. En B2, il faut saisir la formule =ALEA.ENTRE.BORNES(0;1) (réponse a.).

2. Dans la cellule G2, on peut saisir la formule =SOMME(B2:F2) .

3. Dans la cellule G6, on peut saisir la formule=SOMME(G2:G5) ; le contenu de la cellule G7 s’obtient

alors avec la formule =G6/4 .4. Voir � chier corrigé.5. Pour e� ectuer d’autres simulations, il su� t d’utiliser l’instruction de « recalcul ».Pour cela :– sur Excel, on utilise la touche F9 ;– sur OpenO� ce, on utilise la combinaison de touchesCtrl + Maj + F9 .

13 1. Voir � chier corrigé.2. a. – Pour obtenir les consommations minimales pour chaque famille, en B5, on peut saisir la formule =MIN(B2:B4) . Il su� t ensuite de recopier cette formule

en C5 et en D5.– De même, la formule =MAX(B2:B4) , saisie en B6et recopiée en C6 et en D6, permet d’obtenir les consommations maximales pour chaque famille.

75Fonctionnalités de base du tableur

– En� n, la formule =MOYENNE(B2:B4) , saisie en B7et recopiée en C7 et en D7, permet d’obtenir les consommations moyennes pour chaque famille.b. On utilise les mêmes instructions que celle de la question précédente. La recopie des formules étant cette fois verticale :

14 1. En E2, on peut saisir la formule =B2+C2*D2 .2. Dans la cellule E7, on saisit la formule =SOMME(E2:E6) .

Dans la cellule E8, on saisit la formule =MIN(E2:E6) .Dans la cellule E9, on saisit la formule =MAX(E2:E6) .Dans la cellule E10, on saisit la formule =MOYENNE(E2:E6) .

3. Voir � chier corrigé.

15 Exercice résolu

16 1. Dans la cellule A5, on saisit la formule =A4+0,5 .2. Dans la cellule B4, on peut saisir la formule =A4^2+B$1*A4+3 .

17 Dans la cellule D4, le professeur d’EPS peut saisir la formule =D$1/(B4+C4/60) .

18 – On entre 185 dans la cellule A2.– On entre 1 dans la cellule B3, on saisit la formule

=B3+1 dans la cellule B4, puis on recopie cette formule vers le bas.– On entre 1 000 dans la cellule C2, on saisit la formule =C2+A$2 dans la cellule C3, puis on recopie cette

formule vers le bas.

19 1. Dans la cellule C1, on saisit la formule =B1+5 .2. Dans la cellule B2, on peut saisir la formule =$B3*B1^2 .

3. Pour calculer la distance de freinage dans la cellule B2, on doit faire le produit du contenu de la cellule B3 par le carré du contenu de la cellule B1. La formule =B3*B1^2

se recopierait vers la droite en =C3*C1^2  : il faut donc � xer le nom de la colonne de la cellule B3 par $. La formule =$B3*B1^2 qui se recopie vers la droite en

=$B3*C1^2 convient bien.

20 Dans la cellule B3, on saisit la formule =$A3*B$2 , puis on recopie vers le bas et vers la droite a� n de compléter la plage B3:D7.

21 1. On entre 1 dans A2, on saisit la formule =A2+1

dans la cellule A3, puis on la recopie vers le bas.2. On entre 1 dans B1, on saisit la formule =B1+1

dans la cellule C1, puis on la recopie vers la droite.3. a. Dans B2, on entre la formule =$A2^2+B$1^2 .b. Grâce aux dollars, la formule précédente se recopie correctement dans la plage de cellules B3:B51.c. Voir � chier corrigé.

22 1. Dans la cellule D5, on saisit la formule =B5*C5 .Dans la cellule E5, on saisit la formule =D5*D$1 .2. Dans la cellule D10, on saisit la formule

=SOMME(E5:E8) .3. Dans la cellule D11, on saisit la formule =SOMME(D5:D8)+D10 .

23 1. Dans la cellule E2, on saisit la formule =SOMME(B2:D2) .

2. Dans la cellule E4, on saisit la formule =(B4*B$2+C4*C$2+D4*D$2)/E$2 .

24 Exercice résolu.

25 1. =A2+1 .2. =150+SI(A2<10;A2*40;A2*60) .

26 Exercice résolu.

27 On saisit en D2 la formule =NB.SI(B2:B16;"F") .

28 1. =A2+1 .2. On saisit en B3 la formule =ALEA.ENTRE.BORNES(1;6) , puis on la recopie vers le bas.3. C’est la formule donnée en a..

29 1. On saisit la formule =ALEA.ENTRE.BORNES(1;6)

dans la cellule A2.2. On saisit =ALEA.ENTRE.BORNES(1;6) en B2, puis =A2+B2 dans la cellule C2.

3. On recopie vers le bas la zone A2:C2 jusqu’à la ligne 51.4. On entre les nombres 2, 3, 4, …, 12 dans la zone de cellules E1:O1.5. On saisit en E2 la formule =NB.SI($C2:$C51;E1) .6. =NB.SI(C2:C51;">5") .

N° d’éditeur : 10192849Dépôt légal : Juillet 2013

Imprimé en France par Maury